Math Workbook
Math Workbook
Exercise Book
Practice Problems
Concept Summaries
Algebra Review
Very Detailed Solution To Each Problem
by Paul Tsopméné
Source File. To get the source file, please get in touch with the
author at the following email address: [email protected]
iii
For Instructors
Dear instructors,
Thank you for your interest in this open exercise book. I hope you find it a
valuable addition to your existing teaching material.
I would love to hear your feedback and adoption decision for your course.
Your feedback will be highly appreciated and will be used for future im-
provement of the book.
Please take a few minutes to fill in the survey, which can be accessed in the
following way:
Survey Link
Thank you for your time and support on this open exercise book develop-
ment.
Best Regards,
Paul Tsopméné
Preface ix
I Exercises 1
1 Linear Functions 3
1.1 Slopes and Equations of Lines . . . . . . . . . . . . . . . . . . . . . . . . . 3
1.2 Linear Functions and Applications . . . . . . . . . . . . . . . . . . . . . . . 3
4 Differentiation Rules 31
4.1 Basic Rules of Differentiation . . . . . . . . . . . . . . . . . . . . . . . . . 31
4.2 The Product and Quotient Rules . . . . . . . . . . . . . . . . . . . . . . . 32
4.3 The Chain Rule . . . . . . . . . . . . . . . . . . . . . . . . . . . . . . . . . 33
4.4 Derivatives of Exponential Functions . . . . . . . . . . . . . . . . . . . . . 34
4.5 Derivatives of Logarithmic Functions . . . . . . . . . . . . . . . . . . . . . 35
4.6 Derivatives of Trigonometric Functions . . . . . . . . . . . . . . . . . . . . 36
4.7 Exponential Growth and Decay . . . . . . . . . . . . . . . . . . . . . . . . 37
v
vi CONTENTS
5.3 Concavity . . . . . . . . . . . . . . . . . . . . . . . . . . . . . . . . . . . . 43
5.4 Curve Sketching . . . . . . . . . . . . . . . . . . . . . . . . . . . . . . . . . 44
II Solution to Exercises 53
1 Linear Functions 55
1.1 Slopes and Equations of Lines . . . . . . . . . . . . . . . . . . . . . . . . . 55
1.2 Linear Functions and Applications . . . . . . . . . . . . . . . . . . . . . . . 60
The author wrote this exercise book when teaching MATH 116 (Calculus I for Management
and Economics) at the University of British Columbia Okanagan. The book is primarily
for students taking a first-year differential calculus course (with business and economics
applications). Students taking a general Calculus I course can also use it to get down basic
content in differential calculus.
The most important way to learn calculus is through problem-solving. While going
through the solution to a problem, students are often faced with several issues. They may
not see the connection between the concept taught in class and the solution. Others may
not understand the solution because a step is missing or there are not enough explanations.
Or because they have weak algebra skills. I think that nearly everybody agrees that the
latter issue is the main reason many students struggle with calculus.
My main goal in this book is to address these issues to help students learn the material
more efficiently and get better results. To that end, I have included the following features.
Concept Summaries Boxes: They allow students to review the material and make
the book self-contained. Each concept involved in a problem is summarized in a box
right before the solution. The relevant information needed to solve the problem is
given. And the solution clearly shows how that information is used.
“Help Texts”: They are located to the right of the solution sequence and help
students recognize which rule, property, or procedure is being applied.
Algebra Review: They help students understand the solution, especially those with
weaker algebra skills. Basic algebra rules and procedures (exponent rules, adding
fractions, factoring, etc.) are listed next to the steps within solutions. Also, not only
the calculus steps are shown in the solution, but also the algebra steps. The idea is
that students with a poor algebra background can just jump in and learn as they go.
The book is divided into two parts. The first part includes a variety of problems as well
as applications in business and economics. The idea is not to have as many problems as
in a traditional textbook but a fair amount of problems that cover essential/basic skills in
differential calculus. The second part includes full solutions to every problem. Hyperlinks
ix
x Preface
are added to ease the navigation of the book (they are mainly used to go back and forth
between the two parts).
To the Student: If you try to solve a problem and have no idea what to do, you can
look at the solution and try to understand it (if there is a box, you should first try to
read and understand its content). Try to solve the subsequent few problems yourself before
checking the solution. It’s not a big deal if you make a mistake; you will learn better from
that mistake. If your final answer does not match the one in the solution, try to see if it
is equivalent (since there are often several different
√
ways of expressing the answer). For
x
example, if the answer given in the book is x and you obtain √1x , you are right because
√
x
rationalizing √1 will give you .
x x
PartI
Exercises
1
Chapter1
Linear Functions
(a) (1, 2) and (4, 11) (e) (−2, −3) and (0, 6)
(b) (0, 1) and (2, 9) (f ) (−15, −14) and (−7, −2)
(c) (5, 6) and (6, 4) (g) (1, 1) and ( 31 , 34 )
(d) (10, 13) and (7, 19) (h) ( 21 , 0) and (− 34 , 45 )
2. Find the value of x such that the slope of the line through (−1, 1) and (2, 4x) is 2.
[Solution on page 57]
3. Find an equation of the line through the given point with slope m. [Solution on
page 57]
1
(a) (2, 3) and m = 4 (b) (0, 1) and m = 3
4. Find an equation of the line through the given points. [Solution on page 58]
(a) x = 2 (c) y = 2x − 1
(b) y = 1 (d) y = −x + 4
3
4 Chapter 1. Linear Functions
(a) Find the slope of the sales line, and give an equation for the line in the form
y = mx + b.
(b) Use your answer from part (a) to find out how many years must pass before the
sales surpass $48,500.
Let x represent the number of hours studied and let y represent the grades on exam.
[Solution on page 61]
(a) Plot the date. Are the data exactly linear? Could the data be approximated by
a linear equation?
(b) Use the points (1, 10) and (5, 20) to determine a linear equation (of the form
y = mx + b) that models the data. What does the slope of the graph of the line
indicate?
(c) Use the equation in part (b) to estimate the grade of a student who studied 6
hours.
4. Let f (x) = 5 − 3x and g(x) = 34 x + 7. Find the following. [Solution on page 62]
√
(a) f (2) (c) f 2
(b) g 13
p = D(q) = 10 − 0.5q
where p is the price expressed in dollars and q is the quantity demanded (in thou-
sands).
(a) Find the price for each quantity. (i) q = 0 shoe; (ii) q = 5000 shoes.
(b) Find q for each price. (i) p = $4; (ii) p = $9.
(c) Suppose the supply function is given by p = S(q) = 0.25q + 1. Graph the supply
and demand functions on the same axes.
(d) Find the equilibrium quantity and price.
1.2. Linear Functions and Applications 5
5. The revenue from producing x units of a product is given by R(x) = −x2 + 26x, while
the cost in dollars is given by C(x) = 7x + 48 (revenue and cost are in thousands of
dollars). [Solution on page 74]
(a) Find the minimum break-even quantity.
7
8 Chapter 2. Quadratic, Exponential, and Logarithmic Functions
6. Suppose the revenue and cost functions (in thousands of dollars) are given by
x2 35
R(x) = − + 5x and C(x) = x +
3 3
[Solution on page 75]
(a) The exponential function f (x) = ax is defined for every real number x.
(b) The function f (x) = ax is increasing when 0 < a < 1.
(c) The function f (x) = ax is decreasing when a > 1.
(d) The graph of f (x) = ax intersects the x-axis.
(e) The y-intercept of f (x) = ax is 1.
(f ) It is possible to find x such that ax ≤ 0.
4. Investing $2000 at a rate of 12% compounded annually, how much would you have
after 5 years? [Solution on page 79]
5. Investing $7000 at a rate of 4% compounded quarterly, how much would you have
after 3 years? [Solution on page 80]
6. If x is a positive number (x > 0), show that log9 x = 21 log3 x. [Solution on page 86]
11. Suppose you invest $1000 at a rate of 4%. After t years your amount on deposit is
the double of the initial amount. Find t in each case. [Solution on page 90]
12. Suppose you invest a certain amount of money. After t years your amount on deposit
is A(t) = 400 ln(t + 4), where A is in millions of dollars. [Solution on page 91]
2. Let f be the function whose graph is shown in Figure 3.1. Use the graph to state the
value of each quantity, if it exists. If it does not exist, explain why. [Solution on
page 94]
−2 2 4 6 x
−2
Figure 3.1
11
12 Chapter 3. Limits and Derivatives
3. Let f be the function whose graph is shown in Figure 3.2. Use the graph to state the
value of each quantity, if it exists. If it does not exist, explain why. [Solution on
page 94]
(a) lim− f (x) (c) lim f (x) (e) lim f (x) (g) lim f (x)
x→2 x→2 x→6 x→−1+
−2 2 4 6 8 x
−1
−2
Figure 3.2
4. Let f be the function whose graph is shown in Figure 3.3. Use the graph to state the
value of each quantity, if it exists. If it does not exist, explain why. [Solution on
page 95]
(a) lim f (x) (c) lim f (x) (e) lim f (x) (g) lim− f (x)
x→−1− x→−1 x→3 x→6
5. Let f be the function whose graph is shown in Figure 3.4. State the equations of the
vertical asymptotes. [Solution on page 95]
9x2 −4
(b) lim x5 − 4x3 − 7x + 4 (d) lim
x→2 x→1 x+1
3.2. Finding Limits Using Rules and One-Sided Limits 13
−4 −2 2 4 6 x
Figure 3.3
−1 1
Figure 3.4
x4√
−3x2 +1 3 +x
(e) lim x+98
(h) lim e−2x
x→2 x→0
2
√
3
(f ) lim (x −3x−8)
x−1
2x−3
x→5
√
x2 +x
(g) lim x2 − 16 (i) lim1 ln 2x+3
x→3 x→ 2
x2 +3x−10 x2 −x
(d) lim 2 (i) lim 3 x
x→−5 x +6x+5 x→0
1
2x2 −x−6 − 18
(e) lim 2 (j) lim x
x→2 x +2x−8 x→8 x−8
x→4 x−2
x→0
√ √
(c) lim x−5 (h) x−2
lim −3x+11
x→25 x−25 x→4
√ √ √
(d) lim x3− x
2 −81 (i) lim 2x+5− x+7
x→9 x→2 x−2
√
(e) lim x4−2 x
2 −x−12 (j) lim √ x−3
x2 −5x+10−2
x→4 x→3
5. Let
x2 + x + 1 if x ≤ 1 −3 − x if x < −2
f (x) = and g(x) =
x+2 if x > 1 x2 − 10 if x ≥ −2
(a) Find f (−2) (c) Find f (2) (e) Find lim f (x)
x→1
6. Let
x − 5 if x < −2
f (x) = 0 if −2 ≤ x < 4
x2
2
− 8 if x ≥ 4
[Solution on page 109]
3.3. Infinite Limits and Vertical Asymptotes 15
4−x
2. Find lim 2 [Solution on page 114]
x→2 x −4x+4
−5
3. Find lim 2 [Solution on page 115]
x→−2 (x+2)
1
4. Find lim 2 [Solution on page 116]
x→0 x
x2 +3x−4
5. Find lim 2 [Solution on page 116]
x→1 x −2x+1
7x
6. Let f (x) = (x−5)3
. [Solution on page 117]
√1 2x3 −x2 +3
(c) lim (h) lim 2 +3x−1
x→∞ x x→∞ −4x
x2 +1 x7 −x3 +1
(d) lim 2 (i) lim 4 2
x→∞ 3x +x+1 x→−∞ −x +3x +8
√
7x3 x+x2
(e) lim 3 (j) lim 2x−x
x→−∞ −4
x 2
x→∞
5. In each case, find the horizontal asymptotes (if any). [Solution on page 126]
4x3 +1 x3
(a) f (x) = x3 −x
(c) f (x) = x2 +1
x
(b) f (x) = x2 +1
3.5 Continuity
1. Let f be the function whose graph is shown in Figure 3.5. Find all the points of
discontinuity. Verify that each point does not satisfy the definition of continuity.
[Solution on page 127]
y
6
−8 −6 −4 −2 2 4 6 8 x
−1
−2
Figure 3.5
3. In each case, determine if the function is continuous at the given point. [Solution on
page 129]
x + 2 if x < 0
(a) f (x) = point: x = 0
x2 if x > 0
2x − 1 if x ≤ 1
(b) f (x) = x2 point: x = 1
x+1
if x > 1
x√2 + 2x if x < −2
(c) f (x) = point: x = −2
x + 3 if x ≥ −2
e2x
if x < 0
(d) f (x) = 2 if x = 0 point: x = 0
3
x + 1 if x > 0
18 Chapter 3. Limits and Derivatives
x2 −2x−3
−2x+6
if x < 3
(e) f (x) = point: x = 3
−2 + ln(x − 2) if x ≥ 3
ex−1
2
if x ≤ 1
(f ) f (x) = √
point: x = 1
x−1
x−1
if x > 1
5. In each case, find all constants k such that the function is continuous at the given
point. [Solution on page 132]
kx2 − 6 if x ≤ 2
(a) f (x) = point: x = 2
3x + k if x > 2
2x2 −3x−5
if x 6= −1
(b) f (x) = x+1 point: x = −1
kx + 5 if x = −1
−2x3 − 4x if x < 1
(c) f (x) = k 2 − 5k if x = 1 point: x = 1
−6x if x > 1
6. Let f be the function whose graph is shown in Figure 3.5. Determine if f is continuous
on the following intervals: [Solution on page 134]
(−7, −5), (−6, −4), [0, 2], [5, 6], [4, 6], and [3, 4].
√
7. Show that the function f (x) = 9 − x2 is continuous on the interval [−3, 3]. [Solution
on page 135]
9. Show that the function f (x) = |x| is continuous for all x. [Solution on page 136]
3.6. Rates of Change 19
x + 2 if x ≤ k
10. Let f (x) = Find all constants k that make f continuous on
x2 if x > k.
R = (−∞, ∞). [Solution on page 136]
11. Find the intervals where each function is continuous. [Solution on page 137]
√
(a) f (x) = x5 − 3x4 + 7x − 1 (c) h(x) = 2x − 6
2x
(d) k(x) = e
x2
(b) g(x) = 2 (e) l(x) = ln(−3x + 12)
x − 16
12. Find all values x = c where the function is discontinuous. [Solution on page 138]
2x + 1 |x − 6|
(a) f (x) = (c) f (x) =
(2x − 3)(x + 7) x−6
x3
(b) f (x) = (d) f (x) = x4 − x + 1
x2 − 3x + 2
14. Show that the equation x3 + x − 1 = 0 has a solution between 0 and 1. [Solution on
page 139]
15. Show that the equation x5 − 2x2 = 7 has a solution between −1 and 2. [Solution on
page 139]
2. In each case, find the instantaneous rate of change for f (x) when x = a. [Solution
on page 142]
3. A car is moving along a straight road. Suppose its position (in kilometers) is given
by s(t) = 25t2 , where t is measured in hours. [Solution on page 143]
4. An object moves along a straight line. Its position in feet at time t is given by
1
s(t) = t2 − 5t + 19,
3
where t is measured in seconds. [Solution on page 144]
5. Suppose that the cost in dollars of producing x items of a certain commodity is given
by
C(x) = 200 + 21x − x2 (0 ≤ x ≤ 10)
[Solution on page 146]
(a) Find the average rate of change of cost if the number of items produced changes
from 2 to 3.
(b) Suppose the production level is x = 2 items. Find the cost of producing one
additional item. Compare your answer to the one found in part (a).
(c) Find the instantaneous rate of change of cost when 2 items are produced. Com-
pare your answer to the one found in part (b).
6. Suppose that the profit (in thousands of dollars) from selling x items of a certain
product is given by
P (x) = 3x2 − 9x + 8
[Solution on page 147]
3.7. Definition of the Derivative 21
(a) Find the average rate of change of profit when the number of items sold changes
from 1 to 2.
(b) Find the average rate of change of profit when the number of items sold changes
from 2 to 5.
(c) Find the instantaneous rate of change of profit when x = 1. Interpret your
answer.
(d) Find and interpret the instantaneous rate of change of profit when x = 5.
−2 −1 1 2 3 4 x
−2
P1 −4
−6
Figure 3.6
(a) The tangent line to the curve at P1 (−1, −4) passes through the points (−1, −4)
and (0, 5). Find the derivative f 0 (−1).
(b) Find the derivative f 0 (1) and the equation of the tangent line at (1, −2).
(c) Find the points where the derivative is zero.
(d) Is the derivative f 0 (3) negative? Justify your answer.
2. Let f be the function whose graph is shown in Figure 3.7. Find f 0 (1). [Solution on
page 150]
22 Chapter 3. Limits and Derivatives
1 x
−1
Figure 3.7
3. Find an equation of the tangent line to f (x) = −x2 at (1, −1). [Solution on
page 151]
√
4. Find an equation of the tangent line to f (x) = x at (4, 2). [Solution on page 152]
5. Let f (x) = −3x2 + 4x. Find the derivative of f at a = 2 using the limit definition of
derivative. [Solution on page 153]
8. Give the derivative of each function just by considering the slope of the tangent line.
[Solution on page 155]
9. In each case, find f 0 (x) by using the definition of the derivative. [Solution on
page 156]
√
(a) f (x) = k, where k is a (c) f (x) = x2 (e) f (x) = x
constant.
1
(b) f (x) = x (d) f (x) = x3 (f ) f (x) =
x
10. In each case, find f 0 (x) by using the definition of the derivative. Then find f 0 (−3), f 0 (0),
and f 0 (2) (if the derivative exists). [Solution on page 158]
3.8. Graphical Differentiation 23
12. Consider the function f whose graph is shown in Figure 3.8. [Solution on page 162]
y
7
−6 −5 −4 −3 −2 −1 1 2 3 4 5 6 x
−1
−2
Figure 3.8
−3 3 x
2. Let f be the function whose graph is shown below. Sketch the graph of the derivative
of f . [Solution on page 165]
−3 −2 −1 1 2 3 x
−1
3. Sketch the graph of the derivative of the following function. [Solution on page 165]
y
3
2
1
−3 −2 −1 1 2 3 x
−1
−2
−3
4. Sketch the graph of the derivative of the following function. [Solution on page 166]
3.8. Graphical Differentiation 25
4
3
2
1
−1 1 2 3 4 5 x
−1
5. Sketch the graph of the derivative of the following function. [Solution on page 170]
3
2
1
−3 −2 −1 1 2 x
−1
−2
6. Sketch the graph of the derivative of the following function. [Solution on page 171]
y
4
3
2
1
−1 1 2 3 4 x
−1
−2
7. Sketch the graph of the derivative of the following function. [Solution on page 172]
26 Chapter 3. Limits and Derivatives
y
4
−4 −2 2 4 x
−2
−4
8. Sketch the graph of the derivative of the following function. [Solution on page 173]
y
4
−2 2 4x
−2
9. Sketch the graph of the derivative of the following function. [Solution on page 174]
y
4
−6 −4 −2 2 x
10. Sketch the graph of the derivative of the following function. [Solution on page 175]
3.8. Graphical Differentiation 27
y
3
−5 5 x
11. Let f be the function whose graph is shown below. [Solution on page 176]
y
4
−2 2 4 6 x
12. Let f be the function whose graph is shown below. [Solution on page 178]
−4 −2 2 4x
−2
−4
13. Shown is the graph of a function f and its derivative f 0 . Decide which graph is that
of f and which is the graph of f 0 . Justify your answer. [Solution on page 178]
(b) 10 (a)
−2 2 4 6x
−5
14. Shown is the graph of a function f and its derivative f 0 . Decide which graph is that
of f and which is the graph of f 0 . Justify your answer. [Solution on page 179]
y
(a) 5 (b)
−4 −2 2 x
−5
−10
−15
15. Consider the graphs in (a)–(d). Match each graph with the graph of its derivative in
(i)–(iv). [Solution on page 179]
y y
(a) (b)
x x
3.8. Graphical Differentiation 29
y y
(c) (d)
x x
y y
(i) (ii)
x x
y y
(iii) (iv)
x x
16. Consider the graphs in (a)–(d). Match each graph with the graph of its derivative in
(i)–(iv). [Solution on page 180]
y y
(a) (b)
x x
y y
(c) (d)
x x
30 Chapter 3. Limits and Derivatives
y y
(i) (ii)
x x
y y
(iii) (iv)
x x
17. Let P (x) be the profit in thousands of dollars from selling x × 100 units of a certain
product. Shown is the graph of P . [Solution on page 180]
1 2 3 4 x
(a) Find the interval where the rate of change of profit (also called the marginal
profit) is positive.
(b) Find the rate of change of profit at the level of production of 3 × 100 (x = 3)
units.
(c) Sketch the graph of the marginal profit. What does the graph tell you?
Chapter4
Differentiation Rules
3. Find an equation of the tangent line to the curve y = 2x3 −x2 +2 at the point P (1, 3).
[Solution on page 189]
4. Find the points on the curve y = x3 − 3x + 1 where the tangent line is horizontal.
[Solution on page 190]
5. The cost (in dollars) of producing x units of a certain commodity is
C(x) = 3x2 + 75x + 400
[Solution on page 190]
31
32 Chapter 4. Differentiation Rules
Also suppose that the demand function is given by p = −0.01q + 60 (where p is the
price per unit).
x3 −x
3. Find an equation of the tangent line to the curve y = x2 +1
at the point P (1, 0).
[Solution on page 200]
4. Suppose the cost (in dollars) of producing x units of a certain product is given by
C(x) = 4000 + 3x. [Solution on page 201]
(a) Find the average cost for each production level. (i) 8 units, (ii) 20 units.
4.3. The Chain Rule 33
4x2 + 100
C(x) = and R(x) = 7x + 200.
3x + 2
[Solution on page 202]
(a) Find
2. In each case, write h(x) as the composition of two functions. (There may be more
than one way to do this.) [Solution on page 204]
√
(a) h(x) = (x2 + 3)5 (c) h(x) = − 3 10 + 7x
√ 1 3
(b) h(x) = 7x + 4 (d) h(x) = (x2 + x) 4 − 3(x2 + x) 4 + 11.
dy
3. In each case, find dx
. [Solution on page 205]
4. Suppose that the revenue (in dollars) from the sale of x items of a certain product is
given by p
R(x) = 16 3 (x2 + x)2 .
[Solution on page 208]
(a) Find the marginal revenue when (i) x = 100 items, (ii) x = 250 items.
34 Chapter 4. Differentiation Rules
The quantity (q) and price per item (p) are related by the following equation:
p
q = 16000 − 2.5p
√
7. Find an equation of the tangent line to the curve y = x2 x3 − 2 at P (3, 45). [Solution
on page 213]
x
8. Find the points on the curve y = (x2 +28)4
where the tangent line is horizontal. [Solu-
tion on page 213]
2 −6x
4. Find the points on the curve y = ex where the tangent line is horizontal. [Solution
on page 221]
2 +7
(a) y = x2x (c) y = |1 + e2x |x
(b) y = (x2 + 1)3x
6. Find the points on the curve y = xx where the tangent line is horizontal. [Solution
on page 232]
C(x) = 3 log2 x + 10
3. Find an equation of the tangent line to the curve y = cos(3x) at the point π9 , 12 .
4. Find an equation of the tangent line to the curve y = sin(sin x) at (π, 0). [Solution
on page 244]
4.7. Exponential Growth and Decay 37
2. One can prove that the converse to the statement “if y grows or decays exponentially,
then the rate of change of y is proportional to y” is true. Specifically, one can prove
that if the rate of change of a quantity y = y(t) is proportional to y, say dy dt
= ky,
kt
then y has the form y = y0 e , where y0 is the quantity present at time t = 0. Use
this to find an expression for y in each case. And state whether y grows or decays
exponentially. [Solution on page 246]
3. In each case, find an expression for y = y(t), assuming exponential growth or decay.
[Solution on page 246]
38 Chapter 4. Differentiation Rules
(a) y(0) = 75, y(2) = 105 (c) y(2) = 265.46, y(7) = 308.42
(b) y(0) = 10000, y(5) = 7046.88 (d) y(3) = 1182.80, y(7) = 558.72
4. A bacteria culture initially contains 2000 bacteria and grows exponentially. After 4
hours the culture contains 3502 bacteria. Let y be the number of bacteria after t
hours. [Solution on page 248]
5. A culture initially contains 33000 bacteria and grows at a rate proportional to its
size. After 7 hours the culture contains 54000 bacteria. [Solution on page 249]
m(t) = m0 e−0.02476t
where m0 is the initial quantity and t is measured in days. What is the half-life of
this substance? [Solution on page 251]
(a) A sample has a mass of 35 mg initially. Find the mass remaining after 200 days.
(b) How many days does it take a sample weighting 35 mg to decay to a mass of 6
mg?
9. The world population in 2000 was approximately 6.143 billion. Assume that the pop-
ulation grows at the rate of approximately 1.31% per year. Suppose t = 0 corresponds
to 2000. [Solution on page 252]
10. Suppose you invest P dollars. After t years your amount on deposit is A dollars. In
each case, find the interest rate if the money is compounded continuously. [Solution
on page 254]
11. Suppose you invest $25500 in an account paying interest compounded continuously.
Also suppose that after 1 year your amount on deposit is $27076.83. Let A = A(t)
be the amount on deposit after t years. [Solution on page 254]
12. Alex deposits $7500 in an account paying interest compounded continuously. After 2
years the amount on deposit is $8247.44. Find the amount on deposit after 8 years.
[Solution on page 255]
13. Find the present value of A = $14000 in each case. [Solution on page 255]
14. Find the present value of A = $78500 if interest is 2.78% compounded continuously
for 4 years. [Solution on page 256]
15. Suppose you have to make a car payment of $40000 in 5 years. What is the present
value of the payment if it includes annual interest of 3.9% compounded continuously.
[Solution on page 257]
40 Chapter 4. Differentiation Rules
Chapter5
1
(a) f (x) = −2x2 + 8x − 3 (f ) f (x) = x
(b) f (x) = x3 + x2 + 5 2x+1
(g) f (x) = x−3
4 3 2
(c) f (x) = 3x + 8x − 18x
√ (h) f (x) = x ln(3x)
(d) f (x) = 3 x
√
(e) f (x) = 1 − x2 (i) f (x) = x2 · 3−x
2. Find the intervals where each function is increasing and where it is decreasing. [So-
lution on page 262]
2x+1
(a) f (x) = −2x2 + 8x − 3 (e) f (x) = x−3
x
(b) f (x) = x3 + x2 + 5 (f ) f (x) = x2 +1
3
(c) f (x) = 3x4 + 8x3 − 18x2 (g) f (x) = xe−x
2
2
(d) f (x) = (x − 1) 3 (h) f (x) = ln 7x +6
x2 +3
41
42 Chapter 5. Applications of the Derivative I
y
4
−2 −1 1 2 3 4 x
−2
−4
Figure 5.1
5.3 Concavity
1. Let f be the function whose graph is shown in Figure 5.2. Find the intervals where
the function is concave upward or concave downward. Find any inflection points.
[Solution on page 278]
y
4
−2 −1 1 2 3 4 x
−2
−4
Figure 5.2
3. Find the third and fourth derivative of each function. [Solution on page 280]
4. In each case, find the intervals where f (x) is concave upward or concave downward.
Find any inflection points. [Solution on page 281]
x
(a) f (x) = x2 − 6x + 3 (e) f (x) = x2 +1
−4x
(b) f (x) = x3 − 3x2 + 5 (f ) f (x) = xe
4 3
(c) f (x) = x − 4x (g) f (x) = ln(x2 + 4)
−3
(d) f (x) = x+1
(h) f (x) = (x − 1)4
44 Chapter 5. Applications of the Derivative I
5. In each case find the critical numbers of f (x). Then use the second derivative test to
decide whether the critical numbers lead to local extrema. If f 00 (c) = 0 or f 00 (c) does
not exist for a critical number c; then the second derivative test gives no information.
In this case, use an alternative method instead. [Solution on page 287]
(a) f (x) = 1 − 8x − x2
(b) f (x) = −x3 + 6x2 + 7
(c) f (x) = (x + 1)4
6. In each case find the Point of Diminishing Returns for the revenue function R(x).
[Solution on page 288]
−2 −1 1 2 3 4 x
−2
Figure 6.1
2. In each case, find the absolute extrema, as well as all values of x where they occur.
[Solution on page 304]
3. Suppose the cost (in dollars) of producing x units of a certain product is given by
45
46 Chapter 6. Applications of the Derivative II
(a) Find a formula for the cost of this box in terms of x and h.
(b) Express the volume of the box in terms of x and h.
(c) If the volume must be 768 cm3 , find h in terms of x.
(d) If the volume of the box must be 768 cm3 , what value of x should be used for
the cheapest box?
2. Use implicit differentiation to find an equation of the tangent line to the curve at the
given point. [Solution on page 323]
(a) x2 − xy − y 2 = 1 at (2, 1)
(b) (x2 + y 2 )2 = 2(x3 + y 2 ) at (1, 1)
d2 y
dy
3. If x2 −y 2 = 4, find the second derivative y 00 = dx2
= d
dx dx
by implicit differentiation.
[Solution on page 324]
4. [Solution on page 325]
Suppose the cost function C is given as follows.
√
C 2 − 40 = x2 + 80 x
dC
Find the marginal cost dx
at x = 4, and interpret your answer.
5. [Solution on page 326]
Suppose the cost and revenue functions are given by
R = 80x − 0.25x2 and C = 7x + 20
If the rate of change of production is 5 units per hour, find the rate of change of each
quantity when x = 100 units.
2. In each case, calculate and interpret the elasticity of demand. [Solution on page 332]
2
(a) q = p
(b) q = e5−0.3 ln p
(c) q = 15000p−1.68
3. In each case, calculate and interpret the elasticity of demand for the given demand
q. [Solution on page 334]
6.5. Differentials and Linear Approximations 49
2. Find the differential of each function y and evaluate it for the given values of x and
dx = ∆x. [Solution on page 340]
√
(a) y = x + 3, x = 1, dx = 0.1 (c) y = 15x+6
−x+2
, x = −2, dx = 0.03
√ x
(b) y = 4x2 + 13, x = 3, dx = 0.04 (d) y = e 8 , x = 0, dx = −0.02
3. In each case, use the differential to approximate the change in y as x changes from
x1 to x2 . [Solution on page 341]
ln(x−2)
(a) y = −x3 − x2 + 150, x1 = 1, x2 = 2 (c) y = x+1
, x1 = 3, x2 = 2.9
−2x
(b) y = xe , x1 = 0, x2 = 0.2
Use the differential to approximate the change in cost in each case. [Solution on
page 348]
(a) Use the differential to approximate the change in profit if x increases from 500
to 501.
(b) At the level of production of 500 units, find the marginal profit. And compare
your answer to the one found in part (a).
where q is the quantity demanded (in hundreds of units) and p is the price in dollars.
Use differentials to estimate the change in price when q changes from 300 to 370 units.
2. In each case, approximate a solution, to the nearest hundredth, for the equation in
the given intervals. [Solution on page 354]
6.6. Newton’s Method 51
3. In each case, use Newton’s method to approximate the given number correct to four
decimal places. [Solution on page 356]
√ √
(a) 5 (c) 3 50
√ √
(b) 23 (d) 7 139
4. Explain why Newton’s method doesn’t work for the equation x2 + 3x + 4 = 0 in the
interval [0, 1]. [Solution on page 358]
(a) Find the exact solution. Then find an approximate solution (round your answer
to two decimal places).
(b) If the initial guess is c1 = 2.1, use Newton’s method to approximate the solution
to the nearest hundredth. Compare your answer to the one found in part (a).
(c) Explain why Newton’s method does not work if the initial guess is c1 = 3.
(a) Use Newton’s method to approximate the critical number for f (x) to the nearest
hundredth.
(b) Decide whether the critical number found in part (a) leads to a relative minimum
or a relative maximum.
(a) Sketch the graphs of S and C on the same axes and show that the equation
S(t) = C(t) has only one solution t ≥ 0.
(b) Find the number of years, to the nearest hundredth, that the firm will realize
savings.
10. Suppose the total profit (in thousands of dollars) from selling x kilograms of a certain
product is given by
1
P (x) = − x5 + 2x3 + 3x2 + 5x − 4
7
It turns out that the function P has only one critical number c > 0. [Solution on
page 364]
(a) Find the number of kilograms, to the nearest hundredth, that should be sold in
order to maximize the total profit.
(b) What is the maximum profit?
PartII
Solution to Exercises
53
Chapter1
Linear Functions
(a) Solution.
Vertical Lines
Given two points P1 (x1 , y1 ) and P2 (x2 , y2 ),
Slope of a Line
We now want to find the slope of the line through the points (1, 2) and (4, 11).
Let P1 = (1, 2) and P2 = (4, 11). We have x1 = 1 and y1 = 2. We also have
x2 = 4 and y2 = 11. Using the formula (1.1.1), we get
y2 − y1 11 − 2 9
m= = = = 3.
x2 − x1 4−1 3
55
56 Chapter 1. Linear Functions
6 − (−3) 6+3 9
m= = = .
0 − (−2) 0+2 2
−2 − (−14) −2 + 14 12 3
m= = = = .
−7 − (−15) −7 + 15 8 2
(g) Solution.
Fractions
We recall the sum, difference, multiplication, and quotient of two fractions.
Let ab and dc be fractions.
Sum: a
b
+ c
d
= ad+bc
bd
.
Subtraction: a
b
− c
d
= ad−bc
bd
.
Multiplication: a
b
× c
d
= ac
bd
.
a
Quotient: b
c = a
b
× d
c
= ad
bc
.
d
For the sum and difference one could also use the method of common
denominators that we won’t recall.
2. [Exercise on page 3]
Solution.
The slope of the line through (−1, 1) and (2, 4x) is m = 4x−1
2−(−1)
= 4x−1
2+1
= 4x−1
3
.
Since the slope is 2, we must have 4x−1
3
= 2. Multiplying both sides of the latter
equation by 3, we get 4x − 1 = 6. Adding 1 to both sides, we get 4x = 7.
Dividing both sides by 4, we get x = 74 .
3. [Exercise on page 3]
(a) Solution.
Point-Slope Form
Slope-Intercept Form
The y-intercept of a line is the point of intersection between the
line and the y-axis. This is obtained by substituting x by 0 into
the equation of the line. For example, the y-intercept of the line
y = 2x + 3 is 2(0) + 3 = 0 + 3 = 3.
Slope-Intercept Form: y = mx + b.
This is the most common form for writing the equation of a line. We
will always use that form.
We now want to find an equation of the line through (2, 3) with slope m = 4.
Let x1 = 2 and y1 = 3. Using the point-slope form, we get the equation y − 3 =
4(x − 2). Distributing the right hand side, we get y − 3 = 4x − 8. Adding 3 to
both sides, we get y = 4x − 5. So the equation of the line through (2, 3) with
slope m = 4 is y = 4x − 5.
Since the line passes through (2, 3), we have 3 = 4(2) + b, that is, 3
= 8 +b. Subtracting 8 from both sides, we get −5 = b. So b = −5.
Therefore, the required equation is y = 4x − 5
(f ) Solution.
The slope is m = 10−1
5
− 21
= 94 = 92 .
2 2
5. [Exercise on page 3]
(a) Solution.
−2 −1 1 2 3 x
−1
−2
(b) Solution.
−2 −1 1 2 x
−1
1 (1, 1)
−2 −1 1 2 3 x
−1 (0, −1)
−2
4 (0, 4)
3 (1, 3)
2
1
1 2 3 4 5 x
(a) Solution.
The sentence “The sales of a small company were $17,000 in its fourth year”
means that the sales line passes through the point (4, 17000). And the sentence
“$25,000 in its sixth year” means that the sales line passes through (6, 25000).
Thus, we are looking for the line that passes through the points (4, 17000) and
(6, 25000).
The slope is
25000 − 17000 8000
m= = = 4000.
6−4 2
The required equation is
y − 17000 =4000(x − 4)
y − 17000 =4000x − 16000
y =4000x + 1000
(b) Solution. The sales surpass $48,500 if 4000x + 1000 > 48500. Subtracting
1000 from both sides, we get 4000x > 47500. Dividing both sides by 4000, we
get x > 47500
4000
= 11.875 ≈ 12. So 12 years must pass before the sales surpass
$48,500.
2. [Exercise on page 4]
(a) Solution. Plotting the points (1, 10), (2, 14), (3, 16), (4, 18), and (5, 20), we get
Figure 1.5. According to the plot, the data are not exactly linear (as they don’t
lie along a straight line). But the plot is approximately linear, so it could be
approximated by a linear equation.
(b) Solution.
The slope is m = 20−10
5−1
= 10
4
= 2.5.
The equation is y − 10 = 2.5(x − 1), that is, y − 10 = 2.5x − 2.5. So
y = 2.5x + 7.5 is the equation of the line that models the data.
The slope indicates that the grade on exam increases by about 2.5 when the
number of hours studied increases by 1.
62 Chapter 1. Linear Functions
y
20
15
10
1 2 3 4 5 x
Figure 1.5
(c) Solution. To estimate the grade of a student who studied 6 hours, all we have
to do is to substitute x by 6 into the equation y = 2.5x + 7.5. We obtain
y = 2.5(6) + 7.5 = 15 + 7.5 = 22.5.
3. [Exercise on page 4]
(a) Solution.
Linear Functions
A linear function is a function of the form f (x) = mx + b, where m and b
are both real numbers.
4. [Exercise on page 4]
5. [Exercise on page 4]
1.2. Linear Functions and Applications 63
(a) Solution.
(i) For 0 shoe, q = 0, and therefore the corresponding price is p = D(0) =
10 − 0.5(0) = 10 − 0 = $10.
(ii) For 5000 shoes, q = 5 (remember the quantity is in thousands), and therefore
the corresponding price is p = D(5) = 10 − 0.5(5) = 10 − 2.5 = $7.5.
(b) Solution.
(i) If the price is p = 4, then 10 − 0.5q = 4. Subtracting 10 from both sides, we
−6
get −0.5q = −6. Dividing both sides by −0.5, we get q = −0.5 = 12. Thus,
at a price of $4, the quantity demanded is 12, 000 shoes.
(ii) If the price is p = 9, then 10 − 0.5q = 9. Solving this for q, we get q =
9−10
−0.5
−1
= −0.5 = 2. Thus, at a price of $9, the quantity demanded is 2, 000
shoes.
(c) Solution. The graphs of D(q) and S(q) are shown in Figure 1.6.
p
D(q) = 10 − 0.5q
S(q) = 0.25q + 1
10
4 Equilibrium point
5 10 15 20 25 q
Figure 1.6
(d) Solution.
To find the equilibrium point, one needs to solve the equation S(q) = D(q).
64 Chapter 1. Linear Functions
Finding the equilibrium quantity. Set S(q) = D(q), that is, 0.25q + 1 =
10−0.5q. Gathering the q’s on the left hand side, we get 0.75q = 9. Dividing
9
both sides by 0.75, we get q = 0.75 = 12. So the equilibrium quantity is
12, 000 shoes.
The equilibrium price is p = S(12) = 0.25(12) + 1 = 3 + 1 = $4.
6. [Exercise on page 5]
Solution.
4 3
q = 105 − q
5 5
7 3
q = 105 Add q to both sides
5 5
7q = 525 Multiply both sides by 5
525
q= = 75
7
So the equilibrium quantity is q = 75.
The equilibrium price is p = S(75) = 45 (75) = 60.
7. [Exercise on page 5]
Solution.
The company spends $0.24 to produce a pencil. So, the cost of producing x pencils
is 0.24x. Since the fixed cost is $37000, the required linear cost function is C(x) =
0.24x + 37000.
8. [Solution on page 5]
1.2. Linear Functions and Applications 65
(a) Solution. Since the company spends $7 to produce one unit, the cost of pro-
ducing x units is 7x. Taking into account the fixed cost, $80000, we have
C(x) = 7x + 80000
(b) Solution.
Since each unit is sold at $10, the revenue for selling x units is R(x) = 10x.
(c) Solution.
9. [Exercise on page 5]
(a) Solution.
Break-Even Quantity
Let C(x), R(x), and P (x) be the cost, revenue, and profit functions respec-
tively. The break-even quantity is the value of x where revenue equals cost
(R(x) = C(x)). This can also be found by solving the equation P (x) = 0.
Set R(x) = C(x). Then 20x = 3x + 85. Solving this for x, we get 17x = 85, so
that x = 85
17
= 5. Thus, the break-even quantity is x = 5.
(b) Solution.
First, we need to find the profit function:
(c) Solution. We need to solve the equation P (x) = 11815 for x. We have 17x −
85 = 11815. Adding 85 to both sides, we get 17x = 11900. Dividing both sides
by 17, we get x = 11900
17
= 700. So 700 units will produce a profit of $11815.
Chapter2
2. [Exercise on page 7]
(a) Solution. One can see that x is a common factor. Taking that out, we get
x2 − 5x = x(x − 5).
(b) Solution. One can factor this by grouping:
67
68 Chapter 2. Quadratic, Exponential, and Logarithmic Functions
(c) Solution. First we recall the difference of squares which states that for any
numbers a and b, one has a2 − b2 = (a − b)(a + b). Applying this, we get
x2 − 25 = x2 − 52 = (x − 5)(x + 5).
(d) Solution. We have
(f ) Solution. First, one has a = 2, b = −7, and c = 3. We are looking for two
integers whose product is ac = 6 and whose sum is b = −7. After trying a
number of combinations, we get that the relevant numbers are −1 and −6. So
2x2 − 7x + 3 = 2x2 − x − 6x +3
| {z } | {z }
= x(2x − 1) − 3(2x − 1) = (2x − 1)(x − 3).
3. [Exercise on page 7]
(a) Solution.
ab = 0 implies that a = 0 or b = 0
Alternate Solution
An alternate way to solve the equation 4x2 − 9 = 0 is to use the following
fact: √
x2 = a implies that x = ± a,
where a is a nonnegative number.
4x2 − 9 = 0
4x2 = 9 Add 9 to both sides
9
x2 = Divide both sides by 4
4
r
9 3
x=± =±
4 2
3
So the solutions are 2
and − 32 .
(b) Solution.
x2 − 3x = 0
x(x − 3) = 0 Factor
x = 0 or x − 3 = 0
x = 0 or x = 3
3x + 2 = 0 or −x + 1 = 0
3x = −2 or −x = −1
2
x=− or x=1
3
70 Chapter 2. Quadratic, Exponential, and Logarithmic Functions
Alternate Solution
Another way to solve the equation −3x2 +x+2 = 0 is to use the quadratic
formula.
Quadratic Formula
4. [Exercise on page 7]
(a) Solution.
4
x-intercepts
2
y-intercept
2 4 6 x
−1
vertex
(b) Solution.
The coefficients are a = −2, b = 7, and c = 4. Since a is negative, the
parabola opens downward.
The y-intercept is (0, 4).
2.1. Quadratic Functions 73
Factoring f (x), we get f (x) = (−2x − 1)(x − 4). And solving the equation
f (x) = 0, we get −2x − 1 = 0 or x − 4 = 0, that is, x = − 12 or x = 4. These
are the x-intercepts.
For the vertex, we have − 2ab 7
= − 2(−2) = 74 . And
2
7 7 7 49 49
f = −2 +7 + 4 = −2 + +4
4 4 4 16 4
98 196 64 81
=− + + = .
16 16 16 8
So the vertex is 47 , 81
8
.
The graph of f (x) = −2x2 + 7x + 4 is shown in Figure 2.2.
y
10
2 4 x
(c) Solution.
The coefficients are a = 1, b = 4, and c = 4. Since a > 0, the parabola
opens upward.
The y-intercept is (0, 4).
Setting x2 + 4x + 4 = 0, we get (x + 2)2 = 0, which implies that x = −2.
So there is only one x-intercept, namely −2.
We have
b 4
− =− = −2 and f (−2) = (−2)2 + 4(−2) + 4 = 4 − 8 + 4 = 0.
2a 2(1)
−4 −3 −2 −1 1 x
So the vertex is 13 , − 23 .
y
1
−2 −1 1 2 x
−1
−2
−3
5. [Exercise on page 7]
(a) Solution. We need to solve the equation R(x) = C(x) for x. This is amount
to solving −x2 + 26x = 7x + 48 or −x2 + 19x − 48 = 0. Using the quadratic
2.1. Quadratic Functions 75
formula, we get
p √
−19 ±(19)2 − 4(−1)(−48) −19 ± 361 − 192
x= =
2(−1) −2
√
−19 ± 169 −19 ± 13
= = .
−2 −2
So
−19 + 13 −6 −19 − 13 −32
x= = = 3 or x = = = 16.
−2 −2 −2 −2
We then have two break-even quantities: 3 and 16. The minimum one is 3.
(b) Solution.
From the problem, the revenue function is R(x) = −x2 + 26x.
The graph of the revenue function is a parabola, which is opens downward
because a = −1 is negative. So the vertex of the parabola will give us the
maximum revenue.
Finding the vertex. We have − 2a b 26
= − 2(−1) = 13, and f (13) = −(13)2 +
26(13) = 169. So the vertex is (13, 169), and therefore
the maximum revenue is $169000. (Remember that the profit is in thou-
sands of dollars.)
(c) Solution.
The profit function is
b 19 19
− =− = = 9.5.
2a 2(−1) 2
And
6. [Exercise on page 8]
(a) Solution. To find the break-even quantity, we need to solve the equation R(x) =
C(x).
x2 35
− + 5x = x +
3 3
−x2 + 15x = 3x + 35 Multiply both sides by 3
76 Chapter 2. Quadratic, Exponential, and Logarithmic Functions
−12 + 2 −12 − 2
x= or x=
−2 −2
x=5 or x=7
1 35 35 35
P (6) = − (6)2 + 4(6) − = −12 + 24 − = 12 −
3 3 3 3
36 − 35 b ac − b
= a− =
3 c c
1
=
3
(a) Solution.
2.2. Exponential Functions and Compound Interest 77
Exponential Functions
Let a be a real number such that a > 0 and a 6= 1. The function defined
by f (x) = ax is called the exponential function of base a. Some facts about
exponential functions include the following.
The statement “the exponential function f (x) = ax is defined for every real
number x” is True.
4 y = 2x
1
y = (0.5)x
−4 −3 −2 −1 1 2 3 4 x
Figure 2.5
2. [Exercise on page 8]
32x 3−2
(a)
3x−1
Solution.
√ 1
(c) Solution. Remembering that 3
a = a 3 , we have
3. [Exercise on page 8]
ax = ay if and only if x = y.
Note. Before using this, we have to make sure that we have the same
base on both sides of the equation ax = ay .
(52 )x = 58
52x = 58
2x = 8
x=4 Divide both sides by 2.
(e) Solution. The equation 36x = 216 is equivalent to (62 )x = 63 or 62x = 63 . This
latter equation implies that 2x = 3. Dividing both sides by 2, we get x = 23 .
(f ) Solution.
Since 27 = 33 and 81 = 34 , the equation 27x−1 = 813x+2 becomes (33 )x−1 =
(34 )3x+2 , that is, 33x−3 = 312x+8 .
This latter equation is equivalent to 3x − 3 = 12x + 8. Subtracting 12x from
both sides, we get −9x − 3 = 8. Adding 3 to both sides, we get −9x = 11.
Dividing both sides by −9, we get x = − 11 9
.
(g) Solution. The equation 2x = 1 is equivalent to 2x = 20 . So x = 0.
(h) Solution.
2 2
Adding 336 to both sides of the equation 34x − 336 = 0, we get 34x = 336 .
This implies that 4x2 = 36. Dividing both sides by 4, we get x2 = 9. So
x = ±3.
2 2
(i) Solution. The equation 5x 5x = 56 is equivalent to 5x +x = 56 . So x2 + x = 6
and therefore x2 +x−6 = 0. Factoring the left hand side, we get (x−2)(x+3) = 0.
So x = 2 or x = −3.
x2 2
(j) Solution. The equation 73x−2 7
= 1 is equivalent to 7x −(3x−2) = 1, that is,
2
7x −3x+2 = 70 . This implies that x2 − 3x + 2 = 0. Factoring the left hand side,
we get (x − 1)(x − 2) = 0, which implies that x = 1 or x = 2.
2 3
(k) Solution. The equation (3x )x = 3x is equivalent to 3x = 3x , that is, x3 = x.
Moving x to the left hand side, we have x3 − x = 0. Factoring this, we get
x(x2 − 1) = 0. Factoring further, we get x(x − 1)(x + 1). This implies that
x = 0 or x = 1 or x = −1.
4. [Exercise on page 8]
Solution.
Compound Amount
The formula for the compound amount is
r tm
A=P 1+ ,
m
80 Chapter 2. Quadratic, Exponential, and Logarithmic Functions
where
From the problem, P = 2000, r = 12% = 0.12, t = 5. Since the interest is com-
pounded annually, m = 1. Substituting these into the formula, we get
5(1)
0.12
A = 2000 1 + = 2000(1.12)5 = 2000(1.762341) ≈ 3524.68.
1
So A ≈ $3524.68.
5. [Exercise on page 8]
Solution. Here P = 7000, r = 4% = 0.04, t = 3. Since the interest is compounded
quarterly, we have m = 4. Using the formula, we get
3(4)
0.04
A = 7000 1 + = 7000(1 + 0.01)12 = 7000(1.01)12 ≈ 7887.77.
4
So A ≈ $7887.77.
6. [Exercise on page 8]
Solution.
(b) The interest earned is the final amount minus the initial amount: 16138.67 −
12000 = 4138.67.
2.2. Exponential Functions and Compound Interest 81
7. [Exercise on page 9]
Solution.
First, we need to find the compound amount. We have P = 5000, r = 7%, t = 9,
and m = 12 (because the interest is compounded monthly). Using the formula,
we get
9(12) 108
0.07 0.07
A = 5000 1 + = 5000 1 + ≈ 9370.88.
12 12
The interest earned is 9370.88 − 5000 = $4370.88.
8. [Exercise on page 9]
Solution.
Continuous Compounding
If the number of times interest is compounded in a year goes to infinity,
we say that the interest is compounded continuously.
A = P ert ,
where
(a) Solution.
Logarithms–Definition
For example, what is log2 8? To find log2 8, we need to find the exponent
y of 2 such that 2y = 8. Since 23 = 8, we have that log2 8 = 3. Similarly,
log2 32 = 5 because 25 = 32.
2. [Exercise on page 9]
(a) Solution.
2.3. Logarithmic Functions 83
Logarithmic Functions
Let a > 0, a 6= 1. The function defined by
is called the logarithmic function of base a. Some facts about the loga-
rithmic function include the following.
The function f (x) = loga (x) is defined only for positive number. So
loga (0) is undefined, loga (−4) is undefined, etc.
The x-intercept of f (x) = loga (x) is 1 because loga (1) = loga (a0 ) =
0.
The graph of f (x) = loga (x) is increasing when a > 0 and decreasing
when 0 < a < 1. (For example, see Figure 2.6.)
y = log2 x
2
1 2 3 4 x
−2
y = log0.5 x
−4
Figure 2.6
3. [Exercise on page 9]
Alternate Solution
Another way to compute this is to use the following property (i).
(b) Solution.
40
log2 (40) − log2 (5) = log2 Property (ii)
5
= log2 (8)
= log2 (23 )
=3
(c) Solution.
4. [Exercise on page 9]
(a) Solution. Using Property (iii), we get log2 x2020 = 2020 log2 x .
2.3. Logarithmic Functions 85
(b) Solution.
log x = log10 x.
ln x = loge x.
We have
y8
log = log y 8 − log x3 = 8 log y − 3 log x.
x3
(d) Solution.
Let a > 0, a 6= 1. Then for every positive real number x (x > 0),
one has
aloga x = x
loga (ay ) = y
and
ln(ey ) = y for every real number y. (2.3.1)
In particular, ln(e) = 1.
86 Chapter 2. Quadratic, Exponential, and Logarithmic Functions
3 3
ln(x8 e−x ) = ln x8 + ln e−x
3
= 8 ln x − x3 ln e−x = −x3 by (2.3.1)
(e) Solution.
√ 1 1
x2 − 3x = ln(x2 − 3x) 3 = ln (x2 − 3x).
3
ln
3
(f ) Solution.
(3x − 1)4 x2
ln = ln((3x − 1)4 x2 ) − ln(2x + 1)5
(2x + 1)5
= ln(3x − 1)4 + ln x2 − ln(2x + 1)5
= 4 ln (3x − 1) + 2 ln x − 5 ln (2x + 1).
5. [Exercise on page 9]
(b) Solution.
1 1
3 ln x − ln(5x) + 7 = 3 ln x − ln(5x) + ln(e7 ) ln(e7 ) = 7 by (2.3.1)
2 2
1
= ln(x ) − ln((5x) 2 ) + ln(e7 )
3
3
x
= ln √ + ln(e7 )
5x
3 7
x e
= ln √ .
5x
(c) Solution. Using the fact that 4 = log(104 ), the expression becomes
(a) Solution.
88 Chapter 2. Quadratic, Exponential, and Logarithmic Functions
Equality of logarithms
Let a > 0, a 6= 1, and let x, y be positive numbers.
If a = e, this becomes
So x = −2 is a valid solution.
Thus, the solutions are −2 and 3.
(c) Solution. To isolate x, we need to take the natural exponential of both sides.
This gives eln x = e2 . Since eln x = x, the latter equation becomes x = e2 , which
is the solution to the original equation.
Alternate Solution
Since ln x = loge x, the equation becomes loge x = 2. By definition of
logarithms, loge x = 2 if and only if e2 = x. So x = e2 .
q
−1
x = − e10 . The negative value is to be rejected because when we substitute it
into the original equation, we have the logarithm of a negative
q number, which
e−1
is undefined. So the solution to the original equation is x = 10
.
(a) Solution. Taking the natural logarithm on both sides of the equation 2x = 5,
we get ln(2x ) = ln 5. This is equivalent to x ln 2 = ln 5. Dividing both sides by
ln 5
ln 2, we get x = ln 2
.
Alternate Solution
The equation 2x = 5 suggests that the base is 2. So by definition of
logarithms, 2x = 5 if and only if x = log2 5. This solution is the same as
the one we found using the natural logarithm. Indeed, using the Change-
of-Base Theorem for Logarithms, we get log2 5 = ln 5
ln 2
.
(b) Solution. Taking natural logarithms on both sides of the equation 32x−1 = 7,
the equation becomes ln(32x−1 ) = ln 7. Using the property ln(xr ) = r ln x, the
latter equation becomes (2x − 1) ln 3 = ln 7. Dividing both sides by ln 3, we
get 2x − 1 = ln 7
ln 3
. Adding 1 to both sides, we get 2x = ln 7
ln 3
+ 1 = ln 7+ln
ln 3
3
. So
ln 7+ln 3
x = 2 ln 3 .
Alternate Solution
Using the definition of logarithms, we have 32x−1 = 7 if and only if 2x−1 =
log3 7. Solving this for x, we get x = 1+log
2
37
. This is the same as ln27+ln
ln 3
3
.
Indeed,
1 + log3 7 1 + ln 7
ln 3 Change-of-Base Theorem for Log-
=
2 2 arithms (loga x = ln x
ln a
)
ln 3+ln 7
ln 3
=
2
ln 3 + ln 7
=
2 ln 3
(c) Solution. Taking the natural logarithm on both sides of the equation 36x =
4x+1 , we get 6x ln 3 = (x + 1) ln 4. Distributing the right hand side, we get
6x ln 3 = x ln 4+ln 4. Subtracting x ln 4 from both sides, we get 6x ln 3−x ln 4 =
ln 4. Factoring the left hand side, we get x(6 ln 3 − ln 4) = ln 4. This implies
that x = 6 lnln 4
3−ln 4
.
(d) Solution. Dividing both sides of the equation 2e−3x = 9 by 2, we get e−3x = 92 .
9
Taking the natural logarithm on both sides, we get −3x ln
e = ln 2 . (Remem-
9
ber ln e = 1.) The latter equation becomes −3x = ln 2 . Dividing both sides
ln( 9 )
by −3, we get x = −32 .
90 Chapter 2. Quadratic, Exponential, and Logarithmic Functions
(e) Solution. Subtracting 3 from both sides of the equation 2e−0.1x + 3 = 15, we
get 2e−0.1x = 12. Dividing both sides by 2, we get e−0.1x = 6. Taking the natural
logarithm on both sides, we get −0.1x = ln 6. Dividing both sides by −0.1, we
get x = −10 ln 6.
(a) Solution.
ax = ex ln a .
(a) Solution.
First, we have P = 1000, r = 4% = 0.04, and m = 1 (because the interest
is compounded annually). We also have A = 2000 because the amount has
double.
Substituting these into the formula for the compound amount, we get 2000 =
1000(1 + 0.04
1
)t(1) . This is equivalent to 2000 = 1000(1.04)t .
We solve the latter equation for t. Dividing both sides by 1000, we get
(1.04)t = 2. Taking the natural logarithm on both sides, we get t ln(1.04) =
ln 2. This implies that
ln 2 0.69314718056
t= = ≈ 17.67.
ln(1.04) 0.03922071315
(b) Solution.
If the interest is compounded quarterly, m = 4, and the formula for the
compound amount becomes 2000 = 1000 1 + 0.04 4
)t(4) . This is equivalent
to 2000 = 1000(1 + 0.01) , that is, 2000 = 1000(1.01)4t . So (1.01)4t = 2.
4t
We solve this latter equation for t. Taking the natural logarithm on both
sides, we get 4t ln(1.01) = ln 2. This implies that
ln 2
t= ≈ 17.41.
4 ln(1.01)
(c) Solution.
2.3. Logarithmic Functions 91
Because the interest is compounded continuously, the formula for the com-
pound amount is A = P ert . Substituting A with 2000, P with 1000, and r
with 0.04, we get 2000 = 1000e0.04t .
We solve this latter equation for t. Dividing both sides by 1000, we get
e0.04t = 2. Taking the natural logarithm on both sides, we get 0.04t = ln 2.
Dividing both sides by 0.04, we get
ln 2
t= ≈ 17.33.
0.04
12. [Exercise on page 10]
A(0) 2(400 ln 4)
ln(t + 4) = 2 = = 2 ln 4
400 400
So ln(t + 4) = 2 ln 4. Taking the natural exponential on both sides, we get
t + 4 = e2 ln 4 . Subtracting 4 from both sides, we have
2
t = e2 ln 4 − 4 = eln(4 ) − 4 = eln(16) − 4 = 16 − 4 = 12
Thus, it takes 12 years for the account to double its initial value.
92 Chapter 2. Quadratic, Exponential, and Logarithmic Functions
Chapter3
(a) Solution.
lim f (x) = L
x→a
The table tells us that f (x) gets closer and closer to 4 as x approaches 1. We
can then make the guess that lim (x2 + 3) = 4.
x→1
x2 −4
(b) Solution. Let f (x) = x+2
. Consider the following table.
93
94 Chapter 3. Limits and Derivatives
x2 −4
From the table, we can make the guess that lim = −4.
x→−2 x+2
ex −1
(c) Solution. Let f (x) = x
. Consider the following table.
ex −1
From the table, we can make the guess that lim x
= 1.
x→0
(a) f (2) does not exist because the function is not defined at 2 (as there is a hole
on the graph).
(b) We have lim f (x) = 3 because when x approaches 2 from either side, f (x) gets
x→2
closer and closer to 3.
(c) f (4) = 6.
(d) lim f (x) = 5 because when x approaches 4 from either side, f (x) is gets closer
x→4
and closer to 5.
(e) f (3) = 4
(f ) lim f (x) = 4.
x→3
One-Sided Limits
Left-Sided: When x gets closer and closer to a and x is less than a
(x < a), we say that x approaches a from the left and we write x → a− .
The limit of f (x) from the left is written lim− f (x).
x→a
A Two-Sided limit lim f (x) exists if and only if both one-sided limits
x→a
exist and are the same. More precisely,
lim f (x) = L if and only if lim− f (x) = L and lim+ f (x) = L .
x→a x→a x→a
So, if the limit from the left is not the same as the limit from the right,
then lim f (x) does not exist.
x→a
3.1. Finding Limits Using Tables and Graphical Limits 95
(c) lim f (x) does not exist because the limit from the left is not equal to the
x→2
limit from the right.
(d) f (2) = 1
(e) lim f (x) = 3 because the limit from the left ( lim− f (x) = 3) is equal to the
x→6 x→6
limit from the right ( lim+ f (x) = 3).
x→6
(h) lim+ f (x) does not exist because the function is not defined for values of x
x→7
bigger than 7.
(c) lim f (x) does not exist because the limit from the left is not the same as
x→−1
the limit from the right.
(d) f (−1) = 2.
(e) lim f (x) does not exist because the limit from the left ( lim− f (x) = 4) is not
x→3 x→3
the same as the limit from the right ( lim+ f (x) = 6).
x→3
(f ) f (3) = 6.
(g) lim− f (x) = 4.
x→6
(h) f (6) does not exist because the function f (x) is not defined at x = 6.
Vertical Asymptotes
Let f (x) be a function, and let a be a number. The line x = a is called vertical
asymptote of the curve y = f (x) if at least one of the following holds.
96 Chapter 3. Limits and Derivatives
provided that An exists. (This limit does not exist, for example,
when A < 0 and n = 12 .)
3.2. Finding Limits Using Rules and One-Sided Limits 97
Polynomials
A term is an expression of the form axn , where a is a real number,
called coefficient, x is the variable, and n is the exponent. Examples
1
of terms include 2x3 , −3x6 , 13 x 2 .
1
= −81 + 5(9) 2 Rule 5
= −81 + 5(3)
= −81 + 15 = −66
(d) Solution.
9(1)2 − 4
= Rule 5
1+1
9−4 5
= =
2 2
(e) Solution.
(2)4 − 3(2)2 + 1
= √ Rule 5 and Rule 6
2 + 98
16 − 12 + 1
= √
100
5 1
= =
10 2
(f ) Solution.
√ √
(x2 − 3x − 8) 3 2x − 3 lim (x2 − 3x − 8) 3 2x − 3
lim = x→5 Rule 4
x→5 x−1 lim (x − 1)
x→5
√
lim (x2 − 3x − 8) lim 3 2x − 3
= x→5 x→5
Rule 3
lim (x − 1)
x→5
p
(52 − 3(5) − 8) 3 2(5) − 3
= Rule 5 and Rule 6
5−1
√
(25 − 15 − 8) 3 10 − 3
=
4
2√ 3 1√ 3
= 7= 7
4 2
√
(g) Solution. The limit lim x2 − 16 does not exist because the limit
x→3
(h) Solution.
3 +x lim −2x3 +x
lim e−2x = ex→0 Rule 8
x→0
3
= e−2(0) +0 Rule 5
= e0 = 1
(i) Solution.
!
x2 + x x2 + x
lim ln = ln lim1 Rule 9
x→ 21 2x + 3 x→ 2 2x + 3
1 1
+
= ln 4 2 Rule 4 and Rule 5
1+3
3
4
3
= ln = ln
4 16
x2 − 16
=x+4 for all x 6= 4.
x−4
x2 −16
Using Rule 7, we get lim x−4
= lim (x + 4). This latter limit is equal to
x→4 x→4
x2 −16
4 + 4 = 8 by Rule 5. Thus, lim x−4
= 8.
x→4
0
Indeterminate Form 0
P (x)
Let f (x) = Q(x) be a function where P (x) and Q(x) are both polynomials
(such function is called a rational function). Let a be a number such that
P (a)
P (a) = 0 and Q(a) = 0. Then the quotient Q(a) = 00 is undefined and is
P (x)
called an indeterminate form. To find lim , one can proceed as follows.
x→a Q(x)
ii. Simplify.
iii. Substitute x with a into the simplified function. The result is the
P (x)
limit lim Q(x) .
x→a
100 Chapter 3. Limits and Derivatives
2
(b) Solution. Substituting x with 1 into the function f (x) = x 5x−5
+2x−3
, we get 00 ,
which is undefined. Since the numerator and denominator are both polynomials,
we can factor and simplify.
2
(c) Solution. Substituting x with 3 into x x−2x−3
2 −9 , we get the indeterminate form 00 .
Since the numerator and denominator are both polynomials, we need to factor
them and simplify.
3+1
= Substitute x with 3
3+3
4 2
= =
6 3
x2 +3x−10
(d) Solution. Substituting x with −5 into x2 +6x+5
, we get the indeterminate form
0
0
.
x2 + 3x − 10 (x + 5)(x − 2)
lim = lim Factor
x→−5 x2 + 6x + 5 x→−5 (x + 5)(x + 1)
x−2
= lim Simplify
x→−5 x + 1
−5 − 2
= Substitute x with −5
−5 + 1
−7 7
= =
−4 4
2x2 −x−6
(e) Solution. Substituting x with 2 into x2 +2x−8
, we get 00 , which is an indetermi-
3.2. Finding Limits Using Rules and One-Sided Limits 101
nate form.
2x2 − x − 6 (x − 2)(2x + 3)
lim 2
= lim Factor
x→2 x + 2x − 8 x→2 (x − 2)(x + 4)
2x + 3
= lim Simplify
x→2 x + 4
2(2) + 3
= Substitute x with 2
2+4
7
=
6
3x2 +8x−3
(f ) Solution. Substituting x with −3 into x2 +3x
, we get the indeterminate form
0
0
.
3x2 + 8x − 3 (x + 3)(3x − 1)
lim 2
= lim Factor
x→−3 x + 3x x→−3 (x + 3)x
3x − 1
= lim Simplify
x→−3 x
3(−3) − 1
= Substitute x with −3
−3
−10 10
= =
−3 3
1 8x2 +2x−3
(g) Solution. Substituting x with 2
into 2x2 +3x−2
, we get the indeterminate form
0
0
.
x − 12 (8x + 6)
8x2 + 2x − 3
lim 2
= lim1 1
Factor
x→ 12 2x + 3x − 2 x→ 2 x − 2 (2x + 4)
8x + 6
= lim1 Simplify
x→ 2 2x + 4
8 12 + 6
1
= Substitute x with
2 21 + 4
2
4+6 10
= = =2
1+4 5
102 Chapter 3. Limits and Derivatives
(h) Solution.
2
x2 + 14x − 15
x + 14x − 15
lim log2 = log2 lim Rule 9
x→1 x2 − 1 x→1 x2 − 1
(x − 1)(x + 15)
= log2 lim Factor
x→1 (x − 1)(x + 1)
x + 15
= log2 lim Simplify
x→1 x + 1
1 + 15 Substitute
= log2
1+1 x with 1
16
= log2 = log2 (8) = log2 (23 ) = 3
2
(i) Solution.
x2 −x x2 −x
lim x
lim 3 x = 3x→0 Rule 8
x→0
(x−1)x
lim x
= 3x→0 Factor
lim (x−1)
= 3x→0 Simplify
= 30−1 Substitute x with 0
1
= 3−1 =
3
(j) Solution.
1
x
− 18 8−x
a c ad − bc
lim = lim 8x − =
x→8 x − 8 x→8 x − 8 b d bd
a
8−x b a
= lim =
x→8 8x(x − 8) c bc
−(x − 8)
= lim Factor the numerator
x→8 8x(x − 8)
−1
= lim Simplify
x→8 8x
1
=− Substitute x with 8
8(8)
1
=−
64
3. [Exercise on page 14]
(a − b)(a + b) = a2 − b2 . (3.2.1)
√ √
5(3 + 2 7) 5(3 + 2 7)
= =− .
9 − 28 19
√
2x 2x 2x
lim √ = lim+ √ √ Rationalize the denominator
x→0+ 2x x→0 ( 2x)( 2x)
√
2x 2x
= lim+
x→0 2x
√
= lim+ 2x Simplify 2x
x→0
p
= 2(0) Substitute x with 0
=0
(b) Solution.
√
x−4 (x − 4)( x + 2)
lim √ = lim √ √ Rationalize the denominator
x→4 x − 2 x→4 ( x − 2)( x + 2)
√
(x − 4)( x + 2) Distribute the denominator
= lim √
x→4 ( x)2 − (2)2 using (3.2.1)
√
(x − 4)( x + 2) √ √ √
= lim ( x)2 = ( x)( x) = x
x→4 x−4
√
= lim x + 2 Simplify
x→4
√
= 4+2 Substitute x with 4
3.2. Finding Limits Using Rules and One-Sided Limits 105
Alternate Solution
√ √
Using the fact that x − 4 = ( x − 2)( x + 2), we get
√ √
x−4 ( x − 2)( x + 2)
lim √ = lim √
x→4 x − 2 x→4 x−2
√
= lim x + 2 Simplify
x→4
√
= 4+2 Substitute x with 4
=2+2=4
(c) Solution.
√ √ √
x−5 ( x − 5)( x + 5)
lim = lim √ Rationalize the numerator
x→25 x − 25 x→25 (x − 25)( x + 5)
√
( x)2 − (5)2 Distribute the numerator
= lim √
x→25 (x − 25)( x + 5) using (3.2.1)
x − 25
= lim √
x→25 (x − 25)( x + 5)
1
= lim √ Simplify
x→25 x+5
1
=√ Substitute x with 25
25 + 5
1 1
= =
5+5 10
(d) Solution.
√ √ √
3− x (3 − x)(3 + x)
lim = lim 2 √ Rationalize the numerator
x→9 x2 − 81 x→9 (x − 81)(3 + x)
9−x Distribute the numerator
= lim √
x→9 (x2 − 81)(3 + x) using (3.2.1)
9−x
= lim √ Factor x2 − 81
x→9 (x − 9)(x + 9)(3 + x)
−(x − 9)
= lim √ Factor the numerator
x→9 (x − 9)(x + 9)(3 + x)
−1
= lim √ Simplify
x→9 (x + 9)(3 + x)
1
=− √ Substitute x with 9
(9 + 9)(3 + 9)
1 1
=− =−
18(6) 108
106 Chapter 3. Limits and Derivatives
(e) Solution.
√ √ √
4−2 x (4 − 2 x)(4 + 2 x) Rationalize the
lim = lim 2 √
x→4 x2 − x − 12 x→4 (x − x − 12)(4 + 2 x) numerator
√
(42 ) − (2 x)2 Distribute the numer-
= lim 2 √
x→4 (x − x − 12)(4 + 2 x) ator using (3.2.1)
16 − 4x
= lim √
x→4 (x2 − x − 12)(4 + 2 x)
−4(x − 4) Factor the numerator
= lim √
x→4 (x − 4)(x + 3)(4 + 2 x) and denominator
−4
= lim √ Simplify
x→4 (x + 3)(4 + 2 x)
−4
= √ Substitute x with 4
(4 + 3)(4 + 2 4)
−4 1
= =−
7(8) 14
(f ) Solution.
√ √ √
x2 + 16 − 4 ( x2 + 16 − 4)( x2 + 16 + 4) Rationalize the
lim = lim √
x→0 x2 x→0 x2 ( x2 + 16 + 4) numerator
(x2 + 16) − 16 Distribute the numer-
= lim √
x→0 x2 ( x2 + 16 + 4) ator using (3.2.1)
x2
= lim √
x→0 x2 ( x2 + 16 + 4)
1
= lim √ Simplify
x→0 x2 + 16 + 4
1
=√ Substitute x with 0
2
0 + 16 + 4
1 1 1
=√ = =
16 + 4 4+4 8
3.2. Finding Limits Using Rules and One-Sided Limits 107
(g) Solution.
√ √ p
1 − 1 − x2 (1 − 1 − x2 )(1 + 1 − x2 ) Rationalize the
lim = lim p
x→0 x x→0 x(1 + 1 − x2 ) numerator
1 − (1 − x2 ) Distribute the numer-
= lim √
x→0 x(1 + 1 − x2 ator using (3.2.1)
1 − 1 + x2
= lim p
x→0 x(1 + 1 − x2 )
x2
= lim p
x→0 x(1 + 1 − x2 )
x
= lim √ Simplify
x→0 1 + 1 − x2
0
= √ Substitute x with 0
1 + 1 − 02
0 0 0
= √ = = = 0
1+ 1 1+1 2
(h) Solution.
√ √
x−2 4−2
lim = Substitute x with 4
x→4 −3x + 11 −3(4) + 11
2−2 0
= = =0
−12 + 11 −1
Note. Here we don’t need to rationalize because when we substitute x with 4,
0
we get −1 which is a defined fraction. (A fraction is undefined only when the
denominator is 0.)
√ √
(i) Solution. Substituting x with 2 into f (x) = 2x+5− x−2
x+7
, we get 00 , which is
undefined. To find lim f (x), we need to first rationalize the numerator of f (x).
x→2
√ √ √ √ √ √
2x + 5 − x + 7 ( 2x + 5 − x + 7)( 2x + 5 + x + 7)
lim = lim √ √
x→2 x−2 x→2 (x − 2)( 2x + 5 + x + 7)
(2x + 5) − (x + 7) Distribute the
= lim √ √
x→2 (x − 2)( 2x + 5 + x + 7) numerator
x−2
= lim √ √
x→2 (x − 2)( 2x + 5 + x + 7)
1
= lim √ √ Simplify
x→2 2x + 5 + x + 7
1
=p √ Substitute x with 2
2(2) + 5 + 2 + 7
1 1 1
=√ √ = =
9+ 9 3+3 6
108 Chapter 3. Limits and Derivatives
(a) The function f has two pieces: x2 + x + 1 for x ≤ 1, and x + 2 for x > 1. (A
function that has more that one piece is called a piecewise function.) To find
f (−2) we use the first piece because −2 ≤ 1. So
f (1) = 12 + 1 + 1 = 1 + 2 = 3
3.2. Finding Limits Using Rules and One-Sided Limits 109
f (2) = 2 + 2 = 4
(d) Since the second piece of g(x), x2 − 10, is defined for x ≥ −2, and since 0 ≥ −2,
it follows that
g(0) = 02 − 10 = −10
(e) We want to find lim f (x). When we write x → 1, it means that x can get closer
x→1
to 1 from the left (x < 1) or x can approach 1 from the right (x > 1). Since
f (x) consists of two pieces and the piece to the left of 1 (x2 + x + 1) is different
from the piece to the right of 1 (x + 2), we need to find one-sided limits.
Limit from the left.
Since the limit from the left is equal to the limit from the right, it follows that
lim f (x) exists and is equal to 3. That is, lim f (x) = 3.
x→1 x→1
Since the limit from the left is not equal to limit from the right, lim g(x) does
x→−2
not exist.
(a) The function f consists of three pieces: First piece: x − 5 for x < −2. Second
2
piece: 0 for −2 ≤ x < 4. Third piece: x2 − 8 for x ≥ 4. To find f (−2), we use
the second piece (because −2 ≤ −2 < 4) and we get f (−2) = 0.
(b) Since 4 ≥ 4, f (4) is obtained by substituting x with 4 into the third piece, that
is,
42 16
f (4) = −8= −8=8−8=0
2 2
(c) To find lim f (x), we need to find one-sided limits.
x→−2
110 Chapter 3. Limits and Derivatives
Since the limit from the left does not match the limit from the right, it follows
that lim f (x) does not exist.
x→−2
x2 42
lim f (x) = lim+ −8 = −8=0
x→4+ x→4 2 2
(a) Solution.
Absolute Value
The absolute value of a number x, demoted |x|, is defined as x if x ≥ 0.
For x < 0, the absolute value of x is defined to be −x. So
(
x if x ≥ 0
|x| =
−x if x < 0
Step 1. Remove all the absolute values by using the definition and
rewrite the function as a piecewise function.
Let f (x) = |x − 2|. First, we need to take away the absolute value symbol.
By definition,
x−2 if x − 2 ≥ 0 x−2 if x − 2 ≥ 0
f (x) = =
−(x − 2) if x − 2 < 0 −x + 2 if x − 2 < 0
x−2 if x ≥ 2
=
−x + 2 if x < 2
3x + (x + 6) if x + 6 ≥ 0 3x + x + 6 if x + 6 ≥ 0
f (x) = =
3x + (−(x + 6)) if x + 6 < 0 3x − x − 6 if x + 6 < 0
4x + 6 if x + 6 ≥ 0 4x + 6 if x ≥ −6
= =
2x − 6 if x + 6 < 0 2x − 6 if x < −6
x+3
= lim− Simplify
x→1 −1
1+3
= = −4 Substitute x with 1
−1
x2 + 2x − 3 (x − 1)(x + 3)
lim+ f (x) = lim+ = lim+ Factor
x→1 x→1 x−1 x→1 x−1
x+3
= lim+ Simplify
x→1 1
1+3
= =4 Substitute x with 1
1
Since the limit from the left is not equal to the limit from the right, it follows
2 +2x−3
that lim x |x−1| does not exist.
x→1
Suppose we want to find lim f (x). If we substitute x with a and we run into
x→a
a problem of the form k0 , where k 6= 0, we have to find one-sided limits. To do
this, one can perform the so-called Sign Analysis of f (x). (This is illustrated
below step by step.)
If f (x) is negative when x is close to a and x < a, then the limit from
the left is lim− f (x) = −∞.
x→a
If f (x) is positive when x is close to a and x < a, then the limit from the
left is lim− f (x) = ∞.
x→a
If f (x) is negative when x is close to a and x > a, then the limit from
the right is lim+ f (x) = −∞.
x→a
If f (x) is positive when x is close to a and x > a, then the limit from the
right is lim+ f (x) = ∞.
x→a
x+1
Let us explain how to perform the sign analysis with the limit lim .
x→3 −3x+9
3.3. Infinite Limits and Vertical Asymptotes 113
−∞ −1 3 ∞
Step 4. Find the sign of f (x). The numbers −1 and 3 define three intervals:
(−∞, −1), (−1, 3), and (3, ∞). The goal of this step is to find the sign
of f (x) on each interval. To get the sign on a given interval, choose an
arbitrary number in that interval. Then substitute that number into the
function. If the result is positive then the function is positive on the
interval. If the result is negative, then the function is negative on the
x+1
interval. Let us illustrate this with our function f (x) = −3x+9 .
Finding the sign of f (x) on the interval (−∞, −1). Choose for exam-
−2+1
ple −2. Then f (−2) = −3(−2)+9 = −1
15
. Since the result is negative,
the function f (x) is negative on the interval (−∞, −1) as shown in
Figure 3.2.
−
−∞ −1 3 ∞
choose −2
−1
f (−2) = 15
<0
x+1
Figure 3.2: Sign of f (x) = −3x+9
on the interval (−∞, −1)
Finding the sign of f (x) on the interval (−1, 3). Choose for example
0+1
0. Then f (0) = −3(0)+9 = 19 > 0. Because the result is positive, the
function is positive on the interval (−1, 3) as shown in Figure 3.3.
+
−∞ −1 3 ∞
choose 0
1
f (0) = 9
>0
x+1
Figure 3.3: Sign of f (x) = −3x+9
on the interval (−1, 3)
114 Chapter 3. Limits and Derivatives
−
−∞ −1 3 ∞
choose 4
f (4) = − 35 < 0
x+1
Figure 3.4: Sign of f (x) = −3x+9
on the interval (3, ∞)
Finding the sign of f (x) on the interval (3, ∞). Choose for example
4+1 5
4. Then f (4) = −3(4)+9 = −3 < 0. Since the result is negative, the
function is negative on the interval (3, ∞) as shown in Figure 3.4.
Putting all these together, we get the sign of f (x) as shown in Figure 3.5.
− + −
−∞ −1 3 ∞
x+1
Figure 3.5: Sign Analysis for f (x) = −3x+9
x+1
lim− = ∞.
x→3 −3x + 9
Limit from the right. The sign analysis shows that f (x) is negative
when x is close to 3 and x > 3. So a similar reasoning as before
enables us to conclude that
x+1
lim+ = −∞.
x→3 −3x + 9
Conclusion: Because the limit from the left is not equal to the limit from the right,
x+1
we can conclude that lim −3x+9 does not exist.
x→3
Finding the numbers that make the numerator 0. Set 4 − x = 0. Solving this
equation for x, we get x = 4.
Finding the numbers that make the denominator 0. Set x2 −4x+4 = 0. Factoring
the left hand side, we get (x − 2)(x − 2) = 0. This implies that x = 2.
Marking the numbers 2 and 4 on the real line, we get Figure 3.6.
−∞ 2 4 ∞
Figure 3.6
+ + −
−∞ 2 4 ∞
choose 0 choose 3 choose 5
4 1
f (0) = 4
>0 f (3) = 1
>0 f (5) = − 19 < 0
4−x
Figure 3.7: Sign Analysis for f (x) = x2 −4x+4
− −
−∞ −2 ∞
choose −3 choose 0
f (−3) = − 51 < 0 f (0) = − 54 < 0
−5
Figure 3.8: Sign Analysis for f (x) = (x+2)2
Alternate Solution
−5
For the function f (x) = (x+2)2 , the numerator, −5, is always negative. And the
2
denominator, (x+2) , is a perfect square; so it always positive for every x 6= −2.
−5
So the fraction (x+2)2 is always negative for every x 6= −2. And therefore the
−5
limit from either side is −∞. This implies that lim (x+2) 2 = −∞.
x→−2
x2 + 3x − 4 (x − 1)(x + 4)
lim 2
= lim Factor
x→1 x − 2x + 1 x→1 (x − 1)2
x+4
= lim Simplify
x→1 x − 1
1+4
= Substitute x with 1
1−1
5 k
= Problem of the form
0 0
Because we have a problem of the form k0 , we need to find the one-sided limits and see
if they are equal or not. First, we need to perform the sign analysis of the function
f (x) = x+4
x−1
.
+ − +
−∞ −4 1 ∞
choose −5 choose 0 choose 2
−1 4 6
f (−5) = −6
>0 f (0) = −1
<0 f (2) = 1
>0
x+4
Figure 3.9: Sign Analysis for f (x) = x−1
x+4 x+4
lim− = −∞ and lim+ = ∞.
x→1 x−1 x→1 x−1
x+4
Since the one-sided limits do not match, lim does not exist.
x→1 x−1
7x 7(5) 35
(a) Substituting x with 5 into f (x) = (x−5) 3 , we get (5−5)3 = 0 , which is a problem
+ − +
−∞ 0 5 ∞
choose −1 choose 3 choose 6
−7 21 42
f (−1) = −216
>0 f (3) = −8
<0 f (6) = 1
>0
7x
Figure 3.10: Sign Analysis for f (x) = (x−5)3
7x 7x
lim− = −∞ and lim+ =∞
x→5 (x − 5)3 x→5 (x − 5)3
Since these limits do not match, it follows that lim f (x) does not exist.
x→5
(b) Since lim− f (x) = −∞ (or since lim+ f (x) = ∞), it follows (by the definition
x→5 x→5
of vertical asymptote) that the line x = 5 is a vertical asymptote of the curve
y = f (x).
Step 2. For every solution found in Step 1, find the limit of f (x) as x
approaches that solution. Let a be a solution to the equation q(x) =
0.
If one of the limits lim f (x), lim− f (x), or lim+ f (x) is ±∞,
x→a x→a x→a
then the line x = a is a vertical asymptote of f .
Otherwise, the line x = a is not a vertical asymptote.
x2 +x−6
We want to find the vertical asymptotes of f (x) = x2 −x−2
.
x+3
= lim Simplify
x→2 x + 1
2+3 5
= = Substitute x with 2
2+1 3
Since lim f (x) is a number, it follows that the line x = 2 is not a vertical
x→2
asymptote.
Finding the limit as x approaches −1. Substituting x with −1 into f (x), we
run into a problem of the form k0 with k 6= 0 (actually k = −6). To solve this
problem, we need to perform the sign analysis of f (x).
The numerator of f (x) is 0 when x = 2 or x = −3. And the denomina-
tor is 0 when x = 2 or x = −1. These numbers determine four intervals:
(−∞, −3), (−3, −1), (−1, 2), and (2, ∞). The sign of f (x) in each of these in-
tervals is shown in Figure 3.11.
From the sign analysis, we have
+ − + +
−∞ −3 −1 2 ∞
choose −4 choose −2 choose 0 choose 3
1 3
f (−4) = 3
>0 f (−2) = −1 < 0 f (0) = 3 > 0 f (3) = 2
>0
x2 +x−6
Figure 3.11: Sign Analysis of f (x) = x2 −x−2
Since lim f (x) = −∞ (or since lim + f (x) = ∞), it follows that the line
x→−1− x→−1
x = −1 is a vertical asymptote of the curve y = f (x).
(a) In section 3.2, we used nine rules to find the limits of various functions as x
approaches a number. In this section we are interested in limits at infinity. We
will need the following rules.
lim xn = ∞.
x→∞
(e)
√
3
1
lim x = lim x 3
x→∞ x→∞
120 Chapter 3. Limits and Derivatives
=∞ Rule 10
(ii) If k > 0 and lim f (x) = −∞, then lim kf (x) = −∞.
x→a x→a
(f ) For the limit lim 3x2 , we have k = 3 and lim x2 = ∞. Using Property (i), we
x→∞ x→∞
have that lim 3x2 = ∞,
x→∞
Let P (x) be a polynomial. Then the limit of P (x) is the limit of the term with
respect to the highest power. That is, if P (x) = an xn +an−1 xn−1 +· · ·+a1 x+a0
is a polynomial of degree n, then
1
(a) Solution. Using Rule 12, we get lim 4 = 0.
x→−∞ x
(b) Solution.
5 5 1
lim 9
= lim 9
x→−∞ 3x 3 x→−∞ x
5
= (0) Rule 12
3
=0
(d) Solution.
Step 1. Divide each term on top and bottom by the highest power of
x in the denominator.
Step 3. Use the rules for limits, including the rules for limits at infin-
ity.
x2 +1
For the limit lim3x 2 +x+1 , the highest power of x in the denominator is x2 . So
x→∞
we need to divide each term by x2 .
x 2
x2 + 1 x2
+ x12
lim = lim 2 Divide each term by x2
x→∞ 3x2 + x + 1 x→∞ 3x + x + 1
x2 x2 x2
1 + x12
= lim Simplify
x→∞ 3 + 1 + 12
x x
lim 1 + x12
x→∞
= 1 1
Rule 4
lim 3+ x
+ x2
x→∞
1+0
= Rule 1 and Rule 12
3+0+0
1
=
3
7x3
(e) Solution. For the limit lim 3 , the highest power of x in the denominator
x→−∞ −4
x
122 Chapter 3. Limits and Derivatives
7x 3
7x3 3
lim 3 = lim x3 x Divide each term by x3
x→−∞ x − 4 x→−∞ − 4
x3 x3
7
= lim Simplify
x→−∞ 1 − x43
lim (7)
x→−∞
= 4
Rule 4
lim 1− x3
x→−∞
lim (7)
x→−∞
= 4
Rule 2
lim (1) − lim x3
x→−∞ x→−∞
lim (7)
x→−∞
= 1
Rule 1
lim (1) − 4 lim x3
x→−∞ x→−∞
7
= Rule 1 and Rule 12
1 − 4(0)
7
= =7
1−0
(f ) Solution.
−3x 5
−3x + 5 x2
+ x2
lim = lim x2
Divide each term by x2
x→−∞ x2 − x + 1 x→−∞ x2
− x
x2
+ 1
x2
−3
+ x52 x
= lim Simplify
x→−∞ 1 − 1 + 12
x x
−3 5
lim x
+ x2
x→−∞
= 1 1
Rule 4
lim 1− x
+ x2
x→−∞
(g) Solution.
3x 2
3x2 − 2x x3
− 2x
x3
lim = lim 3 2 Divide each term by x3
x→∞ −5x3 + x2 − x x→∞ − 5x + x − x
x3 x3 x3
3
− x22 x
= lim Simplify
x→∞ −5 + 1 − 1
x x2
3 2
lim x
− x2
x→∞
= 1 1
Rule 4
lim −5 + x
− x2
x→∞
(h) Solution.
2x3 2
2x3 − x2 + 3 x2
− xx2 + x32 Divide each
lim = lim
x→∞ −4x2 + 3x − 1 x→∞ −4x2
+ 3x − x12 term by x2
x2 x2
2x − 1 + x32
= lim Simplify
x→∞ −4 + x3 − x12
3
lim 2x − 1 + x2
x→∞
= 3 1
Rule 4
lim −4 + x
− x2
x→∞
lim (2x − 1) + 0
x→∞
= Rule 12
−4 + 0 − 0
lim (2x − 1) 1
x→∞
= = − lim (2x − 1)
−4 4 x→∞
1 1
= − lim (2x) = − (∞) = −∞
4 x→∞ 4
124 Chapter 3. Limits and Derivatives
Solution.
x7 x3
x 7 − x3 + 1 x4
− x4
+ x14 Divide each
lim = lim
x→−∞ −x4 + 3x2 + 8 x→−∞ −x4
+ 3x2
+ x84 term by x4
x4 x4
x3 − x1 + x14
= lim Simplify
x→−∞ −1 + x32 + x84
1 1
lim (x3 ) − lim + lim
x x4
x→−∞ x→−∞ x→−∞ Rule 4 and
= 3
8
lim (−1) + lim x2
+ lim x4
Rule 2
x→−∞ x→−∞ x→−∞
lim (x3 ) − 0 + 0
x→−∞
= Rule 12
−1 + 0 + 0
lim (x3 )
x→−∞
=
−1
−∞
= =∞
−1
(i) Solution.
√ 1
x2 2
x + x2 x2
+ xx2
lim = lim 2x x2 Divide each term by x2
x→∞ 2x − x2 x→∞ − x2
x2
1
3 +1
= lim x22 Simplify
x→∞
x
− 1
0+1
= Rule 4, Rule 2, and Rule 12
0−1
1
= = −1
−1
√
For the limit lim x2 + 3 − x, we first need to rationalize.
x→∞
√ √
√ ( x2 + 3 − x)( x2 + 3 + x)
lim x2 + 3 − x = lim √
x→∞ x→∞ x2 + 3 + x
(x2 + 3) − x2
= lim √
x→∞ x2 + 3 + x
3
= lim √ .
x→∞ x2 + 3 + x
From there, we can use the same method as above (that is, divide each term by
the largest power of x in the denominator). It turns out that this does not work
very well when dealing with radicals and limits as x goes to ±∞. Luckily, there
is another technique, that consists of pulling out the largest power
√ of x in the
2
numerator and denominator, and then simplify. For example, x + 3 becomes
s √ r r r
3 3 3 3
x2 1 + 2 = x2 1 + 2 = |x| 1 + 2 = x 1 + 2
x x x x
The latter equality follows from the fact that when x goes to positive infinity, x
is positive, and therefore |x| = x. (If it was x → −∞, then |x| = −x.) So the
limit above becomes
√ 3
lim x2 + 3 − x = lim q
x→∞ x→∞
x 1 + x32 + x
3
= lim q Factor the denominator
x→∞ 3
x 1+ x2
+1
3
= lim q x
x→∞
1 + x32 + 1
0 0
=√ = =0 Use the rules
1+0+1 2
126 Chapter 3. Limits and Derivatives
x+1
= lim √
x→∞ x2 + x + 1 + x
x 1 + x1
Pull out x on the nu-
= lim q merator and x2 in the
x→∞
x2 1 + xx2 + x12 + x
radical symbol
x 1 + x1
= lim √ q
x→∞
x2 1 + xx2 + x12 + x
√
x 1 + x1
x2 = |x| = x (x → ∞ im-
= lim q
x→∞
x 1 + xx2 + x12 + x plies that x > 0)
1
1+ x
= lim q Simplify
x→∞ 1 1
1+ x
+ x2
+1
1+0
=√ Use the rules
1+0+0+1
1 1
=√ =
1+1 2
5. [Exercise on page 16]
3
(a) f (x) = 4x +1
x3 −x
Solution.
Horizontal Asymptotes
Let f be a function, and let b be a real number.
4x3 +1
Consider the function f (x) = x3 −x
.
3.5. Continuity 127
x3
x3 x2 x ∞
lim f (x) = lim 2 = lim x2 1
= lim 1 = = ∞.
x→∞ x→∞ x + 1 x→∞ + x→∞ 1 + x2 1+0
x2 x2
Likewise, lim f (x) = −∞. Since these are not real numbers, there is
x→−∞
no horizontal asymptote.
13 + 8000
x
= lim Simplify
x→∞ 1
13 + 0 k
= = 13 lim =0
1 x→∞ x
Interpreting the answer. The average cost approaches $13 as the number of
items gets larger and larger.
3.5 Continuity
1. [Exercise on page 16]
Solution.
128 Chapter 3. Limits and Derivatives
In Figure 3.5, there are four points where the graph is not continuous:
Regarding g(x),
– condition (1) is satisfied because g(2) is defined as g(2) = 3.
– For condition (2), we need to find the limit of g(x) as x approaches 2.
x2 − 4
lim g(x) = lim
x→2 x→2 x−2
(x − 2)(x + 2)
= lim Factor the numerator
x→2 x−2
= lim (x + 2) Simplify
x→2
So g is not continuous at 2.
For the function h(x),
– condition (1) is satisfied because h(2) is defined as h(2) = 4.
– For condition (2), we have
x2 − 4
lim h(x) = lim = 4.
x→2 x→2 x−2
(a) Solution. The function f consists of two pieces: x + 2 and x2 . The first piece
is defined for every x less than 0, and the second piece is defined for every x
greater than 0. None of these is defined precisely at x = 0. So the function f
is discontinuous at 0 because it is not defined there. (Condition (1) is not
satisfied.)
(b) Solution.
2
The function f has two pieces: 2x − 1 and x+1 x
. The first piece is defined
for every x less than or equal to 1. In particular, the first piece is defined
at x = 1. The second piece is not defined at 1 because we have a strict
inequality. So f (1) is defined and f (1) = 2(1) − 1 = 2 − 1 = 1. This implies
that condition (1) is satisfied.
Checking condition (2). We want to find lim f (x) (if it exists). Since the
x→1
pieceto the left of 1 (2x − 1) is not the same as the piece to the right of 1
x2
x+1
, we need to find the one-sided limits and see if they are equal.
130 Chapter 3. Limits and Derivatives
– Limit from the left. As x approaches 1 from the left, x is less than 1.
So, for the limit from the left, we use the piece 2x − 1:
x2 12 1
lim+ f (x) = lim+ = = .
x→1 x→1 x+1 1+1 2
Since the limit from the left is not equal to the limit from the right, it follows
that lim f (x) does not exist. So f is not continuous at 1.
x→1
(c) Solution.
√
Condition
√ (1) is satisfied because f (−2) is defined as f (−2) = −2 + 3 =
1 = 1.
Checking condition (2). Because the piece to the left of −2 is not the same
as the piece to the right of −2, we need to find one-sided limits.
√ √
lim + f (x) = lim + x+3= −2 + 3 = 1.
x→−2 x→−2
Since the limit from the left is not equal to the limit from the right, lim f (x)
x→1
does not exist. Therefore f is not continuous at −2.
(d) Solution.
Condition (1) is satisfied because f (0) is defined as f (0) = 2.
Checking condition (2). Because the piece to the left of 0 is not the same
as the piece to the right of 0, we need to find one-sided limits.
Since the limit from the left equals the limit from the right, the limit of f (x)
as x approaches 0 exists and lim f (x) = 1.
x→0
Condition (3) is not satisfied because lim f (x) 6= f (0).
x→0
So f is not continuous at 0.
(e) Solution.
Condition (1) holds because f (3) is defined as
Checking condition (2). The limit of f (x) as x approaches 3 from the left is
x2 − 2x − 3
lim− f (x) = lim−
x→3 x→3 −2x + 6
(x − 3)(x + 1)
= lim− Factor the numerator
x→3 −2(x − 3)
x+1
= lim− Simplify
x→3 −2
3+1
= = −2. Substitute x with 3
−2
And the limit from the right is
Since the limit from the left equals the limit from the right, the limit of f (x)
as x approaches 3 exists and lim f (x) = −2.
x→3
Condition (3) is satisfied because lim f (x) = f (3).
x→3
Since all the conditions are satisfied, the function f is continuous at x = 3.
(f ) Solution.
1−1 0
Condition (1) holds because f (1) is defined as f (1) = e 2 = e2 = 12 .
Checking condition (2). The limit of f (x) as x approaches 1 from the left is
ex−1 e1−1 1
lim− f (x) = lim− = = .
x→1 x→1 2 2 2
And the limit from the right is
√
x−1
lim+ f (x) = lim+
x→1 x→1 x−1
√ √
( x − 1)( x + 1)
= lim+ √ Rationalize the numerator
x→1 (x − 1)( x + 1)
x−1
= lim+ √ Distribute the numerator
x→1 (x − 1)( x + 1)
1
= lim+ √ Simplify
x→1 x+1
1 1
=√ = Substitute x with 1
1+1 2
Because the limit from the left equals the limit from the right, the limit of
f (x) as x approaches 1 exists and lim f (x) = 21 .
x→1
Condition (3) holds because lim f (x) = f (1).
x→1
So the function f is continuous at x = 1.
132 Chapter 3. Limits and Derivatives
(a) Solution.
Condition (1) holds because f (2) is defined (for every k) as f (2) = k(2)2 −
6 = 4k − 6.
Condition (2):
lim− f (x) = lim− (kx2 − 6) = k(2)2 − 6 = 4k − 6.
x→2 x→2
and
lim f (x) = 6 + k = 6 + 4 = 10.
x→2+
So lim f (x) = 10 = f (2). And therefore, condition (3) holds for the value
x→2
k = 4.
3.5. Continuity 133
2x2 − 3x − 5
lim f (x) = lim
x→−1 x→−1 x+1
(x + 1)(2x − 5)
= lim Factor the numerator
x→−1 x+1
2x − 5
= lim Simplify
x→−1 1
= 2(−1) − 5 = −7 Substitute x with −1
−7 = −k + 5
−12 = −k Subtract 5 from both sides
12 = k Divide both sides by −1
(c) Solution.
Condition (1) is satisfied (for every k) because f (1) is defined as f (1) =
k 2 − 5k.
Condition (2):
And
lim f (x) = lim+ (−6x) = −6(1) = −6.
x→1+ x→1
Since the limit from the left is equal to the limit from the right, it follows
that lim− f (x) exists and is equal to −6. So condition (2) holds (for every
x→1
k).
Condition (3) holds if and only if lim f (x) = f (1), that is,
x→1
−6 = k 2 − 5k
0 = k 2 − 5k + 6 Add 6 to both sides
k 2 − 5k + 6 = 0
134 Chapter 3. Limits and Derivatives
(k − 2)(k − 3) = 0 Factor
k = 2 or k = 3
On the interval (−7, −5). The function is continuous there because, from the
graph, it is continuous at every point between −7 and −5.
On the interval (−6, −4). The function is not continuous on (−6, −4) because,
from the graph, it is discontinuous at −5 (as f (−5) is undefined).
On the interval [0, 2].
– Clearly f is continuous on the open interval (0, 2) because it is continuous
at every point in (0, 2).
– The function is continuous from the right at x = 0 because lim+ f (x) = 5 =
x→0
f (0).
3.5. Continuity 135
– The function is not continuous from the left at x = 2 because the limit
lim− f (x) is not equal to f (2). Indeed, lim− f (x) = 5 and f (2) = 4.
x→2 x→2
Thus, f is not continuous on the closed interval [0, 2].
On the interval [5, 6] the function is continuous because it clearly satisfies the
three conditions above.
On the interval [4, 6].
– The function is continuous on the open interval (4, 6) because it is continuous
at every point there.
– The function is continuous from the right at x = 4 because lim+ f (x) =
x→4
−2 = f (4).
– The function is continuous from the left at x = 6 because lim− f (x) = 2 =
x→6
f (6).
Thus, f is continuous on the closed interval [4, 6].
On the interval [3, 4].
– The function is continuous on the open interval (3, 4) because it is continuous
at every point there.
– The function is continuous from the right at x = 3 because lim+ f (x) = 2 =
x→3
f (3).
– The function is not continuous from the left at x = 4 because the limit
lim− f (x) is not equal to f (4). Indeed, lim− f (x) = 1 and f (4) = −2.
x→4 x→4
Hence, f is not continuous on the closed interval [3, 4].
Checking the continuity on the open interval (−3, 3). Let c be a number in
(−3, 3).
√
– f (c) = 9 − c2 is defined because c is between −3 and 3.
√ √
– lim f (x) = lim 9 − x2 = 9 − c2 .
x→c x→c
– lim f (x) = f (c).
x→c
Thus f is continuous at c.
Checking the continuity from the right at x = −3. We have
√ p √
lim + f (x) = lim + 9 − x2 = 9 − (−3)2 = 0 = 0 = f (−3).
x→−3 x→−3
On the other hand f (1) = 7(1) − 2 = 5. Since lim+ f (x) 6= f (1), it follows that
x→1
f is not continuous from the right at x = 1.
Checking the continuity from the left at x = 4.
So f is not continuous on the closed interval [1, 4] because it is not continuous from
the right at x = 1.
and
lim f (x) = lim+ (x) = 0.
x→0+ x→0
Thus lim f (x) exists and is equal to 0. Moreover, lim f (x) = f (0). So f is
x→0 x→0
continuous at 0.
The pieces x + 2 and x2 are continuous everywhere (for every k) because they
are polynomials.
Continuity at x = k.
– The function is defined at k and f (k) = k + 2.
– Finding one-sided limits:
lim f (x) = lim− (x + 2) = k + 2,
x→k− x→k
Root
pFunction. If f (x) is continuous on an interval I, then the function
y = f (x) is continuous for all x in I where f (x) ≥ 0.
The function l(x) = ln(−3x + 12) is continuous for all x where −3x + 12 > 0.
We need to solve this inequality.
−3x + 12 > 0
−3x > −12 Subtract 12 from both sides
1 1
x < −12 − Multiply both sides by k = −
3 3
x<4
Since the limit from the left is not equal to the limit from the right, the function
is not continuous at 0.
Checking the continuity at 3.
This shows that the limit exists and lim f (x) = 7. Furthermore, f (3) = 10−3 =
x→4
7 = lim f (x). Thus, f is continuous at 3.
x→3
Conditions (i) and (ii) are called hypotheses. And the sentence “then there
exists a number c in (a, b) such that f (c) = N ” is called the conclusion or
thesis. To apply the theorem, we have to check the hypotheses first.
Let f (x) = x3 +x−1. Clearly f is continuous on the closed interval [0, 1] because
it is a polynomial.
Finding the values of f at the endpoints: f (0) = −1 and f (1) = 1 + 1 − 1 = 1.
Finding N . The original equation is f (x) = 0. So N = 0. (If the equation is
f (x) = B, then N is B.)
Clearly N is between f (0) and f (1).
So by the Intermediate Value Theorem, there exists c in (0, 1) such that f (c) = 0.
So by the Intermediate Value Theorem, there exists c in (−1, 2) such that f (c) = 7.
If one unit costs $15, x units will cost 15x, and this holds for all x ≤ 280.
If one unit costs $12, x units will cost 12x, and this holds for all x > 280.
So the cost function is
15x if x ≤ 280
C(x) =
12x if x > 280
Since the limit from the left is not equal to the limit from the right, the function
C is not continuous at x = 280. Thus, C is discontinuous at x = 280.
(a) Solution.
f (b) − f (a)
b−a
3.6. Rates of Change 141
Here f (x) = 10, a = 1, b = 2. The average rate of change of f (x) over the
interval [1, 2] is
f (2) − f (1) 10 − 10 0
= = =0
2−1 1 1
(b) Solution. On the interval from x = −3 to x = 5, the average rate of f (x) = −π 2
is
f (5) − f (−3) −π 2 − (π 2 ) −π 2 + π 2 0
= = = =0
5 − (−3) 5+3 8 8
(c) Solution. The average rate of change of f (x) = 4x + 5 over the interval [0, 7] is
f (7) − f (0) [4(7) + 5] − [4(0) + 5] (28 + 5) − (0 + 5)
= =
7−0 7 7
33 − 5 28
= = =4
7 7
(d) Solution. First we need to find f (−2) and f (8).
f (−2) = −(−2)2 + 3(−2) + 8 = −(4) − 6 + 8 = −4 − 6 + 8 = −2
f (8) = −(8)2 + 3(8) + 8 = −64 + 24 + 8 = −32
The average rate of change over the interval [−2, 8] is
f (8) − f (−2) −32 − (−2) −32 + 2 −30
= = = = −3
8 − (−2) 8+2 10 10
(e) Solution. First we need to find f (−11) and f (−1).
f (−11) = 6 + (−11) − (−11)3 = 6 − 11 − (−1331) = −5 + 1331 = 1326
f (−1) = 6 + (−1) − (−1)3 = 6 − 1 − (−1) = 5 + 1 = 6
The average rate of change over the interval [−11, −1] is
f (−1) − f (−11) 6 − 1326 6 − 1326 −1320
= = = = −132
−1 − (−11) −1 − (−11) −1 + 11 10
1 1 1
f ( 12 ) − f ( 14 ) e2( 2 ) − e2( 4 ) e1 − e 2
1 = =
2
− 41 2
4
− 41 1
4
√ a ac
= 4 e − e ≈ 4.278 b
=
c
b
142 Chapter 3. Limits and Derivatives
(h) Solution. The average rate of change of f (x) = ln(1 + x2 ) over the interval
from x = − 13 to x = 51 is
1
f ( 15 ) − f (− 13 ) − ln 1 + 91
ln 1 + 25
1 =
5
− (− 13 ) 1
5
+ 13
26
− ln 10
ln 25 9 a c ad + bc
= 3+5 + =
15
b d bd
26
− ln 10
ln 25 9 15 26 10
= 8 = ln − ln ≈ −0.124
15
8 25 9
(a) Solution.
f (a + h) − f (a)
lim (3.6.1)
h→0 h
= lim 3 Simplify
h→0
=3
= 12 − 0 = 12 Substitute h with 0
(a) Solution.
Let s(t) be the position of an object moving along a straight line at time
t.
s(t2 ) − s(t1 )
Average velocity between t1 and t2 =
t2 − t1
s(t1 + h) − s(t1 )
Instantaneous velocity at time t1 = lim
h→0 h
If the position is in kilometers (km) and time is in hours (h), then the unit
of the average (or instantaneous) velocity is km/h.
Here the position is s(t) = 25t2 . The average velocity of the car between t = 0
and t = 2 is
s(2) − s(0) 25(2)2 − 25(0)2 25(4) − 0 100
= = = = 50 km/h
2−0 2 2 2
s(9 + h) − s(9)
lim
h→0 h
Finding s(9 + h). This is obtained by substituting t with 9 + h into s(t). Since
s(t) = 31 t2 − 5t + 19,
1
s(9 + h) = (9 + h)2 − 5(9 + h) + 19
3
3.6. Rates of Change 145
1
= (81 + 18h + h2 ) − 5(9 + h) + 19 (a + b)2 = a2 + 2ab + b2
3
1
= 27 + 6h + h2 − 45 − 5h + 19 Distribute
3
1
= 1 + h + h2
3
Finding s(9). Substituting t with 9 into s(t), we get
1
s(9) = (9)2 − 5(9) + 19 = 27 − 45 + 19 = 1
3
The required instantaneous velocity is
1 + h + 31 h2 − (1)
s(9 + h) − s(9)
lim = lim
h→0 h h→0 h
1 2
h + 3h
= lim
h→0 h
h 1 + 13 h
= lim Factor the numerator
h→0 h
1
= lim 1 + h Simplify
h→0 3
1
= 1 + (0) = 1 Substitute h with 0
3
Here the position is in feet (ft) and time is in seconds (s). So the instantaneous
velocity when t = 9 is 1 ft/s.
(b) Solution. First we need to find s(13 + h)
1
s(13 + h) = (13 + h)2 − 5(13 + h) + 19
3
1
= (169 + 26h + h2 ) − 5(13 + h) + 19 (a + b)2 = a2 + 2ab + b2
3
169 26 1
= + h + h2 − 65 − 5h + 19 Distribute
3 3 3
31 11 1 2 a a ± bc
= + h+ h ±c=
3 3 3 b b
Substituting t with 13 into s(t), we get
1 169 169 169 − 138 31
s(13) = (13)2 − 5(13) + 19 = − 65 + 19 = − 46 = =
3 3 3 3 3
The instantaneous velocity when t = 13 is
31 11
+ 13 h2 − 31
s(13 + h) − s(13) 3
+ 3
h 3
lim = lim
h→0 h h→0 h
11
3
h + 31 h2
= lim
h→0 h
146 Chapter 3. Limits and Derivatives
11
+ 13 h
h 3
= lim Factor the numerator
h→0 h
11 1
= lim + h Simplify
h→0 3 3
11 1
= + (0) ≈ 3.67 ft/s Substitute h with 0
3 3
5. [Exercise on page 20]
(a) Solution. The average rate of change in the cost as the number of items
produced changes from 2 to 3 is
C(3) − C(2) [200 + 21(3) − (3)2 ] − [200 + 21(2) − (2)2 ]
=
3−2 1
(200 + 63 − 9) − (200 + 42 − 4) 254 − 238
= = = 16
1 1
(b) Solution. If the production is at the level of x = 2 items, the cost of producing
one additional item is
C(3) − C(2) = [200 + 21(3) − (3)2 ] − [200 + 21(2) − (2)2 ] = 254 − 238 = 16
We notice that this coincides with the answer found in part (a). This is true in
general.
C(n + 1) − C(n)
(c) Solution. The instantaneous rate of change of cost C(x) = 200 + 21x − x2 when
x = 2 is
C(2 + h) − C(2) [200 + 21(2 + h) − (2 + h)2 ] − [200 + 21(2) − (2)2 ]
lim = lim
h→0 h h→0 h
3.6. Rates of Change 147
C(n + h) − C(n)
C(n + 1) − C(n) ≈ lim
h→0 h
As we will see in the next section, the instantaneous rate of change is
nothing but the derivative.
This means that at the level of production of x = 1 item, the profit is decreasing
at the rate of $3 per item.
(d) Solution. The instantaneous rate of change of profit when x = 5 is
P (5 + h) − P (5)
lim
h→0 h
This means that when 5 items are manufactured, the profit is increasing at the
rate of $21 per item.
(a) Solution.
3.7. Definition of the Derivative 149
f (a + h) − f (a)
f 0 (a) = lim (3.7.1)
h→0 h
if this limit exists.
y
y = f (x)
T
P
f (a)
f 0 (a) = slope of T
a x
If the limit (3.7.1) does not exist, there is no tangent at the point
(a, f (a)).
Recall that the slope of the line through two points P1 (x1 , y1 ) and
−y1
P2 (x2 , y2 ) is m = xy22 −x 1
.
y
T1
6
−2 −1 1 2 3 4 x
−2
P1 −4
T2
−6
Figure 3.12
through the points (1, −2) and (0, 1). Since the slope of T2 is
1 − (−2) 1+2 3
m= = = = −3,
0−1 −1 −1
The tangent line to the curve at (1, f (1)) passes through the points (0, −1) and
(1, 0) as shown in Figure 3.7.
0−(−1)
So the slope of the tangent line is m = 1−0
= 0+1
1−0
= 1.
Therefore, the derivative of f at 1 is f 0 (1) = 1.
To find
f (a + h) − f (a)
f 0 (a) = lim ,
h→0 h
one can proceed as follows.
f (a + h) = (a + h)2 − 4(a + h) + 1.
x 2(a+h)−ea+h
If f (x) = 2x−e
√
5x3 − x+7
, then f (a + h) = √
5(a+h)3 − a+h+7
.
f (a+h)−f (a)
Step 3. Divide f (a + h) − f (a) by h. The fraction h
is called the
difference quotient.
We want to find the equation of the tangent line to the curve y = −x2 at (1, −1).
f (1+h)−f (1)
First we need to find the slope, which is given by m = f 0 (1) = lim h
.
h→0
– Finding f (1 + h). We have
Replace x with 1 + h
f (1 + h) = −(1 + h)2
into f (x) = −x2
= −(1 + h)(1 + h) = −(1 + h + h + h2 ) Distribute
= −1 − 2h − h2
f (1 + h) − f (1) = −1 − 2h − h2 − (−(−1)2 )
152 Chapter 3. Limits and Derivatives
= −1 − 2h − h2 − (−1)
= −1 − 2h − h2 + 1 = −2h − h2
f (1 + h) − f (1)
f 0 (1) = lim
h→0 h
−2h − h2
= lim
h→0 h
h(−2 − h)
= lim Factor the numerator
h→0 h
= lim (−2 − h) Simplify
h→0
An equation of the tangent line to the curve f (x) = −x2 at (1, −1) is
y − (−1) = m(x − 1)
y − (−1) = −2(x − 1) Substitute m with −2
y + 1 = −2x + 2
y = −2x + 1 Subtract 1 from both sides
Solution.
f (4+h)−f (4)
We first need to find the slope, which is given by m = f 0 (4) = lim h
.
h→0
√
– Finding f (4 + h). We have f (4 + h) = 4 + h.
– Finding f (4 + h) − f (4). We have
√ √ √
f (4 + h) − f (4) = 4+h− 4= 4 + h − 2.
√
f (4+h)−f (4) 4+h−2
– Dividing by h, we get h
= h
.
3.7. Definition of the Derivative 153
1
y − 2 = (x − 4)
4
1
y−2= x−1 Distribute the right hand side
4
1
y = x+1 Add 2 to both sides
4
Finding f (2 + h).
Finding f (2 + h) − f (2).
= −4 − 8h − 3h2 − (−12 + 8)
= −4 − 8h − 3h2 − (−4)
= −4 − 8h − 3h2 + 4 = −8h − 3h2
f (2+h)−f (2) −8h−3h2
Dividing by h, we get h
= h
.
Taking the limit as h approaches 0, we get
f (2 + h) − f (2) −8h − 3h2
f 0 (2) = lim = lim
h→0 h h→0 h
h(−8 − 3h)
= lim Factor
h→0 h
= lim (−8 − 3h) Simplify
h→0
Dividing by h, we get
h
f (3 + h) − f (3) 16+4h
=
h h
a
h b a
= =
(16 + 4h)h c bc
Taking the limit as h → 0, we get
f (3 + h) − f (3) h
f 0 (3) = lim = lim
h→0 h h→0 (16 + 4h)h
1
= lim Simplify
h→0 16 + 4h
1 1
= = Substitute h with 0
16 + 4(0) 16
8. [Exercise on page 22]
(a) Solution.
Short Solution
Since f (x) = 10 = 0x + 10 is a line, and since the slope of that line is
m = 0, it follows that f 0 (x) = 0.
(b) Solution. The function f (x) = −2x + 3 is a line with slope m = −2. So
f 0 (x) = −2.
5 − 4x 5 4 a±b a b
f (x) = = − x = ±
6 6 6 c c c
5 2
= − x,
6 3
(a) Solution.
f (x + h) − f (x)
f 0 (x) = lim Use Definition (3.7.2)
h→0 h
k−k
= lim f (x + h) = k
h→0 h
0
= lim = lim (0) = 0.
h→0 h h→0
(b) Solution.
f (x + h) − f (x)
f 0 (x) = lim Use Definition (3.7.2)
h→0 h
(x + h) − x Since f (x) = x, it follows
= lim
h→0 h that f (x + h) = x + h
x+h−x h
= lim = lim
h→0 h h→0 h
=1
3.7. Definition of the Derivative 157
(c) Solution.
f (x + h) − f (x)
f 0 (x) = lim Use Definition (3.7.2)
h→0 h
(x + h)2 − x2
= lim f (x + h) = (x + h)2
h→0 h
(x2 + 2xh + h2 ) − x2
= lim (a + b)2 = a2 + 2ab + b2
h→0 h
2xh + h2 h(2x + h)
= lim = lim Factor
h→0 h h→0 h
= lim (2x + h) Simplify
h→0
= 2x + 0 = 2x Substitute h with 0
(d) Solution.
f (x + h) − f (x)
f 0 (x) = lim Use Definition (3.7.2)
h→0 h
(x + h)3 − x3
= lim f (x + h) = (x + h)3
h→0 h
x3 + 3x2 h + 3xh2 + h3 − x3
= lim (a + b)3 = a3 + 3a2 b + 3ab2 + b3
h→0 h
3x2 h + 3xh2 + h3
= lim
h→0 h
h(3x2 + 3xh + h2 )
= lim Factor
h→0 h
= lim (3x2 + 3xh + h2 ) Simplify
h→0
(e) Solution.
f (x + h) − f (x)
f 0 (x) = lim Use Definition (3.7.2)
h→0 h
√ √
x+h− x √
= lim f (x + h) = x+h
h→0 h
√ √ √ √
x + h − x ( x + h + x)
= lim √ √ Rationalize
h→0 h( x + h + x)
(x + h) − x
= lim √ √ Distribute the numerator
h→0 h( x + h + x)
h
= lim √ √
h→0 h( x + h + x)
1
= lim √ √ Simplify
h→0 x+h+ x
1
=√ √ Substitute h with 0
x+0+ x
1 1
=√ √ = √
x+ x 2 x
(f ) Solution.
f (x + h) − f (x)
f 0 (x) = lim Use Definition (3.7.2)
h→0 h
1 1
x+h
− x 1
= lim f (x + h) =
h→0 h x+h
x−x−h
(x+h)x a c ad − bc
= lim − =
h→0 h b d bd
−h a
(x+h)x −h b a
= lim = lim =
h→0 h h→0 h(x + h)x c bc
−1
= lim Simplify
h→0 (x + h)x
1 1
=− = − 2.
(x + 0)x x
(a) Solution.
f (x + h) − f (x)
f 0 (x) = lim Use Definition (3.7.2)
h→0 h
−4(x + h) + 1 − (−4x + 1)
= lim f (x + h) = −4(x + h) + 1
h→0 h
−4x − 4h + 1 + 4x − 1 −4h
= lim = lim
h→0 h h→0 h
= lim (−4) = −4 Simplify
h→0
Since f 0 (x) = −4, it follows that f 0 (−3) = −4, f 0 (0) = −4, and f 0 (2) = −4.
(b) Solution.
Finding f (x + h). Substituting x with x + h into f (x) = 3x2 − 5x, we get
Finding f (x + h) − f (x).
(c) Solution.
160 Chapter 3. Limits and Derivatives
(d) Solution.
f (x + h) − f (x)
f 0 (x) = lim Use Definition (3.7.2)
h→0 h
p √
3(x + h) − 3x p
= lim f (x + h) = 3(x + h)
h→0 h
√ √ √ √
( 3x + 3h − 3x)( 3x + 3h + 3x)
= lim √ √ Rationalize
h→0 h( 3x + 3h + 3x)
(3x + 3h) − 3x
= lim √ √ Distribute the numerator
h→0 h( 3x + 3h + 3x)
3h
= lim √ √
h→0 h( 3x + 3h + 3x)
3
= lim √ √ Simplify
h→0 3x + 3h + 3x
3
=p √ Substitute h with 0
3x + 3(0) + 3x
3 3
=√ √ = √ .
3x + 3x 2 3x
Solution.
Differentiability
A function f is said to be differentiable at a point a if the limit
f (a + h) − f (a)
lim
h→0 h
exists. If this limit does not exist, we say that f is not differentiable at x = a.
f (1+h)−f (1)
limit f 0 (1) = lim h
does not exist.
h→0
f (1 + h) − f (1) |(1 + h) − 1| − |1 − 1|
lim = lim f (1 + h) = |(1 + h) − 1|
h→0 h h→0 h
|1 + h − 1| − |0| |h| − 0
= lim = lim
h→0 h h→0 h
|h|
= lim .
h→0 h
Limit from the left. As h approaches 0 from the left, h < 0, and therefore
|h| = −h. So
|h| −h
lim− = lim− = lim− (−1) = −1.
h→0 h h→0 h h→0
Limit from the right. As h approaches 0 from the right, h > 0, and therefore
|h| = h. So
|h| h
lim+ = lim+ = lim+ (1) = 1.
h→0 h h→0 h h→0
Because the limit from the left is not equal to the limit from the right, the limit
lim |h|
h
does not exist. Hence, f is not differentiable at x = 1.
h→0
(a) Solution.
Continuity vs Differentiability
Theorem 3.7.1. Let f be a function.
If f is differentiable at a, then f is continuous at a. So we have the
implication
f is differentiable at a =⇒ f is continuous at a.
The symbol “ =⇒ ” means “implies”.
If f is not continuous at a, then f is not differentiable at a. So we
have the implication
f is not continuous at a =⇒ f is not differentiable at a.
Note. The theorem does not say that if f is not differentiable, then f is
not continuous. For example, the absolute value function f (x) = |x − 1|
3.7. Definition of the Derivative 163
Because the limit from the left is not equal to the limit from the right,
the limit of f (x) as x approaches 1 does not exist. Therefore the second
condition of the definition of continuity is not satisfied. This implies
that f is not continuous at 1.
164 Chapter 3. Limits and Derivatives
Here the graph of f is a line. So the tangent line at every point coincides with
the graph of f .
Since the graph of f is a horizontal line, and since the slope of a horizontal line
is 0, it follows that the derivative at every point is 0. That is, f 0 (x) = 0 for
every x.
So the graph of f 0 is the line of equation y = 0, that is, the x-axis as shown
below.
3.8. Graphical Differentiation 165
y
f
2
f0
−3 3 x
y f
4
3
2
f0
1
−3 −2 −1 1 2 3 x
−1
1−3 1−3 −2
m= = = = −2
−1 − (−2) −1 + 2 1
y
3
2
1
−3 −2 −1 1 2 3 x
−1
f0
−2
−3
f
Solution. Here the graph of f is not a straight line. So the derivative is not a
constant function as before. To sketch the graph of f 0 , the key thing is to be able to
estimate the derivative f 0 (a) for a point x = a.
Step 1. Draw the tangent line to the graph of f at the point (a, f (a)).
Step 2. Choose two points on the tangent line and find the slope between
the two points.
Note. If the tangent line is horizontal, the slope is 0, and therefore the graph
of f 0 passes through the point (a, 0). Or, equivalently, the graph of f 0 intersects
the x-axis at a. Situations where f 0 (a) = 0 include the following.
f (a, f (a))
Tangent line Tangent line
(a, f (a)) f
f 0 (a) = 0 f 0 (a) = 0
f
Tangent line
(a, f (a)) Tangent line
(a, f (a))
0
f (a) = 0 f 0 (a) = 0 f
3.8. Graphical Differentiation 167
Step 1. Find the points (if any) where the tangent line is horizontal or
where the function is not differentiable. These points subdivide
the x-axis into a certain number of intervals (n points will produce
n + 1 intervals).
Step 2. Mark the x-coordinate of each point where the tangent line is hor-
izontal. Specifically, if (a, f (a)) is a point on the graph such that
f 0 (a) = 0, plot the point (a, 0).
Step 3. For each interval found in Step 1, choose one or two numbers in
the interval and estimate the derivative at those numbers. (Choose
more numbers if you want to improve the accuracy of the graph of
the derivative.) If f 0 (a) ≈ m, then plot the point (a, m).
Here we want to sketch the graph of the derivative of the following function.
y
f
1
1 3 5 x
−1
From the graph of f , we see that the tangent line at the point where x = 2 is
horizontal. This determines two open intervals: (−∞, 2) and (2, ∞).
Since the tangent line is horizontal at (2, −1), it follows that f 0 (2) = 0. Plotting
the point (2, 0), we have the following
y
f
1
1 3 5 x
−1
168 Chapter 3. Limits and Derivatives
On the interval (−∞, 2), we need to choose a number of points (one or two) and
estimate the derivative at those points.
5 f
1 3 5 x
−1
* Then we choose two points on the tangent line and compute the slope
m between the two points. Choosing for example the points (0, 3) and
(1, −1), we get
−1 − 3 −4
m= = = −4.
1−0 1
So f 0 (0) ≈ −4.
Note. We are just estimating the slope of the tangent line. So your
answer may be slightly different.
Plotting the point (0, −4), we get
5 f
1
1 3 5 x
−1
−3
– Choose another point in the interval (−∞, 2). Choose for example x = 1.
To estimate the derivative f 0 (1), we first draw the tangent line to the graph
of f at (1, 0).
3.8. Graphical Differentiation 169
5 f
1
1 3 5 x
−1
−3
Then we select two points on the tangent line. Choose for example the
points (0, 2) and (1, 0). The slope between these points is
0−2 −2
m= = = −2
1−0 1
So f 0 (1) ≈ −2. Plotting the point (1, −2), we have the following
y
5 f
1
1 3 5 x
−1
−3
Estimating the derivative at some points on the interval (2, ∞). We are going
to choose two points.
– Select the point (3, 0). The tangent line to the graph of f at the point (3, 0)
is shown below.
y
5 f
1
1 3 5 x
−1
−3
Using the points (2, −2) and (3, 0), the slope of the tangent line is
0 − (−2) 0+2 2
m= = = =2
3−2 1 1
170 Chapter 3. Limits and Derivatives
5 f
1
1 3 5 x
−1
−3
– Select another point in the interval (2, ∞). Choose for example the point
x = 4 (the corresponding point on the graph of f is (4, 3)). Using the same
approach as before, an estimate of the derivative at 4 is m = 4. So f 0 (4) ≈ 4.
Plotting the point (4, 4), we get
y
5 f
1
1 3 5 x
−1
−3
Connecting the points (0, −4), (1, −2), (2, 0), (3, 2) and (4, 4), we get the graph
of the derivative of f :
y
f f0
5
1
−1 1 3 5 7 x
−3
−5
3
2
1
−3 −2 −1 1 2 x
−1
−2
The point x = −1 determines two intervals: (−∞, −1) and (−1, ∞).
Choose two points (or more if we want to improve the accuracy of the graph)
in the interval (−∞, −1). Also choose two points in the interval (−1, ∞). Then
estimate the derivative at those points. Select for example the points −3, −2, 0,
and 1. The estimates at these points are given in the following table. (See
Exercise 4 where we cover some estimates of the derivative in detail.)
x −3 −2 −1 0 1
f 0 (x) ≈ 4 2 0 −2 −4
Plotting the points (−3, 4), (−2, 2), (−1, 0), (0, −2), (1, −4) and connecting them,
we get the graph of the derivative as shown in the following figure.
y
0
f 4
−4 −2 2 x
−2
−4 f
First, observe that the tangent line at (x, f (x)) is horizontal when x = 0 or
x = 2. This means that f 0 (0) = 0 and f 0 (2) = 0. Plotting the points (0, 0) and
(2, 0), we get the following.
172 Chapter 3. Limits and Derivatives
y
4
3
2
1
−1 1 2 3 4 x
−1
−2
The points 0 and 2 determine three intervals: (−∞, 0), (0, 2), and (2∞).
– On the interval (−∞, 0), tangent lines have a negative slope. So f 0 (x) < 0
for x < 0. This means that the graph of the derivative is below the x-axis
on (∞, 0).
– On the interval (0, 2), tangent lines have a positive slope. So f 0 (x) > 0 for
0 < x < 2. This means that the graph of the derivative is above the x-axis
on (0, 2).
– Similarly, the graph of f 0 is below the x-axis on the interval (2, ∞).
Notice from the graph of f that if we draw tangent lines at (x, f (x)), the largest
slope occurs at x = 1. This means that the largest value of f 0 is f 0 (1).
Choose the points −0.5, 1, 2.5. The estimates of the derivative at these points
are given in the following table. (See Exercise 4 where we cover some estimates
of the derivative in detail.)
x −0.5 0 1 2 2.5
f 0 (x) ≈ −3.75 0 3 0 −3.75
Connecting the points (−0.5, −3.75), (0, 0), (1, 3), (2, 0), (2.5, −3.75), we get the
graph of f 0 as shown in the following figure.
y
4
−1 1 3 x
−2
−4 f0 f
x −2 −1 0.5 2 3
0
f (x) ≈ 4 0 −2.25 0 4
Connecting the points (−2, 4), (−1, 0), (0.5, −2.25), (2, 0), (3, 4), we get the graph of
f 0 as shown in the following figure.
y
f0 f
4
−4 −2 2 4 x
−2
−4
y
4
−2 2 4x
−2
First, observe that the point (1, 3) is a sharp point or corner. (Remember that
a sharp point is a point where the tangent line from the left does not match the
tangent line from the right.) This means that f is not differentiable at 1. So
f 0 (1) does not exist.
The point x = 1 defines two intervals: (−∞, 1) and (1, ∞).
On the interval (∞, 1), the graph of f is a straight line. This means that the
tangent line at every (x, f (x)), x < 1, coincides with the graph of f . Using the
points (−2, 0) and (0, 2) on the graph of f , the slope is
2−0 2 2
m= = = =1
0 − (−2) 0+2 2
Similarly, on the interval (1, ∞), the derivative is constant. Using the points
(2, 1) and (3, −1) on the graph of f , we find
−1 − 1 −2
m= = = −2
3−2 1
So f 0 (x) = −2 for every x > 1.
y
4
2
f0
f
−2 2 4 x
f0
−2
y
4
−6 −4 −2 2 x
First, we observe from the graph of f that the tangent line is horizontal when
x = −1. So f 0 (−1) = 0. We also observe that f is not differentiable at −2 and
0 as the points (−2, 1) and (0, 1) are sharp points. So f 0 (−2) and f 0 (0) do not
exist.
3.8. Graphical Differentiation 175
The points −2, −1, and 0 determine four intervals: (−∞, −2), (−2, −1), (−1, 0),
and (0, ∞).
On the interval (−∞, −2), the graph of f is a straight line. So f 0 is constant for
x < −2. Using the points (−5, 0) and (−2, 1), we find
1−0 1 1
m= = =
−2 − (−5) −2 + 5 3
On the interval (−2, −1), the graph is not a straight line. So we need to estimate
the derivative at some points. Choose for example x = −1.5. Drawing the
tangent line at the point where x = −1.5, choosing two points on the tangent
line, we get the estimate f 0 (−1.5) ≈ 2.
On the interval (0, ∞), the graph of f is a straight line. This means that f 0 is
constant for x > 0. Using the points (0, 1) and (2, 0), we get
0−1 −1
m= =
2−0 2
Putting all the above information together, we get the graph of f 0 as shown in
the following figure.
y
4
2
0
f
−6 −4 −2 2 4 x
f
−2
−4
Solution.
176 Chapter 3. Limits and Derivatives
y
3
−5 5 x
First, observe that the tangent line at 0 is a vertical line. Since the slope of a
vertical line is undefined, it follows that f is not differentiable at 0. So f 0 (0)
does not exist (DNE).
On the intervals (∞, 0) and (0, ∞), we need to estimate the derivative at some
points. Choose for example the points −5, −1, 1, 5. Estimating the derivative at
these points, we get the following table.
x −5 −1 0 1 5
0
f (x) ≈ −0.114 −0.33 DNE 0.33 0.114
Plotting these points, and noticing from the graph of f that the slope of the
tangent line at (x, f (x)) approaches ∞ as x approaches 0, we get the graph of
the derivative.
f
2
f0
−5 5 x
−2
y
4
−2 2 4 6 x
(a) Solution. From the graph, the tangent line at x = 0 has a negative slope. This
means that f 0 (0) < 0. On the other hand, f 0 (2) = 0 since the tangent line to
the graph of f at 2 is horizontal. So f 0 (2) is bigger than f 0 (0).
(b) Solution. From the graph the slope of the tangent line at x = 1.5 is positive,
while the slope at x = 4 is 0. So f 0 (1.5) is bigger than f 0 (4).
(c) Solution. By definition, the instantaneous rate of change of f when x = 0.5 is
f 0 (0.5). But f 0 (0.5) is the slope the line through the points (−1, 3) and (0, 2),
that is,
2−3 −1 −1
f 0 (0.5) = = = = −1
0 − (−1) 0+1 1
So the instantaneous rate of change of f when x = 0.5 is −1.
(d) Solution.
For x < 1, the graph of f is a straight line. And according to the answer
from part (c), f 0 (x) = −1 for all x < 1.
For x > 3, the graph of f is a horizontal line. This means that f 0 (x) = 0
for every x > 3
On the interval (1, 3), we choose the points 1.5, 2, 2.5, and the estimates of
the derivative at these points are given in the following table.
x 1.5 2 2.5
0
f (x) ≈ 0.75 0 0.75
Using the facts that f (x) = −1 for x < 1, f 0 (x) = 0 for x > 3, and
0
the points (1.5, 0.75), (2, 0), (2.5, 0.75), we get the graph of the derivative as
shown in the following figure.
y
4
f
f0
−2 2 4 6 x
178 Chapter 3. Limits and Derivatives
−4 −2 2 4x
−2
−4
(a) Solution. From the graph of f , the slope of the tangent line at x = −0.5 is
bigger than the slope of the tangent line at x = 2. So f 0 (−0.5) > f 0 (2).
(b) Solution. Estimating the derivative at the points −3, −1, −0.5, 0.5, 1, 3, we get
the following table.
x −3 −1 −0.5 0.5 1 3
0
f (x) ≈ 0.11 1 4 4 1 0.11
2
f0
−4 −2 2 f 4x
−2
−4
Solution.
3.8. Graphical Differentiation 179
(b) 10 (a)
−2 2 4 6x
−5
We will use the fact if the tangent line to a graph at x = a is horizontal, then the
graph of the derivative crosses the x-axis at x = a. So we will look at the points
where the tangent line is horizontal.
Consider the graph (a). The tangent line is horizontal at the points where x = −1
or x = 1 or x = 4. Since the graph (b) crosses the x-axis exactly at the same points
(−1, 1, and 4), it follows that (b) is the derivative of (a). So (a) is the graph of f
and (b) is the graph of f 0 .
y
(a) 5 (b)
−4 −2 2 x
−5
−10
−15
From the graph of (a), we see that the tangent line is horizontal at three points
including the point where x = −1. Since (b) does not cross the x-axis at x = −1, it
follows that (b) cannot be the derivative of (a). So (a) is the derivative of (b). This
means that (b) is the graph of f and (a) is the graph of f 0 .
From the graph (a), we observe that the tangent line is horizontal exactly at one
point (x = 0). So the graph of the derivative of (a) crosses the x-axis exactly at
one point as well. Only (iv) has that property. Thus, (iv) is the derivative
of (a).
The graph (b) has exactly two points where the tangent line is horizontal. So
the graph of its derivative intersects the x-axis exactly at two points. Only (i)
has that property. This means that (i) is the derivative of (b).
The graph (c) is not differentiable at 0 (because it is a sharp point). This means
that the graph of its derivative is not defined at 0. Only (ii) has that property.
So (ii) is the derivative of (c).
Lastly, (iii) is the derivative of (d).
The graphs in (a) and (c) look similar. They have a horizontal tangent line at
0. This means that the graphs of their derivatives cross the x-axis at the origin.
Only (ii) and (iv) have that property. But (c) is not differentiable at two points,
while (a) is differentiable everywhere. So the derivative of (c) is undefined at
two points and the derivative of (a) is defined everywhere. This implies that
(iv) is the derivative of (c) and (ii) is the derivative of (a).
Now, the derivative of (b) is either (i) or (iii). From the graph of (b), observe
that the tangent lines have a negative slope for x < 0 and a positive slope for
x > 0. This means that the graph of the derivative of (b) is below the x-axis for
x < 0, and above the x-axis for x > 0. This corresponds to (iii). So (iii) is the
derivative of (b). Therefore, (i) is the derivative of (d).
(a) Solution. First, remember that the rate of change is the derivative and the
derivative is the slope of the tangent line. From the graph of P , the tangent
lines have a positive slope for 0 ≤ x < 3. So the rate of change of profit is
positive on the interval [0, 3). (Note that 3 is not included because P 0 (3) = 0.)
(b) Solution. From the graph of P , we see that the tangent line is horizontal at
3. This means that P 0 (3) = 0. So the rate of change of profit at the level of
production of 300 units is $0/unit.
(c) Solution. Estimating the derivative of P (or the marginal profit) at the points
1, 2, 3, 4, we get the following table.
x 1 2 3 4
0
P (x) ≈ 1.33 0.67 0 −0.67
Connecting the points (1, 1.33), (2, 0.67), (3, 0), (4, −0.67), we get the graph of
the marginal profit as shown in the following figure.
3.8. Graphical Differentiation 181
2
Profit
1
1 2 3 4 x
−1 Marginal profit
Differentiation Rules
(a) Solution.
Constant Rule
The derivative of a constant is 0. That is, if k is a constant, then
d
[k] = 0.
dx
Power Rule
For any real number n,
d n
[x ] = nxn−1 .
dx
(f ) Solution.
d d
[ku(x)] = k [u(x)].
dx dx
In words, this says that the derivative of a constant times a function is
the constant times the derivative of the function.
d
f 0 (x) = [3x−2 ]
dx
d
= 3 [x−2 ] Constant Multiple Rule
dx
= 3 −2x−2−1
Power Rule
6 1
= 3 −2x−3 = −6x−3 = − 3 x−m =
x xm
(g) Solution.
d x2
0 d 1 2 a 1
f (x) = = x = a
dx 3 dx 3 b b
1 d 2
= x Constant Multiple Rule
3 dx
1
= (2x2−1 ) Power Rule
3
1 2
= (2x1 ) = x
3 3
(h) Solution.
0 d 1 4 1 d h 4i
f (t) = t =3 t3 Constant Multiple Rule
dt 2 2 dt
1 4 4 −1
= t 3 Power Rule
2 3
4 1 a a−b
= t3 −1=
6 b b
2 1
= t3
3
4.1. Basic Rules of Differentiation 185
(i) Solution.
d h√ 3 i d h 3i √ m
f 0 (x) = x = x2 xm = x 2
dx dx
3 3
= x 2 −1 Power Rule
2
3 1 3√ a a−b 1 √
= x2 = x −1= and x 2 = x
2 2 b b
(j)
0 d 2 d −3 1
f (x) = 3
= 2x = x−n
dx x dx xn
d −3
=2 [x ] = 2(−3)x−3−1 Constant Multiple Rule and Power Rule
dx
−6
= −6x−4 = 4
x
(k) Solution.
√
0 d 1 d 1 1
f (x) = √ = n
x = xn
dx 3 x dx x 13
d h −1 i 1
= x 3 = x−n
dx xn
1 1
= − x− 3 −1 Power Rule
3
1 4 −1 a a−b
= − x− 3 = 4 −1=
3 3x 3 b b
(l) Solution.
d hp i d h 5i √ m
f 0 (p) = 3
p5 = p3 n
pm = p n
dp dp
5 5 5 2 a a−b
= p 3 −1 = p 3 Power Rule and −1=
3 3 b b
(m) Solution.
√
√
0 d 8
x d 1 1 n
1
f (x) = = x8 x = xn
dx 2 dx 2
1 d h 1i 1 1 1 −1
= x8 = x8 Constant Multiple Rule and Power Rule
2 dx 2 8
1 −7 1 1 1 a a−b 1
= x 8 = 7 = 7 −1= and x−n = n
16 16 x 8 16x 8 b b x
186 Chapter 4. Differentiation Rules
(n) Solution.
d e2
0 2 d 1
f (q) = =e Constant Multiple Rule
dq q dq q
d −1 1
= e2 q = q −1
dq q
= e2 (−1)q −1−1 = e2 (−q −2 )
Power Rule
e2
1 1
2
=e − 2 =− 2 q −n =
q q qn
(a) Solution.
d
f 0 (x) = [x + 1]
dx
d d
= [x] + [1] Sum Rule
dx dx
=1+0=1
(b) Solution.
d
f 0 (x) = [3x2 + 11x]
dx
d d
= [3x2 ] + [11x] Sum Rule
dx dx
d d
= 3 [x2 ] + 11 [x] Constant Multiple Rule
dx dx
d n
= 3(2x) + 11(1) = 6x + 11 [x ] = nxn−1
dx
(c) Solution.
4.1. Basic Rules of Differentiation 187
d d d
f 0 (x) = [x − 5] = [x] − [5] Difference Rule
dx dx dx
=1−0=1
(d) Solution.
0 d 2 2z
f (z) = z −
dz 5
d 2 d 2z
= z − Difference Rule
dz dz 5
2 d
= 2z − [z] Power Rule and Constant Multiple Rule
5 dz
2 2
= 2z − (1) = 2z −
5 5
(e) Solution.
√
√
0 d t− t d 1
f (t) = = t− t
dt 2 dt 2
√i 1 d d h√ i
1d h Constant Multiple Rule and
= t− t = [t] − t
2 dt 2 dt dt Difference Rule
√
1 d h 1i 1
= 1− t2 t = t2
2 dt
1 1 1 −1 1 1 −1
= 1− t 2 = 1− t 2 Power Rule and ab − 1 = a−b
b
2 2 2 2
1 1 1 1
= 1− 1 t−n =
2 2 t2 tn
√
1 1 2 t−1 b a b ac − b
= 1− √ = √ a− = − =
2 2 t 4 t c 1 c c
188 Chapter 4. Differentiation Rules
(f ) Solution.
d
f 0 (x) = −3x4 − 2x3 + x2 − 1
dx
d d 3 d 2 d Sum and Difference
−3x4 −
= 2x + x − [1]
dx dx dx dx Rules
d d d d
= −3 [x4 ] − 2 [x3 ] + [x2 ] − [1] Constant Multiple Rule
dx dx dx dx
Power Rule and Con-
= −3(4x3 ) − 2(3x2 ) + 2x − 0
stant Rule
= −12x3 − 6x2 + 2x
(g) Solution.
d 2x3 − 3x2
0 d 1 3 2
f (x) = = 2x − 3x
dx 4 dx 4
1 d 3
2x − 3x2
= Constant Multiple Rule
4 dx
1 d 3 d 2
= [2x ] − [3x ] Difference Rule
4 dx dx
1 Constant Multiple Rule and
2(3x2 ) − 3(2x)
=
4 Power Rule
1 3 3
= (6x2 − 6x) = x2 − x
4 2 2
(h) Solution.
d h 5 2
i d h 5i d h 2i
f 0 (x) = x3 − x3 = x3 − x3 Difference Rule
dx dx dx
5 5 2 2
= x 3 −1 − x 3 −1 Power Rule
3 3
5 2 2 1 5 2 2 a a−b
= x 3 − x− 3 = x 3 − 1 −1=
3 3 3 3x 3 b b
(i) Solution.
0 d 2 1 d 2 d 1
f (x) = x − = [x ] − Difference Rule
dx x dx dx x
d
1
1 Power Rule and dx =
= 2x − − 2 x
x d
dx
[x−1 ] = −x−2 = − x12
1
= 2x +
x2
4.1. Basic Rules of Differentiation 189
(j) Solution.
d
f 0 (x) = [1.4x5 − 2.5x2 + 3.8]
dx
d d d
= [1.4x5 ] − [2.5x2 ] + [3.8] Sum and Difference Rules
dx dx dx
d d d
= 1.4 [x5 ] − 2.5 [x2 ] + [3.8] Constant Multiple Rule
dx dx dx
= 1.4(5x4 ) − 2.5(2x) + 0 = 7x4 − 5x Power Rule and Constant Rule
(k) Solution.
√
0 d x d x 1
f (x) = √ = x = x2
dx x dx x 12
d h 1− 1 i xm
= x 2 n
= xm−n
dx x
d h 1 i 1 −1 1
= x2 = x 2 = √ Power Rule
dx 2 2 x
(l) Solution.
√ √
0 d x+x d x x a+b a b
f (x) = = + 2 = +
dx x2 dx x2 x c c c
" 1 #
d x2 x d h 1 −2 1−2
i √ 1 xm
= + = x 2 + x x = x 2 and n = xm−n
dx x2 x2 dx x
d h −3 −1
i d h −3 i d −1
= x +x
2 = x 2 + x Sum Rule
dx dx dx
3 3 3 5
= − x− 2 −1 + (−1)x−1−1 = − x− 2 − x−2 Power Rule
2 2
3 1 1
=− 5 − 2 x−n =
2x 2 x xn
So the equation of the tangent line at P (1, 3) is y − 3 = m(x − 1), that is,
y − 3 = 4(x − 1). This equation is equivalent to y − 3 = 4x − 4. Adding 3 to
both sides, we get y = 4x − 1.
The tangent line is horizontal when the derivative is 0. And the derivative is
dy d 3 d 3 d d
= [x − 3x + 1] = [x ] − 3 [x] + [1]
dx dx dx dx dx
2
= 3x − 3.
Set dy
dx
= 0. Then
3x2 − 3 = 0
3(x2 − 1) = 0 Factor
3(x − 1)(x + 1) = 0 a2 − b2 = (a − b)(a + b)
x = 1 or x = −1
Thus, the points where the tangent line to the curve y = x3 − 3x + 1 is horizontal are
(−1, 3) and (1, −1).
(a) Solution.
Let C(x) be the cost of producing x units of a certain product, and let
R(x) be the revenue function. Then the profit function, P (x), is given by
P (x) = R(x) − C(x).
d
Marginal Cost = C 0 (x) = [C(x)]
dx
4.1. Basic Rules of Differentiation 191
If x units have been produced, the cost of producing one more unit
is C(x + 1) − C(x). That is,
The figure below shows that the marginal cost C 0 (x) is close to C(x + 1) −
C(x). When x is very large, C 0 (x) is very close to C(x + 1) − C(x).
y
y = C(x)
C(x + 1)
C(x + 1) − C(x)
C 0 (x)
P
C(x)
1
x x+1 x
192 Chapter 4. Differentiation Rules
Similarly, if R(x) and P (x) represent the revenue and profit functions
respectively,
From part (a) the marginal cost function is C 0 (x) = 6x + 75. When x = 4,
This means that the approximate cost of producing the 5th unit is $99.
(c) Solution. The marginal cost for the production level x = 35 units is
This means that the approximate cost of producing the 36th unit is
$285.
(a) Solution.
We first need to find the revenue function R(q). Since the demand function
is p = −0.01q + 60 and the price per unit is p, we have that
d d d
R0 (q) = [−0.01q 2 + 60q] = −0.01 [q 2 ] + 60 [q]
dq dq dq
= −0.01(2q) + 60(1) = −0.02q + 60
(b) Solution.
First we need to find the profit function P (q).
This means that the approximate profit to produce the 401st unit
is $51.
(ii) The marginal profit for the production level q = 935 is
One can interpret this as follows. After 935 units have been produced,
the profit to produce one more unit will be approximately $18.9.
(c) Solution. The marginal profit is 0 when −0.06q + 75 = 0. Subtracting 75
from both sides, we get −0.06q = −75. Dividing both sides by −0.06, we get
q = 1250.
(d) Solution. When the marginal profit is 0, q = 1250, and the profit is
In words, this says that the derivative of the product of two functions is
the derivative of the first function times the second plus the first function
194 Chapter 4. Differentiation Rules
d
f 0 (x) = [(5x + 7)(6x2 )]
dx
d d
= [5x + 7](6x2 ) + (5x + 7) [6x2 ] Product Rule
dx dx
= (5 + 0)(6x2 ) + (5x + 7)12x
= 30x2 + 60x2 + 84x = 90x2 + 84x
Alternate Solution
One can find f 0 (x) without using the product rule:
d
f 0 (x) = [(5x + 7)(6x2 )]
dx
d
= [30x3 + 42x2 ] Distribute
dx
d d
= [30x3 ] + [42x2 ] = 90x2 + 84x
dx dx
Note. In Questions (b)–(e) below, where the given function is a product
of two functions, one can find the derivative in two different ways: (1) by
distributing first; (2) by using the product rule. We will use the product
rule. In next sections, we will see products of functions where the product
rule is essential. Examples of such functions include
(b) Solution.
d
f (x) = [(3x + 4)(x − 5)]
dx
d d
= [3x + 4](x − 5) + (3x + 4) [x − 5] Product Rule
dx dx
= (3 + 0)(x − 5) + (3x + 4)(1 − 0)
= 3x − 15 + 3x + 4 = 6x − 11 Distribute
4.2. The Product and Quotient Rules 195
(c) Solution.
d
f 0 (x) = [(5x2 − 2)(x3 + 3x)]
dx
d 2 d 3
5x − 2 (x3 + 3x) + (5x2 − 2)
= x + 3x Product Rule
dx dx
= 10x(x3 + 3x) + (5x2 − 2)(3x2 + 3)
= 10x4 + 30x2 + 15x4 + 15x2 − 6x2 − 6 Distribute
= 25x4 + 39x2 − 6
(d) Solution.
d
f 0 (x) = [(x3 + 1)(2x2 − 4x − 1)]
dx
= 3x2 (2x2 − 4x − 1) + (x3 + 1)(4x − 4) Product Rule
= (6x4 − 12x3 − 3x2 ) + (4x4 − 4x3 + 4x − 4) Distribute
= 10x4 − 16x3 − 3x2 + 4x − 4
(e) Solution.
d √
f 0 (x) = [(2 + x)(x2 − 3x)]
dx
d √ √ d
= [2 + x](x2 − 3x) + (2 + x) [x2 − 3x] Product Rule
dx dx
1 √ d √ 1
= √ (x2 − 3x) + (2 + x)(2x − 3) x = √
2 x dx 2 x
√ √
x2 − 3x + 2 x(2 + x)(2x − 3) a a + bc
= √ +c=
2 x b b
√ √ √
x2 − 3x + 2 x(4x − 6 + 2x x − 3 x)
= √ Distribute
2 x
√ √
x2 − 3x + 8x x − 12 x + 4x2 − 6x
= √ Distribute
2 x
√ √
5x2 − 9x + 8x x − 12 x
= √
2 x
(a) Solution.
196 Chapter 4. Differentiation Rules
In words, this says that the derivative of a quotient is the derivative of the
numerator times the denominator minus the numerator times the deriva-
tive of the denominator, all divided by the square of the denominator:
d d
[Top] × Bottom − Top × dx
d Top dx
[Bottom]
=
dx Bottom (Bottom)2
0 d 5x + 1
f (x) =
dx 5x − 1
d d
dx
[5x + 1](5x − 1) − (5x + 1) dx [5x − 1]
= 2
Quotient Rule
(5x − 1)
5(5x − 1) + (5x + 1)(5)
=
(5x − 1)2
25x − 5 − (25x + 5)
= Distribute
(5x − 1)2
25x − 5 − 25x − 5 −10
= 2
=
(5x − 1) (5x − 1)2
4.2. The Product and Quotient Rules 197
(b) Solution.
0 d 1 + 2x
f (x) =
dx 3 − 4x
d d
dx
[1 + 2x](3 − 4x) − (1 + 2x) dx [3 − 4x]
= 2
Quotient Rule
(3 − 4x)
2(3 − 4x) − (1 + 2x)(−4) 6 − 8x − (−4 − 8x)
= =
(3 − 4x)2 (3 − 4x)2
6 − 8x + 4 + 8x 10
= 2
=
(3 − 4x) (3 − 4x)2
(c) Solution.
d x2 + 1
f 0 (x) = [ ]
dx x3 − 1
d d
dx
[x2 + 1](x3 − 1) − (x2 + 1) dx [x3 − 1]
= Quotient Rule
(x3 − 1)2
2x(x3 − 1) − (x2 + 1)(3x2 )
=
(x3 − 1)2
2x4 − 2x − (3x4 + 3x2 )
=
(x3 − 1)2
2x4 − 2x − 3x4 − 3x2 −x4 − 2x − 3x2
= =
(x3 − 1)2 (x3 − 1)2
(d) Solution.
x3 + 3x
0 d
f (x) =
dx x2 − 4x + 3
d d
dx
[x3 + 3x](x2 − 4x + 3) − (x3 + 3x) dx [x2 − 4x + 3]
= Quotient Rule
(x2 − 4x + 3)2
(3x2 + 3)(x2 − 4x + 3) − (x3 + 3x)(2x − 4)
=
(x2 − 4x + 3)2
(3x4 − 12x3 + 9x2 + 3x2 − 12x + 9) − (2x4 − 4x3 + 6x2 − 12x)
=
(x2 − 4x + 3)2
3x4 − 12x3 + 9x2 + 3x2 − 12x + 9 − 2x4 + 4x3 − 6x2 + 12x
=
(x2 − 4x + 3)2
x4 − 8x3 + 6x2 + 9
=
(x2 − 4x + 3)2
198 Chapter 4. Differentiation Rules
(e) Solution.
0 d 1
f (x) =
dx x3 + 2x2 − 1
d d
dx
[1](x3 + 2x2 − 1) − (1) dx [x3 + 2x2 − 1]
= Quotient Rule
(x3 + 2x2 − 1)2
0(x3 + 2x2 − 1) − (3x2 + 4x)
=
(x3 + 2x2 − 1)2
0 − 3x2 − 4x −3x2 − 4x
= 3 = 3
(x + 2x2 − 1)2 (x + 2x2 − 1)2
(f ) Solution.
√
0 d x
f (x)
dx 3 + x
d √ √ d
dx
[ x](3 + x) − x dx [3 + x]
= 2
Quotient Rule
(3 + x)
1 √ 3+x √
√
2 x
(3 + x) − x(1) √ −
2 x
x
= =
(3 + x)2 (3 + x)2
(3+x)−2x
√
2 x 3 + x − 2x 3−x
= = √ = √
(3 + x)2 2 x(3 + x) 2 2 x(3 + x)2
(g) Solution.
d 2x5 + x4 − 6x
0
f (x) =
dx x
d d
dx
[2x5
+ x4 − 6x]x − (2x5 + x4 − 6x) dx [x]
= 2
Quotient Rule
x
(10x + 4x − 6)x − (2x5 + x4 − 6x)(1)
4 3
=
x2
10x5 + 4x4 − 6x − 2x5 − x4 + 6x
=
x2
8x5 + 3x4 x2 (8x3 + 3x2 )
= =
x2 x2
Simplify (this holds
= 8x3 + 3x2
for all x 6= 0)
Alternate Solution
If we first simplify the function, then we can find f 0 (x) without using the
4.2. The Product and Quotient Rules 199
Quotient Rule.
d 2x5 + x4 − 6x d 2x5 x4 6x
0
f (x) = = + −
dx x dx x x x
d 4
2x + x3 − 6
= Simplify (this holds for all x 6= 0)
dx
d d d
= [2x4 ] + [x3 ] − [6] = 8x3 + 3x2
dx dx dx
Note. This example shows that sometimes it is much easier to simplify
the original function first before taking its derivative.
(h) Solution. There are many ways of calculating f 0 (x): we can use the quotient
rule or simplify the function first and use the power rule or use the product rule.
We will simplify the function first because it is easier.
x2 + 4x + 3 x2 4x 3
f (x) = √ =√ +√ +√
x x x x
x2 x 3 3 1 1 √ 1 xm
= 1 +4 1 + 1 = x 2 + 4x 2 + 3x− 2 . x = x 2 and = xm−n
x2 x2 x2 xn
Now, by using the sum rule, the constant multiple rule, and the power rule, we
get
0 3 1 1 −1 1 −3
f (x) = x + 4
2 x 2 +3 − x 2
2 2 2
3 1 1 3 3 3 1 2 3
= x 2 + 2x− 2 − x− 2 = x 2 + 1 − 3
2 2 2 x2 2x 2
(i) Solution.
d
x + x1 − x dx d
x + x1
0 dx
[x]
f (x) = Quotient Rule
(x + x1 )2
x + x1 − x 1 − x12
d 1 1
= =− 2
(x + x1 )2 dx x x
x + x1 − x + x
x2
1
x
+ xx2
= =
(x + x1 )2 (x + x1 )2
2x
x2 2x 2
= 1 2 = 1 2 = 2 Simplify
(x + x
) x2 (x + x) x x + x1
(j) Solution. Using the quotient rule first, and then the product rule we get
d (6x + 1)(3x2 − 4)
0
f (x) =
dx 7x − 2
200 Chapter 4. Differentiation Rules
d d
dx
+ 1)(3x2 − 4)](7x − 2) − (6x + 1)(3x2 − 4) dx
[(6x [7x − 2]
=
(7x − 2)2
d d d
dx
[6x + 1](3x2 − 4) + (6x + 1) dx [3x2 − 4] − (6x + 1)(3x2 − 4) dx [7x − 2]
=
(7x − 2)2
(6(3x2 − 4) + (6x + 1)(6x)) (7x − 2) − (6x + 1)(3x2 − 4)(7)
=
(7x − 2)2
(18x2 − 24 + 36x2 + 6x) (7x − 2) − (18x3 − 24x + 3x2 − 4)(7)
=
(7x − 2)2
(54x2 + 6x − 24)(7x − 2) − (126x3 − 168x + 21x2 − 28)
=
(7x − 2)2
378x3 − 108x2 + 42x2 − 12x − 168x + 48 − 126x3 + 168x − 21x2 + 28
=
(7x − 2)2
252x3 − 87x2 − 12x + 76
=
(7x − 2)2
Alternate Solution
(6x+1)(3x2 −4)
Distributing the numerator of f (x) = 7x−2
, we find
C(x)
C(x) = .
x
The marginal average cost is the derivative of the average cost func-
tion. That is,
0 d
Marginal average cost = C (x) = C(x)
dx
For C(x) = 4000 + 3x, the average cost function is C(x) = 4000+3x
x
.
4000+3(8)
(i) When x = 8, C(8) = 8
= 503.
4000+3(20)
(ii) When x = 20, the average cost is C(20) = 20
= 203.
(b) Solution. The marginal average cost function is
0 d d 4000 + 3x
C (x) = C(x) =
dx dx x
d d
dx
[4000+ 3x]x − (4000 + 3x) dx [x]
= 2
Quotient Rule
x
(0 + 3)x − (4000 + 3x)(1)
=
x2
3x − 4000 − 3x −4000
= 2
=
x x2
(c) Solution. Remembering that the rate of change of a function is the derivative
of that function, the rate of change of the average cost when x = 15 is
0 4000 4000
C (15) = − 2 = − ≈ −17.78
15 225
202 Chapter 4. Differentiation Rules
This means that at the level of production of 15 units, the average cost is de-
creasing at the rate of $17.78 per unit.
5. [Exercise on page 33]
(a) Solution. First, the average cost is
4x2 +100 a
C(x) 3x+2 4x2 + 100 b a
C(x) = = = =
x x (3x + 2)x c bc
4x2 + 100
= .
3x2 + 2x
The marginal average cost is
d 4x2 + 100
0 d
C (x) = [C(x)] =
dx dx 3x2 + 2x
d d
dx
[4x2 + 100](3x2 + 2x) − (4x2 + 100) dx [3x2 + 2x]
= Quotient Rule
(3x2 + 2x)2
(8x)(3x2 + 2x) − (4x2 + 100)(6x + 2)
=
(3x2 + 2x)2
24x3 + 16x2 − 24x3 − 8x2 − 600x − 200
=
(3x2 + 2x)2
8x2 − 600x − 200
=
(3x2 + 2x)2
(b) Solution.
Let R(x) and P (x) be the revenue and profit functions respectively.
R(x)
The average revenue per item, denoted R(x), is R(x) = x
.
R(x) 7x+200
First, the average revenue is R(x) = x
= x
. The marginal average
revenue is
0 d d 7x + 200
R (x) = R(x) =
dx dx x
d d
dx
[7x + 200]x − (7x + 200) dx [x] (7)x − (7x + 200)(1) Quotient
= 2
=
x x2 Rule
4.3. The Chain Rule 203
7x − 7x − 200 −200
= =
x2 x2
(c) Solution.
First, the profit function is
4x2 + 100
P (x) = R(x) − C(x) = 7x + 200 −
3x + 2
2
(7x + 200)(3x + 2) − (4x + 100)
=
3x + 2
21x2 + 14x + 600x + 400 − 4x2 − 100 17x2 + 614x + 300
= = .
3x + 2 3x + 2
The average profit is
P (x) 17x2 + 614x + 300 17x2 + 614x + 300
P (x) = = = .
x x(3x + 2) 3x2 + 2x
0
Now the marginal average profit, P (x), is the derivative of the average profit function.
d
Using the quotient rule and the fact that dx [17x2 + 614x + 300] = 34x + 614 and
d
dx
[3x2 + 2x] = 6x + 2, we get
Composition of Functions
Suppose f and g are functions. Assume that for all x in the domain of f ,
f (x) is in the domain of g. The composition of g and f , denoted g ◦ f , is
the function defined as
(f ◦ g)(x) = f (g(x)).
204 Chapter 4. Differentiation Rules
(c) Solution.
f (g(x)) = f (5x + 4) = (5x + 4)2 − 3(5x + 4)
= (5x)2 + 2(5x)(4) + 42 − 3(5x + 4) (a + b)2 = a2 + 2ab + b2
= 25x2 + 40x + 16 − 15x − 12 = 25x2 + 25x + 4
(a) Solution.
To write h(x) as the composition of two functions, the idea is to find the inner
and outer functions. For h(x) = (x2 + 3)5 , the inner function is g(x) = x2 + 3
and the outer function is f (x) = x5 . So h(x) = f (g(x)).
4.3. The Chain Rule 205
1
(b) Solution. Here h(x) = (5x + 4) 2 , and the inner function is g(x) = 5x + 4 and
1 √
the outer function is f (x) = x 2 = x. So h(x) = f (g(x)).
√
(c) Solution. Here h(x) = f (g(x)) where g(x) = 10 + 7x and f (x) = − 3 x.
(d) Solution. The function
1 3
h(x) = (x2 + x) 4 − 3(x2 + x) 4 + 11
1 3
can be written as h(x) = f (g(x)), where g(x) = x2 +x and f (x) = x 4 −3x 4 +11.
Alternate Solution
Rewriting h(x) as
1 1
h(x) = (x2 + x) 4 − 3[(x2 + x) 4 ]3 + 11,
1
we have h(x) = f (g(x)) where g(x) = (x2 + x) 4 and f (x) = x − 3x3 + 11.
(a) Solution.
For y = (2x)5 , the inner function is u = 2x and the outer function is y = f (u) =
u5 . Using the chain rule, we get
dy dy du
= ·
dx du dx
d 5 d
= [u ] [2x]
du dx
4
= (5u )(2)
= 10u4 = 10(2x)4 Replace u with 2x
Alternate Solution
Since y is of the form y = (g(x))n with g(x) = 2x, we can use (4.3.3):
dy d d
(2x)5 = 5(2x)5−1 [2x]
= Chain Rule
dx dx dx
d
= 5(2x)4 (2) = 10(2x)4 [2x] = 2
dx
(b) Solution. Since y is of the form y = (g(x))n with g(x) = −3x + 4, we can use
Equation 4.3.3:
dy d d
(−3x + 4)8 = 8(−3x + 4)7 [−3x + 4]
= Chain Rule
dx dx dx
d
= 8(−3x + 4)7 (−3) [−3x + 4] = −3
dx
= 8(−3)(−3x + 4)7 = −24(−3x + 4)7
(c) Solution.
dy d h 2 3
i 3 3 d
= (x − 1) = (x2 − 1) 2 −1 [x2 − 1]
2 Chain Rule
dx dx 2 dx
3 1 3 1
= (x2 − 1) 2 (2x) = (2x)(x2 − 1) 2
2 2
1
= 3x(x2 − 1) 2
(d) Solution.
dy d
(−x3 + 2x + 1)15
=
dx dx
4.3. The Chain Rule 207
d
= 15(−x3 + 2x + 1)14 [−x3 + 2x + 1] Chain Rule
dx
= 15(−x + 2x + 1) (−3x2 + 2)
3 14
(e) Solution.
dy d
3(5x − 1)7
=
dx dx
d
(5x − 1)7
=3 Constant Multiple Rule
dx
d
= 3(7)(5x − 1)6 [5x − 1] Chain Rule
dx
= 21(5x − 1) (5) = 105(5x − 1)6
6
(f ) Solution.
dy d
−2(5x6 − 2x)4
=
dx dx
d 6
(5x − 2x)4
= −2 Constant Multiple Rule
dx
d
= −2(4)(5x6 − 2x)3 [5x6 − 2x] Chain Rule
dx
= −8(5x6 − 2x)3 (30x5 − 2) = −8(30x5 − 2)(5x6 − 2x)3
(g) Solution.
dy d h√ 2 i d h 2 1
i √ 1
= x −x = (x − x) 2 X = X2
dx dx dx
1 1 d
= (x2 − x) 2 −1 [x2 − x] Chain Rule
2 dx
1 1 1 1 1
= (x2 − x)− 2 (2x − 1) = (2x − 1) 1 X −n =
2 2 2
(x − x) 2 Xn
1
(2x − 1) 2x − 1
= 2√ = √
x2 − x 2 x2 − x
Alternate Solution
p
Let y be a function of the form y = g(x). Using the chain rule, one can
easily show that
d
d hp i [g(x)]
g(x) = dxp . (4.3.4)
dx 2 g(x)
208 Chapter 4. Differentiation Rules
dy d h√ 2 i d
[x2 − x] 2x − 1
= x − x = dx√ = √
dx dx 2 x2 − x 2 x2 − x
(h) Solution.
" #
√
dy d 1 d 1 n 1
= √ = X = Xn
dx dx 3 x2 − 1 dx (x2 − 1) 13
d h 2 1
i 1
= (x − 1)− 3 = X −n
dx Xn
1 1 d
= − (x2 − 1)− 3 −1 [x2 − 1] Chain Rule
3 dx
1 4 d 2
= − (x2 − 1)− 3 (2x) [x − 1] = 2x
3 dx
2 4 −2x
= − x(x2 − 1)− 3 = 4
3 3(x2 − 1) 3
(a) Solution.
We first need to find the marginal revenue function.
d p
R0 (x) = [16 3 (x2 + x)2 ]
dx
d h 2 2
i
= 16 (x + x) 3 Constant Multiple Rule
dx
2 2 − 31
= 16 (x + x) (2x + 1) Chain Rule
3
32 1 32(2x + 1)
= (2x + 1) 1 = 1
3 (x2 + x) 3 3(x2 + x) 3
(i) When x = 100,
32(2(100) + 1) 32(201) 2144
R0 (100) = 1 = = √
1
3
≈ $99.19
3(1002 + 100) 3 3(10100) 3 10100
(c) Solution. The marginal average revenue is the derivative of the average revenue
R(x). That is,
d d
[R(x)]x − R(x) dx
0 d d R(x) dx
[x]
R (x) = [R(x)] = = 2
Quotient Rule
dx dx x x
xR0 (x) − R(x) d
= [x] = 1
x2 dx
32(2x+1) 2
x 1 − 16(x2 + x) 3
2
3(x +x) 3 32(2x + 1)
= R0 (x) = 1 by part (a)
x2 3(x2 + x) 3
32x(2x+1) 2
1 − 16(x2 + x) 3
3(x2 +x) 3
=
x2
32x(2x+1)−48(x2 +x)
3(x2 +x) 3
1
a a − bc
= −c= and am · an = am+n
x2 b b
a
32x(2x + 1) − 48(x2 + x) b a
= 1 =
3x2 (x2 + x) 3 c bc
64x2 + 32x − 48x2 − 48x 16x2 − 16x
= 1 = 1
3x2 (x2 + x) 3 3x2 (x2 + x) 3
16x(x − 1) 16(x − 1)
= 1 = 1
3x2 (x2 + x) 3 3x(x2 + x) 3
5. [Exercise on page 34]
(a) Solution.
First, we need to find the revenue function. The revenue is R = pq. To
express this only in terms of q, we need to express p in terms of q.
√
– Squaring the equation q = 16000 − 2.5p, we get q 2 = 16000 − 2.5p.
Subtracting 16000 from both sides, we get q 2 −16000 = −2.5p. Dividing
both sides by −2.5, we get
q 2 − 16000 q 2 − 16000
2 2
p= = 5 = (q − 16000)
−2.5 −2 −5
2q 2 − 32000 2
= = − q 2 + 6400.
−5 5
– The revenue function is
2 2 2
R = pq = − q + 6400 q = − q 3 + 6400q
5 5
Now, we find the profit function P in terms of q.
2
P (q) = R(q) − C(q) = − q 3 + 6400q − (1500q + 4000)
5
2 3 2
= − q + 6400q − 1500q − 4000 = − q 3 + 4900q − 4000
5 5
210 Chapter 4. Differentiation Rules
(a) Solution. For the function y = (−5x + 4)(x3 + 1)6 , we must use the product
rule before using the chain rule. (We first use the product rule because it applies
to the whole function.)
dy d
(−5x + 4)(x3 + 1)6
=
dx dx
d d 3
[−5x + 4] (x3 + 1)6 + (−5x + 4) (x + 1)6
= Product Rule
dx dx
d
= −5(x3 + 1)6 + (−5x + 4)6(x3 + 1)5 (3x2 ) dx
[−5x+4] = −5 and Chain
Rule
3 6 2 3 5
= −5(x + 1) + 18x (−5x + 4)(x + 1)
Factor out (x3 +1)5 , which is
= (x3 + 1)5 −5(x3 + 1) + 18x2 (−5x + 4)
the greatest common factor
= (x3 + 1)5 (−5x3 − 5 − 90x3 + 72x2 ) Distribute inside brackets
= (x3 + 1)5 (−95x3 + 72x2 − 5)
4.3. The Chain Rule 211
(b) Solution.
dy d h √ i
= x 2 − x2
dx dx
d √ 2
d h√ 2
i
= [x] 2 − x + x 2−x Product Rule
dx dx
! d
√ d 2
hp i
[2 − x ] [g(x)]
d
g(x) = dx √ by
= (1) 2 − x2 + x dx√ dx 2 g(x)
2 2 − x2 the Chain Rule
√
−2x
= 2 − x2 + x √
2 2 − x2
√ x2
= 2 − x2 − √
2 − x2
(2 − x2 ) − x2 2 − 2x2 a − cb = ac−b and
= √ =√ √ √ c
2 − x2 2 − x2 ( X)( X) = X
(c) Solution.
dy d 2
(x + 1)3 (x2 + 2)6
=
dx dx
d 2 d 2
(x + 1)3 (x2 + 2)6 + (x2 + 1)3 (x + 2)6
= Product Rule
dx dx
= 3(x2 + 1)(2x)(x2 + 2)6 + (x2 + 1)3 6(x2 + 2)5 (2x) Chain Rule
= 6x(x2 + 1)2 (x2 + 2)6 + 12x(x2 + 1)3 (x2 + 2)5
Factor out 6x(x2 +
= 6x(x + 1) (x + 2) x2 + 2 + 2(x2 + 1)
2 2 2 5
1)2 (x2 +2)5 , the great-
est common factor
= 6x(x2 + 1)2 (x2 + 2)5 (x2 + 2 + 2x2 + 2)
= 6x(x2 + 1)2 (x2 + 2)5 (3x2 + 4)
(d) Solution.
d d
dy dx
[(x + 1)5 ] (x5 + 1) − (x + 1)5 dx [x5 + 1]
= Quotient Rule
dx (x5 + 1)2
5(x + 1)4 (1)(x5 + 1) − (x + 1)5 (5x4 ) Chain Rule and
= d
(x5 + 1)2 dx
[x5 + 1] = 5x4
5(x + 1)4 [(x5 + 1) − (x + 1)(x4 )]
= Factor out 5(x + 1)4
(x5 + 1)2
5(x + 1)4 (x5 + 1 − x5 − x4 )
=
(x5 + 1)2
5(x + 1)4 (1 − x4 )
=
(x5 + 1)2
212 Chapter 4. Differentiation Rules
(e) Solution.
x2
dy d
= √
dx dx x3 + 1
d
√
3 + 1 − x2 d
2
√
[x ] x x 3+1
= dx √
dx
2 Quotient Rule
3
x +1
√ d 3
[x +1]
2x x3 + 1 − x2 dx √ d
hp i
[g(x)]
2 x3 +1 d
g(x) = √
dx
by
= √ 2 dx 2 g(x)
x3 + 1 the Chain Rule
√ 2
2x x3 + 1 − x2 2√3x x3 +1 d 3
= √ 2 [x + 1] = 3x2
x3 + 1 dx
√ 4
2x x3 + 1 − 2√3x x3 +1
√ 2
= X =X
x3 + 1
4x(x3 +1)−3x4
√
2 x3 +1 b ac − b
= a− =
x3 + 1 c c
a
4x4 + 4x − 3x4 b a
= √ =
2(x3 + 1) x3 + 1 c bc
x4 + 4x
= √
2(x3 + 1) x3 + 1
(f ) Solution.
d
x hp i d
[g(x)]
r
dy d x 2 d
g(x) = √
dx
by
= = dxpx +3 dx 2 g(x)
dx dx 2
x +3 2 x2x+3 the Chain Rule
d d
dx
[x](x2 +3)−x dx [x2 +3]
2
(x +3)2
= Quotient Rule
2 x2x+3
p
a
−x2 + 3 b a
= =
2(x2 + 3)2 x2x+3
p
c bc
s
−x2 + 3 x2 + 3
=
2(x2 + 3)2 x
(g) Solution.
" 5 # 4
x4 + 1
4
d x4 + 1
dy d x +1
= =5 Chain Rule
dx dx x2 + 1 x2 + 1 dx x2 + 1
4.3. The Chain Rule 213
4 !
d d
x4 + 1 [x4
+ 1](x2 + 1) − (x4 + 1) dx [x2 + 1]
dx
=5 Quotient Rule
x2 + 1 (x2 + 1)2
4 4 3 2
4x (x + 1) − (x4 + 1)2x
x +1
=5
x2 + 1 (x2 + 1)2
4 4 5
4x + 4x3 − 2x5 − 2x
x +1
=5
x2 + 1 (x2 + 1)2
4 4 5
2x + 4x3 − 2x
x +1
=5
x2 + 1 (x2 + 1)2
4
2x(x4 + 2x2 − 1) 10x(x4 + 2x2 − 1)(x4 + 1)4
4
x +1
=5 =
(x2 + 1)2 x2 + 1 (x2 + 1)6
7. [Exercise on page 34]
Solution.
We first need to find the derivative.
dy d 2√ 3
= [x x − 2]
dx dx
d 2√ 3 d √
= [x ] x − 2 + x2 [ x3 − 2] Product Rule
dx dx
! d
√ d 3
hp i
[x − 2] [g(x)]
d
g(x) = dx √ by
= 2x x3 − 2 + x2 dx√ dx 2 g(x)
2 x3 − 2 the Chain Rule
√ 3x2
2 d 3
3
= 2x x − 2 + x √ [x − 2] = 3x2
2 x3 − 2 dx
√ 3x4
= 2x x3 − 2 + √
2 x3 − 2
4x(x3 − 2) + 3x4 b ac + b √ √
= √ a+ = and X X =X
2 x3 − 2 c c
4x4 − 8x + 3x4 7x4 − 8x
= √ = √
2 x3 − 2 2 x3 − 2
The slope of the tangent line at (3, 45) is obtained by replacing x with 3 into dy
dx
:
7(3)4 − 8(3) 567 − 24 543
m= √ = √ = .
3
2 3 −2 2 25 10
The equation of the tangent line is y − 45 = 543 10
(x − 3). Multiplying out the
right hand side, we get y − 45 = 543
10
x − 1629
10
. Adding 45 to both sides, we get
543 1179
y= x− .
10 10
8. [Exercise on page 34]
Solution.
214 Chapter 4. Differentiation Rules
−2 2 2 1
y= 2 4
=− 4
=− =− .
((−2) + 28) (4 + 28) 1048576 524288
1
This gives the point −2, − 524288 .
* The y-coordinate for x = 2 is
2 2 2 1
y= = = = .
((2)2
+ 28) 4 (4 + 28) 4 1048576 524288
1
This gives the point 2, 524288 .
x
Hence, the tangent line to the curve y = (x2 +28)4
is horizontal at the points
1 1
−2, − 524288 and 2, 524288 .
The Derivative of ex
The derivative of the natural exponential function is ex . That is,
d x
[e ] = ex .
dx
d
f 0 (x) = [2ex ]
dx
d
= 2 [ex ] Constant Multiple Rule
dx
= 2ex
(b) Solution.
d √
f 0 (x) = [1 + 2x − ex ]
dx
d d h√ i d
= [1] + 2x − [ex ] Sum and Difference Rules
dx dx dx
d
d
hp i
[2x] [g(x)]
= 0 + √ − ex
dx
d
dx
g(x) = dx√ by
2 g(x)
2 2x the Chain Rule
2 1
= √ − ex = √ − ex
2 2x 2x
(c) Solution.
d 3 x d 3 x d
f 0 (x) = xe = [x ]e + x3 [ex ] Product Rule
dx dx dx
= 3x e + x e = (3x + x )ex
2 x 3 x 2 3
Factor out ex
(d) Solution.
d d
[x2 ](ex + 3) − x2 dx [ex + 3]
2
0 d x dx
f (x) = = Quotient Rule
dx ex + 3 (ex + 3)2
2x(ex + 3) − x2 (ex + 0) 2xex + 6x − x2 ex
= =
(ex + 3)2 (ex + 3)2
(a) Solution.
216 Chapter 4. Differentiation Rules
d g(x) d
e = eg(x) [g(x)].
dx dx
dy d 3x d
= [e ] = e3x [3x] Chain Rule
dx dx dx
d
= e3x (3) = 3e3x [3x] = 3
dx
(b) Solution.
dy d −5x d
= [e ] = e−5x [−5x] Chain Rule
dx dx dx
= e (−5) = −5e−5x
−5x
(c) Solution.
dy d h x2 +5x+1 i 2 d
= e = ex +5x+1 [x2 + 5x + 1] Chain Rule
dx dx dx
2 +5x+1 2 +5x+1
= ex (2x + 5) = (2x + 5)ex
(d) Solution.
dy d d
= [−e−x ] = − [e−x ] Constant Multiple Rule
dx dx dx
−x d
=− e [−x] Chain Rule
dx
= − e−x (−1) = −(−1)e−x = e−x
(e) Solution.
dy d d −1.5x
−4e−1.5x = −4
= e Constant Multiple Rule
dx dx dx
d
= −4e−1.5x [−1.5x] Chain Rule
dx
= −4e−1.5x (−1.5) = −4(−1.5)e−1.5x = 6e−1.5x
4.4. Derivatives of Exponential Functions 217
(f ) Solution.
dy d h −x2 +1 i d h −x2 +1 i
= 7e =7 e Constant Multiple Rule
dx dx dx
2 d
= 7e−x +1 [−x2 + 1] Chain Rule
dx
2 +1 2 +1
= 7e−x (−2x + 0) = 7(−2x)e−x
2 +1
= −14xe−x
(g) Solution.
dy d 2x d 2x d
e + e−2x = [e ] + [e−2x ]
= Sum Rule
dx dx dx dx
d d
= e2x [2x] + e−2x [−2x] Chain Rule
dx dx
= e (2) + e (−2) = 2e2x − 2e−2x
2x −2x
3
(h) Solution. To take the derivative of y = x2 ex , we must use the product rule
before using the chain rule. The reason we first use the product rule is because
it applies to the whole function, while the chain rule applies only to the term
3
ex .
dy d h x3 i
= xe
dx dx
d 3 d h x3 i
= [x]ex + x e Product Rule
dx dx
x3 x3 d 3
= (1)e + x e [x ] Chain Rule
dx
3 3 3 3
= ex + xex (3x2 ) = (1 + 3x3 )ex Factor out ex
(i) Solution.
dy d 2 −4x d 2 −4x d −4x
= xe = [x ]e + x2 e Product Rule
dx dx dx dx
−4x 2 −4x d
= 2xe +x e [−4x] Chain Rule
dx
= 2xe−4x + x2 e−4x (−4) = 2xe−4x − 4x2 e−4x
= (2x − 4x2 )e−4x Factor out e−4x
(j) Solution.
d
[e−2x ](x2 + 5) − e−2x dx
d
[x2 + 5]
−2x
dy d e dx
= = Quotient Rule
dx dx x2 + 5 (x2 + 5)2
e−2x dx
d
[−2x](x2 + 5) − e−2x dx
d
[x2 + 5]
= Chain Rule
(x2 + 5)2
218 Chapter 4. Differentiation Rules
(k) Solution.
dy d h x√2x+1 i
= e
dx dx
√ d h √ i
= ex 2x+1 x 2x + 1 Chain Rule
dx
d √
√
i
x 2x+1 d h 1
=e [x] 2x + 1 + x (2x + 1) 2 Product Rule
dx dx
√
√
x 2x+1 1 − 21
=e (1) 2x + 1 + x (2x + 1) (2) Chain Rule
2
!
√ √ x 1 1
= ex 2x+1 2x + 1 + (2) 1 X −n = n
2 (2x + 1) 2 X
√
√
x 2x+1 x
=e 2x + 1 + √
2x + 1
a + cb = ac+b
√
(2x + 1) + x and
= ex 2x+1 √ √ √ c
2x + 1 ( X)( X) = X
3x + 1 √
= √ ex 2x+1
2x + 1
(l) Solution.
" 2
#
dy d xex
=
dx dx x + e−x
h i
x2 2 d
d
dx
xe (x + e−x ) − xex dx [x + e−x ]
= Quotient Rule
(x + e−x )2
h i
x2 2 2
d d
(x + e−x ) − xex d d
[e−x ]
[x]e + x ex [x] +
dx dx dx dx Product and
=
(x + e−x )2 Sum Rules
x2 x2 2
e + x(2x)e (x + e−x ) − xex (1 + (−1)e−x )
= Chain Rule
(x + e−x )2
2 2 2 2 2 2
xex + ex e−x + 2x3 ex + 2x2 ex e−x − xex + xex e−x
=
(x + e−x )2
4.4. Derivatives of Exponential Functions 219
2
(e−x + 2x3 + 2x2 e−x + xe−x ) ex
=
(x + e−x )2
(m)
dy d
(1 + xe4x )5
=
dx dx
d
= 5(1 + xe4x )4 1 + xe4x
Chain Rule
dx
4x 4 d d 4x
= 5(1 + xe ) [1] + xe Sum Rule
dx dx
4x 4 d 4x d 4x
= 5(1 + xe ) 0 + [x]e + x [e ] Product Rule
dx dx
4x 4 4x 4x
= 5(1 + xe ) e + x 4e Chain Rule
= 5(1 + xe4x )4 (1 + 4x)e4x
(n) Solution.
dy d h x2 +1 √ i
= e 2x + 3
dx dx
d h x2 +1 i √ 2 d h√ i
= e 2x + 3 + ex +1 2x + 3 Product Rule
dx dx
x2 +1 d 2
√ x2 +1 d
h 1
i
=e [x + 1] 2x + 3 + e (2x + 3) 2 Chain Rule
dx dx
√
x2 +1 x2 +1 1 − 21 d
=e (2x) 2x + 3 + e (2x + 3) [2x + 3] Chain Rule
2 dx
!
x2 +1
√ x2 +1 1 1
= 2xe 2x + 3 + e (2)
2 (2x + 3) 21
2
x2 +1
√ ex +1
= 2xe 2x + 3 + √
2x + 3
2 2 2
2x(2x + 3)ex +1 + ex +1 (4x2 + 6x + 1)ex +1
= √ = √
2x + 3 2x + 3
(a) Solution.
e:
ax = ex ln a .
Using now the chain rule, we get
dy d x ln a d
= e = ex ln a [x ln a] = (ln a)ex ln a = (ln a)ax .
dx dx dx
So
d x
[a ] = (ln a)ax . (4.4.1)
dx
Likewise, if y is of the form ag(x) , then by using the change-of-base
theorem for exponentials and the chain rule, we get
d g(x) d
a = (ln a)ag(x) [g(x)]. (4.4.2)
dx dx
For y = 5x , we have
dy d x d x ln 5
= [5 ] = e Change the base
dx dx dx
d
= ex ln 5 [x ln 5] Chain Rule
dx
x ln 5
=e (ln 5) = (ln 5)ex ln 5 = (ln 5)5x
Alternate Solution
Using (4.4.1), we get
d x
[5 ] = (ln 5)5x .
dx
(b) Solution.
dy h x2 i d h x2 ln 3 i
= 3 = e Change the base
dx dx
2 d
= ex ln 3 [x2 ln 3] Chain Rule
dx
2 d 2 d
= ex ln 3
(2x ln 3) [x ln 3] = (ln 3) [x2 ] = (ln 3)(2x)
dx dx
2 2
= (2x ln 3)ex ln 3
= (2x ln 3)3x
Alternate Solution
Using (4.4.2), we get
d x2 2 d 2 2
[3 ] = (ln 3)3x [x2 ] = (ln 3)3x (2x) = (2x ln 3)3x .
dx dx
4.4. Derivatives of Exponential Functions 221
(c) Solution.
dy d h √ i
= 2·5 x
dx dx
d h √x i
=2 5 Constant Multiple Rule
dx
√ d √
x
= 2 (ln 5)5 [ x] Formula (4.4.2)
dx
√ 1 d √ 1
x
= 2 (ln 5)5 √ x = √
2 x dx 2 x
√
x
(ln 5)5
= √
x
The tangent line is horizontal where the derivative is 0. So we need to solve the
dy 2 2
equation dx = 0. Set (2x − 6)ex −6x = 0. Then 2x − 6 = 0 or ex −6x = 0.
– The equation 2x − 6 = 0 is equivalent to 2x = 6. Dividing both sides by 2,
we get x = 3.
2
– The equation ex −6x = 0 has no solution because the exponential of a number
is always positive (eX > 0 for every number X).
So the derivative is zero when x = 3.
2 −6x 2 −6(3)
Substituting this into ex , we get y = e3 = e9−18 = e−9 .
0 d h√ −x
i
C (x) = 600 − 500 · 1.2
dx
d
[600 − 500 · 1.2−x ] hp i d
[g(x)]
= √ dx d
dx
g(x) = dx√
2 600 − 500 · 1.2−x 2 g(x)
d
dx
[600] d
− 500 dx [1.2−x ] Difference Rule and Constant
= √
2 600 − 500 · 1.2−x Multiple Rule
222 Chapter 4. Differentiation Rules
0 − 500(ln 1.2)1.2−x dx
d
[−x] d g(x) d
= √ a = (ln a)ag(x) [g(x)]
2 600 − 500 · 1.2−x dx dx
−500(ln 1.2)1.2−x (−1) d
= √ [−x] = −1
2 600 − 500 · 1.2−x dx
250(ln 1.2) · 1.2−x
=√ Simplify
600 − 500 · 1.2−x
(i) When x = 0,
(a) Solution.
The Derivative of ln x
Let x be a positive number. By definition,
ln x = y if and only if ey = x.
d 1
[ln x] = . (4.5.1)
dx x
4.5. Derivatives of Logarithmic Functions 223
d
f 0 (x) = [5 ln x] Constant Multiple Rule
dx
1 d 1
=5 [ln x] = by (4.5.1)
x dx x
5
=
x
(b) Solution.
d 3 d 3 d
f 0 (x) =
x − ln x = [x ] − [ln x] Difference Rule
dx dx dx
1 d 1
= 3x2 − [ln x] = by (4.5.1)
x dx x
3
3x − 1 b ac − b
= a− =
x c c
(c) Solution.
d 2
f 0 (x) =
x ln x
dx
d 2 d
= [x ] ln x + x2 [ln x] Product Rule
dx dx
1 d 1
= 2x ln x + x2 [ln x] = by (4.5.1)
x dx x
= 2x ln x + x
(d) Solution.
0 d x
f (x) =
dx 1 + ln x
d d
dx
[x](1 + ln x) − x dx [1 + ln x]
= 2
Quotient Rule
(1 + ln x)
d d d
dx
+ ln x) − x dx
[x](1 [1] + dx
[ln x]
= 2
Sum Rule
(1 + ln x)
(1)(1 + ln x) − x 0 + x1
d 1
= [ln x] = by (4.5.1)
(1 + ln x)2 dx x
1 + ln x − 1 ln x
= 2
=
(1 + ln x) (1 + ln x)2
(a) Solution.
224 Chapter 4. Differentiation Rules
dy d
= [ln(3x)]
dx dx
d
[3x]
= dx By (4.5.2)
3x
3 1
= =
3x x
(b) Solution. Using (4.5.2), we get
d
dy d dx
[−5x + 4] −5
= [ln(−5x + 4)] = = .
dx dx −5x + 4 −5x + 4
(d) Solution.
d d
dy d [x− ex ] d [g(x)]
= [ln(x − ex )] = dx
[ln g(x)] = dx
dx dx x − ex dx g(x)
d d
[x]− dx [ex ] 1 − ex d x
= dx
= [e ] = ex
x − ex x − ex dx
(e) Solution.
dy d d
= [4 ln(−2x)] = 4 [ln(−2x)] Constant Multiple Rule
dx dx dx
d d
[−2x] d [g(x)]
= 4 dx [ln g(x)] = dx
−2x dx g(x)
−2 −8 4
=4 = =
−2x −2x x
4.5. Derivatives of Logarithmic Functions 225
(f ) Solution.
dy d d
= [ln(4x)] + [ln(4x)] Sum Rule
dx dx dx
d d d
[4x] [−4x] d [g(x)]
= dx + dx [ln g(x)] = dx
4x −4x dx g(x)
4 4 1 1 2
= − = + =
4x −4x x x x
(g) Solution.
dy d d d
= [x ln(x2 )] = [x] ln(x2 ) + x [ln(x2 )] Product Rule
dx dx dx dx
d d
2 dx
[x2 ] d dx
[g(x)]
= (1) ln(x ) + x [ln g(x)] =
x2 dx g(x)
2x 2x2
= ln(x2 ) + x 2
= ln(x 2
) + 2
= ln (x2 ) + 2
x x
Note. Do not use the property ln(xn ) = n ln x to simplify further! Indeed, the
function ln(x2 ) + 2 is valid for all numbers except 0 while the function 2 ln x + 2
is valid only for x > 0.
(h) Solution.
dy d 3 d
= [x ] ln(4x2 − 6x + 3) + x3 [ln(4x2 − 6x + 3)] Product Rule
dx dx dx
d d
[4x2 − 6x + 3] d [g(x)]
= 3x2 ln(4x2 − 6x + 3) + x3 dx 2 [ln g(x)] = dx
4x − 6x + 3 dx g(x)
8x − 6
= 3x2 ln(4x2 − 6x + 3) + x3 2
4x − 6x + 3
2 2
8x4 − 6x3
= 3x ln (4x − 6x + 3) +
4x2 − 6x + 3
(i) Solution.
dy d h x2 i
= e ln(3x)
dx dx
d x2 2 d
= [e ] ln(3x) + ex [ln(3x)] Product Rule
dx dx
d d
g(x)
d e = eg(x) dx [g(x)],
x2 d 2 [3x] dx
= e [x ] ln(3x) + ex dx
2
d
d
[g(x)]
dx 3x dx
[ln g(x)] = dx
g(x)
2 3
2
= ex (2x) ln(3x) + ex
3x
x2
x2 e 1 2
= 2xe ln(3x) + = 2x ln (3x) + ex
x x
226 Chapter 4. Differentiation Rules
(j) Solution.
dy d h √ i
= ln 6x + 7
dx dx
d
√ d
dx
[ 6x + 7] d dx
[g(x)]
= √ [ln g(x)] =
6x + 7 dx g(x)
√6
2 6x+7 d hp i g 0 (x)
=√ g(x) = p
6x + 7 dx 2 g(x)
3 3
=√ √ =
6x + 7 6x + 7 6x + 7
(k) Solution.
√
d x d
dy d h √ i [xe + 8] d [g(x)]
= ln(xe x + 8) = dx
√
x
[ln g(x)] = dx
dx dx xe + 8 dx g(x)
√
d d
dx
[xe x ] + dx [8]
= √ Sum Rule
xe x +8
√ √
d d
dx
[x]e x + x dx [e x ]
= √ Product Rule
xe x + 8
d √
√ √
x
e + xe x dx [ x] d g(x) d
= √ e = eg(x) [g(x)]
xe x + 8 dx dx
√ √
x 1
x √
e + xe 2 x d √ 1
= √
x
x = √
xe +8 dx 2 x
√ √ √
2 x+x
1 + 2√x x e x √
2 x
e x b ac + b
= √
x
= √ a+ =
+8 xe xe x + 8 c c
√ √
a
(x + 2 x)e x b a
= √ √ =
xe x + 8 2 x c bc
(l) Solution.
dy d h√ −x i d
[e−x + ln 2x] d hp i d
[g(x)]
= e + ln 2x = dx√ −x g(x) = dxp
dx dx 2 e + ln 2x dx 2 g(x)
d d
d
dx
[e−x ] + dx
d
[ln 2x] −e−x + 2x
2
dx
eg(x) = eg(x) dx [g(x)],
= √ = √ −x d
d
[g(x)]
2 e−x + ln 2x 2 e + ln 2x dx
[ln g(x)] = dx
g(x)
−x 1 −x
−e + 1 − xe x b ac + b ab a
= √ −x = √ a+ = , =
2 e + ln 2x 2x e−x + ln 2x c c c bc
(m) Solution.
xe−x
dy d
=
dx dx 1 + ln(5x)
4.5. Derivatives of Logarithmic Functions 227
d
dx
[xe−x ](1
+ ln(5x)) − xe−x dx d
[1 + ln(5x)]
= Quotient Rule
(1 + ln(5x))2
d
[x]e−x + x dx
d
[e−x ] (1 + ln(5x)) − xe−x dx d
dx
[1 + ln(5x)]
= Product Rule
(1 + ln(5x))2
(e−x + x(−e−x )) (1 + ln(5x)) − xe−x dx d d
[1] + dx [ln(5x)]
=
(1 + ln(5x))2
(a) Solution.
So
d 1
[loga x] = (4.5.3)
dx (ln a)x
(b) Solution.
dy d d
= [7 log8 x] = 7 [log8 x] Constant Multiple Rule
dx dx dx
228 Chapter 4. Differentiation Rules
1 d 1
=7 [loga x] =
(ln 8)x dx (ln a)x
7
=
(ln8)x
(c) Solution.
d ln(−2x3 + x2 )
dy d 3 2
ln X
= log2 (−2x + x ) = loga X =
dx dx dx ln 2 ln a
1 d
= [ln(−2x3 + x2 )] Constant Multiple Rule
ln 2 dx
d d
1 dx [−2x3 + x2 ] d dx
[g(x)]
= [ln g(x)] =
ln 2 −2x3 + x2 dx g(x)
1 −6x2 + 2x −6x2 + 2x
= =
ln 2 −2x3 + x2 (ln2)(−2x3 + x2 )
Alternate Solution
5
(d) Solution. For the function y = log(x2 − x) 2 , the base is a = 10. So
h 5
i
d 2
dy d h 5
i dx
(x − x) 2
d d
[g(x)]
= log(x2 − x) 2 = 5 log a g(x) = dx
dx dx (ln 10)(x2 − x) 2 dx (ln a)g(x)
4.5. Derivatives of Logarithmic Functions 229
5 3
d
2
(x2 − x) 2 dx [x2 − x]
= 5 Chain Rule
(ln 10)(x2 − x) 2
5 3
2
(x2 − x) 2 (2x − 1) 5(2x − 1)
= 5 = Simplify
(ln 10)(x2 − x) 2 2(ln10)(x2 − x)
(a) Solution.
d 1
[ln |x|] = .
dx x
Similarly, one has
d
d dx
[g(x)]
[ln |g(x)|] =
dx g(x)
d 1
[loga |x|] =
dx (ln a)x
d
d dx
[g(x)]
[loga |g(x)|] = .
dx (ln a)g(x)
dy
For y = ln |x|, dx
= x1 .
(b) Solution.
dy d
= [ln | ln x|]
dx dx
d d
dx
[ln x] d dx
[g(x)]
= [ln |g(x)|] =
ln x dx g(x)
1 a
x
1 d 1 b a
= = [ln x] = and =
ln x x ln x dx x c bc
230 Chapter 4. Differentiation Rules
(c) Solution.
d d
dy d hp i
dx
[ln |6 − x|] d hp i
dx
[g(x)]
= ln |6 − x| = p g(x) = p
dx dx 2 ln |6 − x| dx 2 g(x)
d
dx
[6−x] d
d [g(x)]
= p 6−x [ln |g(x)|] = dx
2 ln |6 − x| dx g(x)
−1 a
6−x −1 b a
= p = p =
2 ln |6 − x| 2(6 − x) ln|6 − x| c bc
(d) Solution.
dy d 2
x log5 |x3 + 1|
=
dx dx
d 2 d
[x ] log5 |x3 + 1| + x2 log5 |x3 + 1|
= Product Rule
dx dx
d d
[x3 + 1] d [g(x)]
= 2x log5 |x3 + 1| + x2 dx [loga |g(x)|] = dx
(ln 5)(x3 + 1) dx (ln a)g(x)
3x2
= 2x log5 |x3 + 1| + x2
(ln 5)(x3 + 1)
3x4
= 2xlog5 |x3 + 1| +
(ln5)(x3 + 1)
5. [page 35]
(a) Solution.
dy d 2x
= x
dx dx
d 2x ln x
= e AB = eB ln A
dx
d d g(x) d
= e2x ln x [2x ln x] e = eg(x) [g(x)]
dx dx dx
2x ln x d d
=e [2x] ln x + 2x [ln x] Product Rule
dx dx
1
= e2x ln x 2 ln x + 2x
x
= e2x ln x (2 ln x + 2)
= 2(1 + lnx)e2xlnx = 2(1 + lnx)x2x
4.5. Derivatives of Logarithmic Functions 231
f (x) = x2x
ln(f (x)) = ln(x2x ) = 2x ln x ln(xn ) = n ln x
ln(f (x)) = 2x ln x
d d
[ln(f (x))] = [2x ln x]
dx dx
d
f 0 (x) d d dx
[g(x)]
= [2x ln x] [ln g(x)] =
f (x) dx dx g(x)
f 0 (x) d d
= [2x] ln x + 2x [ln x] Product Rule
f (x) dx dx
f 0 (x)
1
= 2 ln x + 2x
f (x) x
f 0 (x)
= 2 ln x + 2
f (x)
f 0 (x)
Step 3. Solve for f 0 (x). From the equation f (x)
= 2 ln x + 2, it
follows that
(b) Solution.
dy d 2 d h 3x ln(x2 +1) i
(x + 1)3x = AB = eB ln A
= e
dx dx dx
2 d d g(x) d
= e3x ln(x +1) [3x ln(x2 + 1)] e = eg(x) [g(x)]
dx dx dx
2 d d
= e3x ln(x +1) [3x] ln(x2 + 1) + 3x [ln(x2 + 1)] Product Rule
dx dx
d
3x ln(x2 +1) 2 2x d dx
[g(x)]
=e 3 ln(x + 1) + 3x 2 [ln g(x)] =
x +1 dx g(x)
6x2
2
= e3xln(x +1) 3ln(x2 + 1) + 2
x +1
232 Chapter 4. Differentiation Rules
dy d h 2
i d h (x2 +7) ln |1+e2x | i
= |1 + e2x |x +7 = e
dx dx dx
2 2x d
= e(x +7) ln |1+e | (x2 + 7) ln |1 + e2x |
dx
(x2 +7) ln |1+e2x | d 2 2x 2 d 2x
=e [x + 7] ln |1 + e | + (x + 7) ln |1 + e |
dx dx
!
d 2x
2 2x [1 + e ]
= e(x +7) ln |1+e | (2x) ln |1 + e2x | + (x2 + 7) dx
1 + e2x
2e2x
(x2 +7) ln |1+e2x | 2x 2
=e (2x) ln |1 + e | + (x + 7)
1 + e2x
2(x2 + 7)e2x
(x2 +7)ln|1+e2x | 2x
=e 2xln|1 + e | +
1 + e2x
There are six basic trigonometric functions: sine (sin), cosine (cos), tan-
gent (tan), cotangent (cot), secant (sec), and cosecant (csc) functions.
The derivative of the sine function is the cosine function, that is,
d
[sin x] = cos x
dx
The derivative of the cosine function is the opposite of the sine func-
tion, that is,
d
[cos x] = − sin x
dx
The tangent function, denoted tan, is defined as tan x = sin x
cos x
. And
its derivative is given by
d 1
[tan x] = sec2 x =
dx cos2 x
The cotangent function, denoted cot, is defined as cot x = cos x
sin x
. This
1
is precisely the inverse of tangent, that is, cot x = tan x . And its
234 Chapter 4. Differentiation Rules
derivative is
d 1
[cot x] = − csc2 x = − 2
dx sin x
The secant function, denoted sec, is defined as the inverse of the
cosine function, that is, sec x = cos1 x . Its derivative is given by
d
[sec x] = sec x tan x
dx
d
[csc x] = − csc x cot x
dx
(b) Solution.
0 d sin x 3 d sin x d
f (x) = −x = − [x3 ] Difference Rule
dx 4 dx 4 dx
1 d Constant Multiple Rule,
= [sin x] − 3x2
4 dx Power Rule
1 d
= cos x − 3x2 [sin x] = cos x
4 dx
(c) Solution.
d 4 d 4 d
π + 3 tan x = [π ] + [3 tan x] Sum Rule
dx dx dx
d Constant Rule, Con-
= 0 + 3 [tan x]
dx stant Multiple Rule
d
= 3sec2 x [tan x] = sec2 x
dx
(d) Solution.
d
f 0 (x) = [3 sin x − 8 cos x + 2 tan x]
dx
4.6. Derivatives of Trigonometric Functions 235
d d d
= [3 sin x] − [8 cos x] + [2 tan x]
dx dx dx
d d d
= 3 [sin x] − 8 [cos x] + 2 [tan x]
dx dx dx
d
dx
[sin x] = cos x
2 d
= 3 cos x − 8(− sin x) + 2 sec x dx
[cos x] = − sin x
d
dx
[tan x] = sec2 x
= 3 cos x + 8 sin x + 2 sec2 x
(e) Solution.
d
f 0 (x) = [7 sec x − csc x + cot x]
dx
d d d
= 7 [sec x] − [csc x] + [cot x]
dx dx dx
d
dx
[sec x] = sec x tan x
2 d
= 7 sec x tan x − (− csc x cot x) + (− csc x) dx
[csc x] = − csc x cot x
d
dx
[cot x] = − csc2 x
= 7 secx tan x + cscx cot x − csc2 x
(f ) Solution.
d d d
f 0 (x) = [x cot x] = [x] cot x + x [cot x] Product Rule
dx dx dx
= (1) cot x + x(− csc x) = cot x − x csc2 x
2
(g) Solution.
d d d
f 0 (x) = [sin x cos x] = [sin x] cos x + sin x [cos x] Product Rule
dx dx dx
= (cos x) cos x + sin x(− sin x) = cos x − sin2 x
2
(h) Solution.
d d
− sin x dx
0 d sin x dx
[sin x]x [x]
f (x) = = 2
Quotient Rule
dx x x
cos x(x) − sin x(1) d
= [sin x] = cos x
x2 dx
x cos x − sin x
=
x2
(i) Solution.
d d
d h sec x i [sec x] csc x − sec x dx [csc x]
f 0 (x) = = dx
Quotient Rule
dx csc x (csc x)2
236 Chapter 4. Differentiation Rules
(m) Solution.
d d
f 0 (x) = [x cos x] + [x2 sin x]
dx dx
d d d d
= [x] cos x + x [cos x] + [x2 ] sin x + x2 [sin x] Product Rule
dx dx dx dx
4.6. Derivatives of Trigonometric Functions 237
(n) Solution.
0 d sin x
f (x) =
dx 1 + tan x
d d
dx
[sin x](1 + tan x) − sin x dx [1 + tan x]
= Quotient Rule
(1 + tan x)2
cos x(1 + tan x) − sin x(0 + sec2 x)
=
(1 + tan x)2
cos x + cos x tan x − sin x sec2 x
=
(1 + tan x)2
(o) Solution.
csc x √
0 d 1
f (x) = + − x 7
dx cos x 3
d √
d 1 d h csc x i
= + − 7
x
dx cos x dx 3 dx
d d
dx
− 1 dx
[1] cos x [cos x] 1 d d h 1i
= + [csc x] − x7
cos2 x 3 dx dx
(0) cos x − (− sin x) 1 1 1
= 2
+ (− csc x cot x) − x 7 −1
cos x 3 7
sin x 1 1
= 2
− csc x cot x − 6
cos x 3 7x 7
(p) Solution. First we have f (x) = (x cos x) sin x. Now we have
d d
f 0 (x) = [x cos x] sin x + x cos x [sin x] Product Rule
dx dx
d d
= [x] cos x + x [cos x] sin x + x cos x(cos x) Product Rule
dx dx
= (cos x − x sin x) sin x + x cos2 x
= cos x sin x − x sin2 x + x cos2 x
(q) Solution.
d d
f 0 (x) = 5 sin4 x = 5 (sin x)4
dx dx
4−1 d Chain Rule:
= 5 4(sin x) [sin x] d d
dx dx
[(g(x))n ] = n(g(x))n−1 dx [g(x)]
238 Chapter 4. Differentiation Rules
d
= 5 4(sin x)3 cos x
[sin x] = cos x
dx
= 20 cos x sin3 x
Warning. Sine of x raised to the power of n is written sinn x. That is, sinn x =
(sin x)n . But sine of xn is written sin xn . That is, sin xn = sin(xn ). Note that
sinn x is not equal to sin xn in general. The same remark applies to the other
trigonometric functions.
(r) Solution.
d
f 0 (x) = x(1 + tan x)8
dx
d d
[x](1 + tan x)8 + x (1 + tan x)8
= Product Rule
dx dx
8 7 d
= (1)(1 + tan x) + x 8(1 + tan x) [tan x] Chain Rule
dx
d
= (1 + tan x)8 + x 8(1 + tan x)7 sec2 x [tan x] = sec2 x
dx
= (1 + tan x)8 + 8x(1 + tan x)7 sec2 x
= (1 + tan x)7 1 + tan x + 8x sec2 x Factor out (1 + tan x)7
d
cos(g(x)) dx [g(x)]. That is,
d d
[sin(g(x))] = cos(g(x)) [g(x)]
dx dx
d d
[cos(g(x))] = − sin(g(x)) [g(x)]
dx dx
d d
[tan(g(x))] = sec2 (g(x)) [g(x)]
dx dx
d d
[cot(g(x))] = − csc2 (g(x)) [g(x)]
dx dx
d d
[sec(g(x))] = sec(g(x)) tan(g(x)) [g(x)]
dx dx
d d
[csc(g(x))] = − csc(g(x)) cot(g(x)) [g(x)]
dx dx
For y = sin(2x), the inner function is g(x) = 2x, and the outer function is
f (x) = sin x. So
d d
[sin(2x)] = cos(2x) [2x] = cos(2x)(2) = 2 cos (2x)
dx dx
(b) Solution.
dy d
= [cos(−3x + 1)]
dx dx
d d d
= − sin(−3x + 1) [−3x + 1] [cos(g(x))] = − sin(g(x)) [g(x)]
dx dx dx
= − sin(−3x + 1)(−3) = 3 sin ( − 3x + 1)
(c) Solution.
dy d
tan(x3 − x2 )
=
dx dx
d d d
= sec2 (x3 − x2 ) [x3 − x2 ] [tan(g(x))] = sec2 (g(x)) [g(x)]
dx dx dx
= sec (x − x )(3x2 − 2x)
2 3 2
(d) Solution.
dy d
= [x sin(x2 + 1)]
dx dx
d d
[x] sin(x2 + 1) + x sin(x2 + 1)
= Product Rule
dx dx
d d
[sin(g(x))] =
= (1) sin(x2 + 1) + x cos(x2 + 1) [x2 + 1] dx
d
dx cos(g(x)) dx [g(x)]
= sin(x2 + 1) + x cos(x2 + 1)(2x)
= sin (x2 + 1) + 2x2 cos (x2 + 1)
(e) Solution.
dy d 2
= 3x cos(πx − 1)
dx dx
d d
= [3x2 ] cos(πx − 1) + 3x2 [cos(πx − 1)] Product Rule
dx dx
d
2 d [cos(g(x))] =
= 6x cos(πx − 1) + 3x − sin(πx − 1) [πx − 1] dx
d
dx − sin(g(x)) dx [g(x)]
= 6x cos(πx − 1) − 3x2 sin(πx − 1)(π)
= 6x cos (πx − 1) − 3πx2 sin (πx − 1)
(f ) Solution.
dy d
= [sin(cos x)]
dx dx
d d d
= cos(cos x) [cos x] [sin(g(x))] = cos(g(x)) [g(x)]
dx dx dx
d
= cos(cos x)(− sin x) [cos x] = − sin x
dx
= − sin x cos ( cos x)
(g) Solution.
dy d
= [cos(sec(4x))]
dx dx
d d d
= − sin(sec(4x)) [sec(4x)] [cos(g(x))] = − sin(g(x)) [g(x)]
dx dx dx
d d
[sec(g(x))] =
= − sin(sec(4x)) sec(4x) tan(4x) [4x] dx
d
dx sec(g(x)) tan(g(x)) dx [g(x)]
= − sin(sec(4x)) sec(4x) tan(4x)(4)
= −4 sin ( sec (4x)) sec (4x) tan (4x)
4.6. Derivatives of Trigonometric Functions 241
(h) Solution.
dy d h √ 2
i
= sin 1 + x
dx dx
√ d h√ i d d
= cos 1 + x2 1 + x2 [sin(g(x))] = cos(g(x)) [g(x)]
dx dx dx
!
√ d
[1 + x 2
] d p
h i d
[g(x)]
= cos 1 + x2 dx√ g(x) = dxp
2 1 + x2 dx 2 g(x)
√
2x
= cos 1 + x 2 √
2 1 + x2
√
x cos 1 + x2
= √
1 + x2
(i) Solution.
dy d 1
= x sin
dx dx x
d 1 d 1
= [x] sin +x sin Product Rule
dx x dx x
d
1 1 d 1 dx
[sin(g(x))] =
= (1) sin + x cos d
x x dx x cos(g(x)) dx [g(x)]
1 1 1 d 1 1
= sin + x cos − 2 =− 2
x x x dx x x
1 x 1
= sin − 2 cos
x x x
1 1 1
= sin ( ) − cos ( )
x x x
(j) Solution.
dy d h 3
4 i
= sec(x + 1)
dx dx
3 d d d
= 4 sec(x3 + 1) sec(x3 + 1) [(g(x))n ] = n(g(x))n−1 [g(x)]
dx dx dx
d
= 4 sec3 (x3 + 1) sec(x3 + 1) tan(x3 + 1) [x3 + 1] Chain Rule
dx
= 4 sec3 (x3 + 1) sec(x3 + 1) tan(x3 + 1)(3x2 )
= 12x2 sec4 (x3 + 1) tan (x3 + 1)
(k) Solution.
dy d
sin 3e5x
=
dx dx
242 Chapter 4. Differentiation Rules
d d d
= cos 5e5x [3e5x ] [sin(g(x))] = cos(g(x)) [g(x)]
dx dx dx
d
= cos 5e5x 3 [e5x ]
Constant Multiple Rule
dx
d d g(x) d
= cos 5e5x 3e5x [5x] = eg(x) [g(x)]
e
dx dx dx
= cos 5e5x 3e5x (5)
(o) Solution. Using the rules (quotient rule, product rule, chain rule, sum rule,
constant rule, constant multiple rule, power rule), we get
d x5 csc(3x)
dy
=
dx dx 1 + 4 cot x2
d d
dx
[x5
csc(3x)](1 + 4 cot x2 ) − x5 csc(3x) dx [1 + 4 cot x2 ]
=
(1 + 4 cot x2 )2
d d d d
dx
[x5 ] csc(3x) + x5 dx [csc(3x)] (1 + 4 cot x2 ) − x5 csc(3x) dx
[1] + 4 dx [cot x2 ]
=
(1 + 4 cot x2 )2
d
5x4 csc(3x) + x5 (− csc(3x) cot(3x)) dx [3x] (1 + 4 cot x2 )
=
(1 + 4 cot x2 )2
d
x5 csc(3x) 0 + 4(− csc2 (x2 ) dx [x2 ])
−
(1 + 4 cot x2 )2
(5x4 csc(3x) − x5 csc(3x) cot(3x)(3)) (1 + 4 cot x2 ) + 4x5 csc(3x) csc2 (x2 )(2x)
=
(1 + 4 cot x2 )2
(5x4 csc(3x) − 3x5 csc(3x) cot(3x)) (1 + 4 cot x2 ) + 8x6 csc(3x) csc2 (x2 )
=
(1 + 4 cot x2 )2
dy d
= [cos(3x)]
dx dx
d d d
= − sin(3x) [3x] [cos(g(x))] = − sin(g(x)) [g(x)]
dx dx dx
− sin(3x)(3) = −3 sin(3x)
The equation of the tangent line to the curve y = cos(3x) at the point π 1
,
9 2
is
√
1 3 3 π
y− =− x−
2 2 9
√ √
1 3 3 3π 3
y− =− x+
2 2 18
√ √
3 3 3π 3 1
y=− x+ +
2 18 2
244 Chapter 4. Differentiation Rules
First we need to find the derivative. Using the Chain Rule, we have
dy d
= [sin(sin x)]
dx dx
d d d
= cos(sin x) [sin x] [sin(g(x))] = cos(g(x)) [g(x)]
dx dx dx
= cos(sin x)(cos x) = cos x cos(sin x)
d
= 6(sin x)5 cos x [sin x] = cos x
dx
The second derivative is
d 0 d
f 00 (x) = 6(sin x)5 cos x
[f (x)] =
dx dx
d
(sin x)5 cos x
=6
dx
d 5
5 d
=6 (sin x) cos x + (sin x) [cos x] Product Rule
dx dx
4 d 5
= 6 5(sin x) [sin x](cos x) + (sin x) (− sin x)
dx
= 6 5(sin x)4 cos x(cos x) − (sin x)5 sin x
0 d h π i
R (t) = 250 sin t + 0.1 + 700
dt 8
d h π i d
= 250 sin t + 0.1 + [700]
dt 8 dt
d h π i
= 250 sin t + 0.1 + 0
dt 8
π d hπ i
= 250 cos t + 0.1 t + 0.1
8 dt 8
π π
= 250 cos t + 0.1
8 8
125π π
= cos ( t + 0.1)
4 8
(b) Solution. Remember that the instantaneous rate of change of nothing but the
derivative. So the instantaneous rate of change of the revenue when t = 3 is
125π π
R0 (3) = cos (3) + 0.1 ≈ $28.33/month
4 8
Note. Remember that to calculate T (x), where T is a basic trigonometric
function (sin, cos, tan, cot, sec or csc) we need to put the calculator in radian
mode.
(c) Solution. Substituting t with 4 into R0 (t), we get
125π π
R0 (4) = cos (4) + 0.1 ≈ −$9.8/month
4 8
This means that after 4 months, that is, in January, the revenue is decreasing
at the rate of $9.8 per month.
246 Chapter 4. Differentiation Rules
dy d kt d kt
= y0 e = y0 e Constant Multiple Rule
dt dt dt
kt d d g(t) d
= y0 e [kt] e = eg(t) [g(t)]
dt dt dt
= y0 ekt (k)
(b) Solution.
i. For y = 10e7t , the initial quantity is y0 = 10 and the growth constant is
k = 7.
ii. For y = 53e0.3t , the quantity present at time t = 0 is y0 = 53 and the
growth constant is k = 0.3.
iii. For y = 110e−0.04t , the initial quantity is y0 = 110 and the decay constant
is k = −0.04.
iv. First we need to express y = 284e2−5t in the form y0 ekt .
(a) If the initial quantity is 17 and the rate of change of y is 4y, then y0 = 17 and
k = 4. Therefore, y = 17e4t . Since k is positive, y grows exponentially.
(b) Here the initial quantity is 300 and dydt
= −0.05y. So y0 = 300, k = −0.05, and
−0.05t
y = 300e . Since k is negative, y decays exponentially.
(a) Solution. Since y grows or decays exponentially, it is of the form y(t) = y0 ekt ,
where y0 and k are to be found.
The condition y(0) = 75 means that the initial quantity is y0 = 75.
Now, we find k. We will use the information y(2) = 105.
y(2) = 105
75e2k = 105 y(t) = 75ekt , y(2) = 75e2k
105
e2k = Divide both sides by 75
75
4.7. Exponential Growth and Decay 247
105
2k = ln Take the natural logarithm of both sides
75
ln 105
75
k= ≈ 0.17 Divide both sides by 2
2
So y = 75e0.17t .
(b) Solution. Because y grows or decays exponentially, y has the form y(t) = y0 ekt .
Since y(0) = 10000, it follows that y0 = 10000. So y = 10000ekt . To find k, we
solve the equation y(5) = 7046.88 or 10000e5k = 7046.88.
10000e5k = 7046.88
7046.88
e5k = Divide both sides by 10000
10000
7046.88
5k = ln Take the natural logarithm of both sides
10000
ln 7046.88
10000
k= ≈ −0.07 Divide both sides by 5
5
Thus, y = 10000e−0.07t .
(c) Solution. Again, y is of the form y(t) = y0 ekt , where y0 and k are to be found.
Substituting t with 2 into y(t) = y0 ekt , we get y(2) = y0 e2k . From the
problem, y(2) = 265.46. So
y0 e2k = 265.46 (4.7.1)
Similarly, the condition y(7) = 308.42 leads us to
y0 e7k = 308.42 (4.7.2)
To find y0 and k, we need to solve the system formed by the equations (4.7.1)
and (4.7.2). From (4.7.1), we have y0 = 265.46e2k
. Substituting this into (4.7.2), we
have
265.46 7k
e = 308.42
e2k
ea
265.46e5k = 308.42 b
= ea−b
e
308.42
e5k = Divide both sides by 265.46
265.46
308.42
5k = ln Take the natural logarithm of both sides
265.46
ln 308.42
265.46
k= ≈ 0.03
5
Using the equation y0 = 265.46
e2k
, we find
265.46 265.46
y0 = = ≈ 250
e2(0.03) e0.06
So y = 250e0.03t .
248 Chapter 4. Differentiation Rules
(d) Solution. We use the same approach as in part (c). First, the equation y(3) =
1182.80 is equivalent to y0 e3k = 1182.8. The second condition, y(8) = 558.72, is
equivalent to y0 e8k = 558.72. Dividing both sides of this latter equation by e8k ,
we get
558.72
y0 =
e8k
Substituting this into y0 e3k = 1182.8, we have
558.72 3k
e = 1182.8 or 558.72e−5k = 1182.8
e8k
Dividing both sides of the latter equation by 558.72, we get e−5k= 1182.8
558.72
. Taking
1182.8
the natural logarithm on both sides, we have −5k = ln 558.72 . Dividing both
sides by −5, we get
1182.8
ln 558.72
k= ≈ −0.15
−5
558.72
Using the equation y0 = e8k
, we find that
558.72 558.72
y0 = 8(−0.15)
= −0.12 ≈ 1855
e e
Hence, y = 1855e−0.15t .
(a) Solution.
The sentence “grows exponentially” means that y is of the form y(t) = y0 ekt .
The sentence “A bacteria culture initially contains 2000 bacteria” means
that y0 = 2000. So y(t) = 2000ekt .
The sentence “After 4 hours the culture contains 3502 bacteria” means that
y(4) = 3502.
Since y(4) = 2000e4k , the equation y(4) = 3502 becomes
2000e4k = 3502
3502
e4k = Divide both sides by 2000
2000
3502
4k = ln Take the natural logarithm of both sides
2000
3502
ln 2000
k= ≈ 0.14
4
So y = 2000e0.14t .
(b) Solution. The number of bacteria after 7 hours is
(c) Solution. The rate of growth after 7 hours is the derivative of y at 3, that is,
y 0 (3). To get y 0 (3), first we need to find the derivative y 0 (t).
d d 0.14t
y 0 (t) = 2000e0.14t = 2000
e Constant Multiple Rule
dt dt
d d g(t) d
= 2000e0.14t [0.14t] e = eg(t) [g(t)]
dt dt dt
0.14t
= 2000e (0.14) = 280e0.14t
We now have
y 0 (7) = 280e0.14(7) = 280e0.98 ≈ 746.05
Thus, after 7 hours, the population of bacteria is increasing at the rate of ap-
proximately 746 bacteria per hour.
(d) Solution. We need to solve the equation y(t) = 10000 for t.
2000e0.14t = 10000
e0.14t = 5 Divide both sides by 2000
0.14t = ln 5 Take the natural logarithm of both sides
ln 5
t= ≈ 11.5 Divide both sides by 0.14
0.14
So, after approximately 11.5 hours, there will be 10000 bacteria.
(a) Solution. Let y(t) be the number of bacteria after t hours. We want to find an
expression for y(t).
The sentence “grows at a rate proportional to its size” means that y grows
exponentially, that is, y is of the form y(t) = y0 ekt .
The sentence “A culture initially contains 33000 bacteria” means that y0 =
33000. So y(t) = 33000ekt .
The sentence “After 7 hours the culture contains 54000 bacteria” means
that y(7) = 54000. This is equivalent to
33000e7k = 54000
54000
e7k = Divide both sides by 33000
33000
54000
7k = ln Take the natural logarithm of both sides
33000
ln 54000
33000
k= ≈ 0.07 Divide both sides by 7
7
So y(t) = 33000e0.07t .
(b) Solution. The number of bacteria after 8 hours is
y(8) = y(t) = 33000e0.07(8) = 33000e0.56 ≈ 57772
250 Chapter 4. Differentiation Rules
d d 0.07t
y 0 (t) = 33000e0.07t = 33000
e
dt dt
d
= 33000e0.07t [0.07t] = 33000e0.07t (0.07) = 2310e0.07t
dt
Substituting t with 8 into y 0 (t), we get
(d) Solution. We want to find t such that y(t) = 2y0 = 2(33000) = 66000.
y(t) = 66000
33000e0.07t = 66000
e0.07t = 2 Divide both sides by 33000
0.07t = ln 2 Take the natural logarithm of both sides
ln 2
t= ≈ 9.9 Divide both sides by 0.07
0.07
So the population of bacteria will double after approximately 9.9 hours.
(a) Solution. Let m(t) be the mass after t years. We want to find a formula for
m(t). Since the sample decays exponentially, m(t) is of the form m(t) = m0 ekt ,
where m0 is the initial mass. From the sentence “Initially, the sample weighs 10
g”, it follows that m0 = 10. So
m(t) = 10ekt
Now, the sentence “After 6 years, the sample weighs 7 g” means that m(6) = 7.
But m(6) = 10ek(6) = 10e6k . So the equation m(6) = 7 becomes
10e6k = 7
e6k = 0.7 Divide both sides by 10
6k = ln(0.7) Take the natural logarithm of both sides
ln(0.7)
k= ≈ −0.06 Divide both sides by 6
6
This means that after 13 years, the mass of the sample is decreasing at the rate
of 0.275 g per year.
m0
(d) Solution. We need to solve the equation m(t) = 2
for t.
m0
m0 e−0.06t =
2
1
e−0.06t = Divide both sides by m0
2
−0.06t = ln(0.5) Take the natural logarithm of both sides
ln(0.5)
t= ≈ 11.55 Divide both sides by −0.06
−0.06
Thus, half of the weight of the sample will remain after approximately 11.55
years.
Half-life
Let y = y(t) be a quantity that decays over time (for example a radioactive
substance), and let y0 be the initial quantity. The half-life of y is the time
required for y0 to be reduced by one-half. So, if T is the half-life of y, then
y0
y(T ) =
2
y0
To find the half-life of y, all we have to do is to solve the equation y(t) = 2
for t.
Here we have m(t) = m0 e−0.02476t . To find the half-life, we need to solve the equation
m(t) = m20 for t.
m0
m0 e−0.02476t =
2
1
e−0.02476t = Divide both sides by m0
2
252 Chapter 4. Differentiation Rules
(a) Solution. Let m(t) be the mass remaining after t days. First, we need to find
an expression for m(t) of the form m0 ekt .
From the sentence “A sample has a mass of 35 mg initially”, it follows that
m0 = 35. So m(t) = 35ekt .
Since the half-life is 140 days, it follows that m(140) = 35
2
. But m(140) =
35e140k . So the equation m(140) = 35 2
becomes
35
35e140k =
2
1
e140k = Divide both sides by 35
2
140k = ln(0.5) Take the natural logarithm of both sides
ln(0.5)
k= ≈ −0.005 Divide both sides by 140
140
So m(t) = 35e−0.005t .
Now, the mass remaining after 200 days is
35e−0.005t = 6
6
e−0.005t = Divide both sides by 35
35
6 Take the natural loga-
−0.005t = ln
35 rithm of both sides
6
ln 35
t= ≈ 352.72 days Divide both sides by −0.005
−0.005
(a) Solution. We will use the following theorem which was stated in Exercise 2.
4.7. Exponential Growth and Decay 253
y(t) = y0 ekt .
dy
= 0.0131y
dt
This means that k = 0.0131.
So, by Theorem 4.7.1, the population after t years is
(c) Solution. In 2025, t = 25 and the required rate of change is y 0 (25). But
d d 0.0131t
y 0 (t) = 6.143e0.0131t = 6.143
e Constant Multiple Rule
dt dt
d d g(t) d
= 6.143e0.0131t [0.0131t] e = eg(t) [g(t)]
dt dt dt
0.0131t
= 6.143e (0.0131) = 0.08e0.0131t
This means that in 2025, the world population will be increasing at a rate of
approximately 0.11 billion per year.
254 Chapter 4. Differentiation Rules
(c) Solution. The rate of growth of A after 5 years is the derivative of A evaluated
at 5, that is, A0 (5). So we first need to find A0 (t).
d d 0.06t
A0 (t) = 25500e0.06t = 25500
e Constant Multiple Rule
dt dt
d d g(t) d
= 25500e0.06t [0.06t] e = eg(t) [g(t)]
dt dt dt
0.06t
= 25500e (0.06) = 1530e0.06t
This means that after 5 years the amount on deposit is increasing at the rate of
approximately $2065.28 per year.
7500e2r = 8247.44
8247.44
e2r = Divide both sides by 7500
7500
8247.44 Take the natural logarithm
2r = ln
7500 of both sides
8247.44
ln 7500
r= ≈ 0.0475 = 4.75% Divide both sides by 2
2
So
A(t) = 7500e0.0475t
Substituting t with 8 into A(t), we get the amount on deposit after 8 years:
(a) Solution.
256 Chapter 4. Differentiation Rules
Present Value
Suppose P dollars is invested at an interest rate r compounded m
times per year. Then the compound amount (denoted A) after t
years is given by the formula
r tm
A=P 1+ (4.7.3)
m
If A, r, m, and t are given, then P can be found using (4.7.3). The
amount P is called the present value of A dollars. In other words,
the present value of A dollars is the amount of money that should
be invested today in order to end up with A dollars after t years.
(a) Solution.
(ii) Find all numbers for which the derivative is zero. To do this, set the
derivative f 0 (x) equal to zero and solve for x.
(iii) Find all numbers c for which f 0 (c) is undefined and f (c) is defined.
(iv) The critical numbers for f are the numbers found in steps (ii) and
(iii).
259
260 Chapter 5. Applications of the Derivative I
d −1 1
f 0 (x) = [x ] = (−1)x−2 = − 2
dx x
Set f 0 (x) = 0. Then − x1 = 0. Multiplying both sides by x, we get −1 = 0,
which is not true. So the equation f 0 (x) = 0 has no solution.
The derivative does not exist where the denominator is 0, that is, where
x2 = 0 or x = 0. So f 0 (0) does not exist. But the function is undefined at
0 because f (0) = 01 and the division by 0 is not defined. (In other words,
x = 0 is not in the domain of f .) So 0 is not a critical number.
Thus, the function f (x) = x1 has no critical number.
(g) Solution.
The derivative of f (x) = 2x+1
x−3
is
d d
0 dx
[2x + 1](x − 3) − (2x + 1) dx [x − 3]
f (x) = 2
Quotient Rule
(x − 3)
2(x − 3) − (2x + 1)(1) 2x − 6 − 2x − 1 −7
= 2
= 2
=
(x − 3) (x − 3) (x − 3)2
Set (x−3)
−7 2
2 = 0. Then, by multiplying both sides by (x − 3) , we get −7 = 0,
(a) Solution.
Increasing/Decreasing Test
(ii) Mark the critical numbers on the real line, as well as any points
where f is undefined. These points determine a certain number of
open intervals.
(iii) To find the sign of f 0 (x) on an open interval I found in step (ii),
choose any number, say t, in I. Then substitute x with t into
the derivative f 0 (x) . If f 0 (t) is positive, the function is increasing
on I. If f 0 (t) is negative, the function is decreasing on I.
−∞ ∞
2
+ −
−∞ 2 ∞
choose 0 choose 3
f 0 (0) = 8 > 0 f 0 (3) = −4 < 0
+ − +
−∞ ∞
− 23 0
choose −1 choose −0.5 choose 1
f 0 (−1) = 1 > 0 f 0 (−0.5) = −0.25 < 0 f 0 (1) = 5 > 0
(c) Solution.
From Problem 1(c), the critical numbers for f are −3, 0, and 1.
These numbers determine four intervals, and the sign of f 0 (x) in each of
these is shown in Figure 5.4. We choose 0.5 as the test value in the interval
(0, 1): f 0 (0.5) = 10.5 > 0.
− + − +
−∞ −3 0 1 ∞
choose −4 choose −2 choose 2
f 0 (−4) = −240 < 0 f 0 (−2) = 72 > 0 f 0 (2) = 120 > 0
From Figure 5.4, f is increasing on the intervals (−3, 0) and (1, ∞),
and decreasing on the intervals (−∞, −3) and (0, 1).
5.1. Increasing and Decreasing Functions 265
(d) Solution.
2
We first need to find the critical numbers of f (x) = (x − 1) 3 .
– The derivative is
2 1 d 2 1 2
f 0 (x) = (x − 1)− 3 [x − 1] = 1 (1) = 1 .
3 dx 3 (x − 1) 3 3(x − 1) 3
2 1
– Set 1 = 0. Multiplying both sides by 3(x − 1) 3 , we get 2 = 0,
3(x−1) 3
which is not true. So the equation f 0 (x) = 0 has no solution.
– The derivative does not exist when x = 1. Moreover, f (1) is defined
2
since f (1) = (1 − 1) 3 = 0.
– Thus, the critical number for f is 1.
The critical number 1 determines two open intervals: (−∞, 1) and (1, ∞).
And the sign of f 0 (x) in each of these is shown in Figure 5.5.
− +
−∞ 1 ∞
choose 0 choose 2
f 0 (0) = − 23 < 0 f 0 (2) = 2
3
>0
− −
−∞ 3 ∞
choose 0 choose 4
f 0 (0) = − 79 < 0 f 0 (4) = −7 < 0
−7
Figure 5.6: Sign of f 0 (x) = (x−3)2
266 Chapter 5. Applications of the Derivative I
Alternate Solution
−7
The derivative of f (x) is f 0 (x) = (x−3) 2 . Since the denominator is always
(f ) Solution.
We first need to find the critical numbers of f (x) = x2x+1 .
– The derivative is
d d
[x](x2 + 1) − x dx [x2 + 1]
0 d x dx
f (x) = = Quotient Rule
dx x2 + 1 (x2 + 1)2
1(x2 + 1) − x(2x) x2 + 1 − 2x2 1 − x2
= = =
(x2 + 1)2 (x2 + 1)2 (x2 + 1)2
2
– Set (x1−x 2 2 2
2 +1)2 = 0. Multiplying both sides by (x + 1) , we get 1 − x = 0.
− + −
−∞ −1 1 ∞
choose −2 choose 0 choose 2
f 0 (−2) = − 25
3
<0 f 0 (0) = 1 > 0 f 0 (2) = − 25
3
<0
1−x2
Figure 5.7: Sign of f 0 (x) = (x2 +1)2
– The derivative is
d 3 d 3
f 0 (x) = [x]e−x + x [e−x ] Product Rule
dx dx
3 3 d d g(x) d
= (1)e−x + xe−x [−x3 ] e = eg(x) [g(x)]
dx dx dx
3 3 3 3
= e−x + xe−x (−3x2 ) = e−x − 3x3 e−x
3 3
= (1 − 3x3 )e−x Factor out e−x
3 3
– Set (1 − 3x3 )e−x = 0. Then 1 − 3x3 = 0. (Note that e−x 6= 0 for all
x.) Subtracting 1 from both sides, we get −3x3 = −1. Dividing both
sides by −3, we get x3 = 13 . Taking the cube root of both sides, we have
q
x = 3 13 .
– Since the derivative is defined everywhere, there is no c for which f 0 (c)
does not exist. q
– So the critical number for f is 3 13 .
This determines two open intervals, and the sign of f 0 (x) in each of these is
shown in Figure 5.8
+ −
−∞ q ∞
3 1
3
choose 0 choose 1
f 0 (0) = 1 > 0 f 0 (1) = −2e−1 < 0
3
Figure 5.8: Sign of f 0 (x) = (1 − 3x3 )e−x
q
Thus, f is increasing on the interval −∞, 3 13 and decreasing on
q
the interval 3 13 , ∞ .
(h) Solution.
7x2 +6
We first need to find the critical numbers for f (x) = ln x2 +3
.
– The derivative is
h 2 i
d 7x +6 d
0 dx x2 +3 d dx
[g(x)]
f (x) = 7x2 +6
[ln g(x)] =
x2 +3
dx g(x)
d d
dx
[7x2 +6](x2 +3)−(7x2 +6) dx [x2 +3]
2
(x +3)2
= 7x2 +6
Quotient Rule
x2 +3
268 Chapter 5. Applications of the Derivative I
− +
−∞ 0 ∞
choose −1 choose 1
−15
f 0 (−1) = 26
<0 f 0 (1) = 15
26
>0
(a) Solution.
The profit function is
The derivative is
d d
P 0 (x) = [−0.00034x3 ] + [0.656x2 ] = −000034(3x2 ) + 0.656(2x)
dx dx
= −0.00102x2 + 1.312x = x(−0.00102x + 1.312)
5.2. Relative (or Local) Extrema 269
+ −
0 1286.27 4000
choose 1 choose 1300
P 0 (1) = 1.31 > 0 P 0 (1300) = −18.2 < 0
From Figure 5.10, we deduce that the profit is increasing on the interval
(0, 1286.27).
(b) Solution. Again from Figure 5.10, the profit is decreasing when the produc-
tion level is between 1286.27 and 4000 items.
If
f 0 (x) > 0 for all x near c, x < c and f 0 (x) < 0 for all x near c, x > c
If
f 0 (x) < 0 for all x near c, x < c and f 0 (x) > 0 for all x near c, x > c
If the derivative f 0 (x) is positive for all x near c, or negative for all x near
c, then f has no relative minimum or maximum at c.
(a) Solution.
(ii) Use the first derivative test to determine the relative maximum
and/or minimum (if any).
+ −
−∞ 2 ∞
choose 0 choose 3
f 0 (0) = 8 > 0 f 0 (3) = −4 < 0
+ − +
−∞ ∞
− 23 0
choose −1 choose −0.5 choose 1
f 0 (−1) = 1 > 0 f 0 (−0.5) = −0.25 < 0 f 0 (1) = 5 > 0
f (0) = 03 + 02 + 5 = 5.
(c) Solution. In Section 5.1–Problem 2(c), we found the intervals where f is in-
creasing or decreasing. These are summarized in Figure 5.13 below.
− + − +
−∞ −3 0 1 ∞
choose −4 choose −2 choose 2
f 0 (−4) = −240 < 0 f 0 (−2) = 72 > 0 f 0 (2) = 120 > 0
and
f (1) = 3(1)4 + 8(1)3 − 18(1)2 = 3 + 8 − 18 = −7.
(d) Solution. In Section 5.1–Problem 2(d), we found the intervals where f is
increasing or decreasing. These are shown in Figure 5.14.
− +
−∞ 1 ∞
choose 0 choose 2
f 0 (0) = − 23 < 0 f 0 (2) = 2
3
>0
− −
−∞ 3 ∞
choose 0 choose 4
f 0 (0) = − 79 < 0 f 0 (4) = −7 < 0
−7
Figure 5.15: Sign of f 0 (x) = (x−3)2
From Figure 5.15, there is no relative extremum because the sign of the
derivative does not change.
(f ) Solution. In Section 5.1–Problem 2(f), we found the intervals where f is in-
creasing or decreasing. These are shown in Figure 5.16.
From Figure 5.16, the relative minimum of f is f (−1) = −1
(−1)2 +1
= − 12 .
The relative maximum is f (1) = 1
12 +1
= 12 .
(g) Solution.
5.2. Relative (or Local) Extrema 273
− + −
−∞ −1 1 ∞
choose −2 choose 0 choose 2
f 0 (−2) = − 25
3
<0 f 0 (0) = 1 > 0 f 0 (2) = − 25
3
<0
1−x2
Figure 5.16: Sign of f 0 (x) = (x2 +1)2
− +
0 e−1 ∞
5
≈ 0.074
choose 0.05 choose 1
f 0 (0.05) ≈ −1.55 < 0 f 0 (1) ≈ 10.44 > 0
+ −
−∞ q ∞
3 1
3
choose 0 choose 1
f 0 (0) = 1 > 0 f 0 (1) = −2e−1 < 0
3
Figure 5.18: Sign of f 0 (x) = (1 − 3x3 )e−x
d 2x d d
f 0 (x) = 5(e2x − 3)4 [e − 3] [(g(x))n ] = n(g(x))n−1 [g(x)]
dx dx dx
d 2x d
= 5(e2x − 3)4 [e ] − [3] Difference Rule
dx dx
2x 4 2x d d g(x) d
= 5(e − 3) e [2x] − 0 e = eg(x) [g(x)]
dx dx dx
= 5(e2x − 3)4 2e2x = 10e2x (e2x − 3)4
Set f 0 (x) = 0. Then 10e2x (e2x − 3)4 = 0. This implies that (e2x − 3)4 = 0.
(Note that 10e2x 6= 0 for all x.) Taking the 4th root of both sides, we get
e2x − 3 = 0. Adding 3 to both sides, we get e2x = 3. Taking the natural
logarithm on both sides, we get 2x = ln 3. Dividing both sides by 2, we get
x = ln23 .
So the critical number for f is c = ln23 .
The sign of the derivative is shown in Figure 5.19.
From Figure 5.19, there is no relative extremum.
(j) Solution. In Section 5.1–Problem 2(h), we found the intervals where f is
increasing or decreasing. These are shown in Figure 5.20.
5.2. Relative (or Local) Extrema 275
+ +
−∞ ∞
ln 3
2
≈ 0.55
choose 0 choose 1
f 0 (0) = 160 > 0 f 0 (1) = 10e2 (e2 − 3)4 > 0
− +
−∞ 0 ∞
choose −1 choose 1
−15
f 0 (−1) = 26
<0 f 0 (1) = 15
26
>0
7(0)2 + 6
6
f (0) = ln 2
= ln = ln (2)
0 +3 3
(a) Solution.
We first need to find the profit function P (q). Using the formula P (q) =
R(q) − C(q), where R(q) = pq, is the revenue, we get
d d
P 0 (q) = [P (q)] = [−3q 2 + 72q − 100] = −6q + 72
dq dq
+ −
0 12 ∞
choose 1 choose 13
P 0 (1) = 66 > 0 P 0 (13) = −6 < 0
p = −3(12) + 80 = −36 + 80 = 44
(a) Solution.
The profit function is
The derivative is
d d
P 0 (q) = 50qe−0.02q − [120]
Difference Rule
dq dq
d d −0.02q Product Rule and
= [50q]e−0.02q + 50q e −0
dq dq Constant Rule
d d g(q) d
= 50e−0.02q + 50qe−0.02q [−0.02q] e = eg(q) [g(q)]
dq dq dq
d
= 50e−0.02q + 50qe−0.02q (−0.02) [−0.02q] = −0.02
dq
= 50e−0.02q − qe−0.02q = (50 − q)e−0.02q Factor out e−0.02q
+ −
0 50 ∞
choose 1 choose 51
P 0 (1) = 49e−0.02 > 0 P 0 (51) = −e−1.02 < 0
(a) Solution.
The profit function is
+ −
0 1286.27 4000
choose 1 choose 1300
P 0 (1) = 1.31 > 0 P 0 (1300) = −18.2 < 0
From Figure 5.23, the number of units that should be sold in order to
maximize the profit is approximately 1286.27.
(b) Solution. The maximum profit is
5.3 Concavity
1. [Exercise on page 43]
Solution.
The function f whose graph is shown in Figure 5.2 is concave downward on the
interval (−∞, 1) and concave upward on the interval (1, ∞).
Since the concavity changes at x = 1, the point (1, −2) is an inflection point.
(a) Solution.
Second Derivative
Let f be a function.
(b) Solution.
d d 0 d
f 0 (x) = [−5x + 6] = −5 and f 00 (x) = [f (x)] = [−5] = 0
dx dx dx
(c) Solution. The first derivative is
d h −x3 i 3 d d g(x) d
f 0 (x) = e = e−x [−x3 ] e = eg(x) [g(x)]
dx dx dx dx
3 3
= e−x (−3x2 ) = −3x2 e−x
d d
= [2x − 3x2 ]e−3x + (2x − 3x2 ) [e−3x ] Product Rule
dx dx
−3x 2 −3x
= (2 − 6x)e + (2x − 3x )e (−3)
= (2 − 6x)e−3x + (−6x + 9x2 )e−3x
= (2 − 6x − 6x + 9x2 )e−3x = (2 − 12x + 9x2 )e−3x
0 d −x
d
dx
[x + e−x ] d d
dx
[g(x)]
f (x) = log(x + e ) = [loga g(x)] =
dx (ln 10)(x + e−x ) dx (ln a)g(x)
d
[x]
+ dxd
[e−x ] 1 − e−x d −x
= dx
= [e ] = −e−x
(ln 10)(x + e−x ) (ln 10)(x + e−x ) dx
(a) Solution.
Higher Derivatives
Let f be a function.
d (n−1)
f (n) (x) = f (x) .
dx
(a) Solution.
Concavity Test
Let f be a function.
We will write C.D for concave downward and C.U for concave upward.
(b) Solution.
The first derivative of f (x) = x3 − 3x2 + 5 is f 0 (x) = 3x2 − 6x, and the
second derivative is
f 00 (x) = 6x − 6.
Set f 00 (x) = 0. Then 6x − 6 = 0. Adding 6 to both sides, we get 6x = 6.
Dividing both sides by 6, we get x = 1.
The sign of f 00 (x) is shown in Figure 5.24.
(d) Solution.
The first derivative of f (x) = −3
x+1
is
d d
[−3](x + 1) − (−3) dx [x + 1]
f 0 (x) = dx
Quotient Rule
(x + 1)2
0(x + 1) + 3(1) 3
= 2
=
(x + 1) (x + 1)2
The second derivative is
d d
d 0 [3](x + 1)2 − 3 dx [(x + 1)2 ]
f 00 (x) = [f (x)] = dx
Quotient Rule
dx (x + 1)4
0(x + 1)2 − 3(2(x + 1)(1))
=
(x + 1)4
−6(x + 1) −6
= 4
=
(x + 1) (x + 1)3
Set f 00 (x) = 0.Then (x+1)
−6 3
3 = 0. Multiplying both sides by (x + 1) , we get
From Figure 5.26, f is concave upward on the interval (−∞, −1) and
concave downward on the interval (−1, ∞).
Although the concavity changes at −1, there is no inflection point because
f (x) is not defined at −1.
(e) Solution.
Using the quotient rule, the first derivative of f (x) = x
x2 +1
is
d d
0 dx
[x](x2 + 1) − x dx [x2 + 1] (1)(x2 + 1) − x(2x)
f (x) = =
(x2 + 1)2 (x2 + 1)2
x2 + 1 − 2x2 −x2 + 1
= =
(x2 + 1)2 (x2 + 1)2
The second derivative is
d −x2 + 1
00 d 0
f (x) = [f (x)] =
dx dx (x2 + 1)2
284 Chapter 5. Applications of the Derivative I
2x(x2 −3)
Figure 5.27: Sign of f 00 (x) = (x2 +1)3
√
From Figure
√ 5.27, f is concave downward on the intervals (−∞,
√ − 3)
√ (0, 3), and concave upward on the intervals (− 3, 0) and
and
( 3, ∞).
√
Again
√ from Figure 5.27, there are three inflection points: at − 3, 0, and
3. Since,
√ √ √ √
√ 3 3 0 √ 3 3
f (− 3) = =− , f (0) = 2 = 0, f ( 3) = = ,
3+1 4 0 +1 3+1 4
√ √ √ √
the inflection points are − 3, − 43 , (0, 0), and 3, 43 .
(f ) Solution.
The first derivative of f (x) = xe−4x is
d d d
f 0 (x) = [xe−4x ] = [x]e−4x + x [e−4x ] Product Rule
dx dx dx
d g(x) d
= e−4x + x e−4x (−4) = eg(x) [g(x)]
e
dx dx
= (1 − 4x)e−4x Factor out e −4x
d d
= [1 − 4x]e−4x + (1 − 4x) [e−4x ] Product Rule
dx dx
= (−4)e−4x + (1 − 4x)(−4e−4x ) = −4e−4x + (−4 + 16x)e−4x
= (−4 − 4 + 16x)e−4x = (−8 + 16x)e−4x Factor out e−4x
(g) Solution.
The first derivative of f (x) = ln(x2 + 4) is
d d
0 dx
[x2 + 4] 2x d dx
[g(x)]
f (x) = = 2 [ln g(x)] =
x2 + 4 x +4 dx g(x)
The second derivative is
00 d 0 d 2x
f (x) = [f (x)] =
dx dx x2 + 4
d d
dx
[2x](x2 + 4) − 2x dx [x2 + 4]
= Quotient Rule
(x2 + 4)2
2(x2 + 4) − 2x(2x) 2x2 + 8 − 4x2
= =
(x2 + 4)2 (x2 + 4)2
−2x2 + 8 −2(x2 − 4) −2(x − 2)(x + 2)
= = =
(x2 + 4)2 (x2 + 4)2 (x2 + 4)2
−2(x−2)(x+2)
Figure 5.29: Sign of f 00 (x) = (x2 +4)2
d d d
f 0 (x) = 4(x − 1)3 [x − 1] [(g(x))n ] = n(g(x))n−1 [g(x)]
dx dx dx
= 4(x − 1)3 (1) = 4(x − 1)3
Set f 00 (x) = 0. Then 12(x − 1)2 = 0. Dividing both sides by 12, we get
(x − 1)2 = 0. This implies that x = 1.
The sign of f 00 (x) is shown in Figure 5.30.
concave up concave up
+ +
−∞ 1 ∞
choose 0 choose 2
f 00 (0) = 12 > 0 f 00 (2) = 12 > 0
(a) Solution.
The test is inconclusive if f 00 (c) = 0 or f 00 (c) does not exist. In that case,
we use the first derivative test.
f 0 (x) = −8 − 2x
So 17 is a relative maximum of f .
(b) Solution.
The first derivative is
(c) Solution.
The first derivative is
f 0 (x) = 4(x + 1)3
Setting 4(x + 1)3 = 0, we get x = −1. So the critical number of f is −1.
The second derivative is
f 00 (x) = 12(x + 1)2
Evaluating this at the critical number, we have f 00 (−1) = 12(−1 + 1)2 = 0.
Since f 00 (−1) = 0, the second derivative test gives no information about
relative extrema.
So we will use the first derivative test. The sign of the first derivative is
shown in Figure 5.31.
− +
−∞ −1 ∞
choose −2 choose 0
f 0 (−2) = −4 < 0 f 0 (0) = 4 > 0
From Figure 5.31, the function is decreasing on the interval (−∞, −1) and
increasing on (−1, ∞). So f (−1) = (−1 + 1)4 = 0 is a relative minimum.
6. [Exercise on page 44]
(a)
Point of Diminishing Returns
Let y = f (x) be a function (for example the revenue function). The point
of diminishing returns is nothing but the inflection point of f .
We want to find the inflection point of R(x) = −x3 + 12x2 + 100, where 0 ≤ x ≤
10.
The first derivative is R0 (x) = −3x2 + 24x.
The second derivative is R00 (x) = −6x + 24.
Set R00 (x) = 0. Then −6x + 24 = 0. Subtracting 24 from both sides, we get
−6x = −24. Dividing both sides by −6, we get x = 4.
The sign of R00 (x) is shown in Figure 5.32.
Since the concavity changes at 4, there is an inflection point there. Evalu-
ating R(x) at 4, we get
R(4) = −43 + 12(4)2 + 100 = −64 + 192 + 100 = 228.
So the point of diminishing returns is (4, 228).
5.4. Curve Sketching 289
(b) Solution.
The first derivative of R(x) = xe−x is
d d
R0 (x) = [x]e−x + x [e−x ] Product Rule
dx dx
d −x
= e−x + x(−e−x ) [e ] = −e−x
dx
= e−x − xe−x = (1 − x)e−x Factor out e−x
Using the same rules, the second derivative is
d d
R00 (x) = [1 − x]e−x + (1 − x) [e−x ]
dx dx
−x −x
= −e + (1 − x)(−e )
= (−1 − 1 + x)e−x = (−2 + x)e−x
Set R00 (x) = 0. Then (−2 + x)e−x = 0. This implies that −2 + x = 0.
(Remember that ex 6= 0 for all x.) Solving the latter equation, we get
x = 2.
The sign of R00 (x) is given by Figure 5.33.
To sketch the graph of f (x) = −x3 + 3x2 − 1, we will go through the following steps.
Intercepts
The x-intercepts of f are the intersection (if any) between the graph
of f and the x-axis. They are obtained by solving the equation
f (x) = 0 for x.
and
lim f (x) = lim (−x3 + 3x2 − 1) = lim (−x3 ) = ∞
x→−∞ x→−∞ x→−∞
Since lim f (x) is not a number and lim f (x) is not a number either, there
x→∞ x→−∞
is no horizontal asymptote.
Vertical Asymptotes. Polynomial functions do not have vertical asymp-
totes. For rational functions, we can locate the vertical asymptotes by
equating the denominator to 0 after canceling any common factors. Since
f (x) = −x3 + 3x2 − 1 is a polynomial, there is no vertical asymptote.
(d) Intervals where f is Increasing or Decreasing and Relative Extrema
The derivative is f 0 (x) = −3x2 + 6x = −3x(x − 2).
Set −3x(x − 2) = 0. Then x = 0 or x = 2. So the critical numbers are 0
and 2. Evaluating f at the critical numbers, we get
− + −
−∞ 0 2 ∞
choose −1 choose 1 choose 3
f 0 (−1) = −9 < 0 f 0 (1) = 3 > 0 f 0 (3) = −9 < 0
From Figure 5.34, f is decreasing on the intervals (−∞, 0) and (2, ∞), and
increasing on the interval (0, 2).
Relative Extrema. Again from Figure 5.34, the relative minimum is
f (0) = −1, and the relative maximum is f (2) = 3.
(e) Concavity and Inflection Points.
The second derivative is
d 0 d
f 00 (x) = [f (x)] = [−3x2 + 6x] = −6x + 6 = −6(x − 1)
dx dx
From Figure 5.35, f is concave upward on the interval (−∞, 1) and concave
downward on the interval (1, ∞).
Since the concavity changes at 1, there is an inflection point there. Evalu-
ating f at 1, we get
(g) Sketching the Curve. To sketch the curve, one can go through the following
steps.
Plot the intercepts. Here the y-intercept is (0, −1).
Plot the critical points: (0, −1) and (2, 3).
Plot the inflection points: (1, 1).
Graph the asymptotes. Here there is no asymptote.
Connect the points with a curve by taking the concavity into account.
The graph of f is shown in Figure 5.36.
y
4
−3 −2 −1 1 2 3 4 x
−1
−2
− − +
−∞ 0 3 ∞
choose −1 choose 1 choose 4
f 0 (−1) = −16 < 0 f 0 (1) = −8 < 0 f 0 (4) = 64 > 0
This shows that f is decreasing on the intervals (−∞, 0) and (0, 3), and increas-
ing on the interval (3, ∞). The relative minimum of f is f (3) = −6. There is
no relative maximum.
(e) Concavity and Inflection Points.
The second derivative of f (x) is
d 0 d
f 00 (x) = [f (x)] = [4x3 − 12x2 ] = 12x2 − 24x
dx dx
Set f 00 (x) = 0. Then
12x2 − 24x = 0
12x(x − 2) = 0 Factor out 12x
x = 0 or x = 2
The sign of f 00 (x) is shown in the following figure.
concave up concave down concave up
+ − +
−∞ 0 2 ∞
choose −1 choose 1 choose 3
f 00 (−1) = 36 > 0 f 00 (1) = −12 < 0 f 00 (3) = 36 > 0
294 Chapter 5. Applications of the Derivative I
This shows that f is concave upward on the intervals (−∞, 0) and (2, ∞), and
concave downward on the interval (0, 2). Since the concavity changes at 0 and
2, there are inflection points there. Since
f (0) = 21 and f (2) = 24 − 4(2)3 + 21 = 16 − 32 + 21 = 5,
it follows that (0, 21) and (2, 5) are the inflection points of f .
(f ) Summary of All the Information Above in a Table. In the following
table, C.U stands for concave upward, and C.D for concave downward.
30
20
10
−2 −1 1 2 3 4 5 x
−10
(c) Asymptotes.
Horizontal Asymptotes. We need to find the limits at infinity. Remem-
bering how to find the limit of a rational function at infinity, we have
−2x2
lim f (x) = lim
x→∞ x→∞ x2 − 1
−2x2
x2
= lim 2 Divide each term by x2
x→∞ x − 1
x2 x2
−2
= lim Simplify
x→∞ 1 − x12
lim (−2)
x→∞
= 1
lim (1) − lim x2
x→∞ x→∞
−2 1
= lim (k) = k, lim = 0 for all n > 0
1−0 x→∞ x→∞ xn
−2
= = −2
1
d −2x2
0
f (x) =
dx x2 − 1
d d
dx
[−2x2 ](x2 − 1) − (−2x2 ) dx [x2 − 1]
= Quotient Rule
(x2 − 1)2
−4x(x2 − 1) + 2x2 (2x)
=
(x2 − 1)2
−4x3 + 4x + 4x3 4x
= 2 2
= 2
(x − 1) (x − 1)2
− − + +
−∞ −1 0 1 ∞
choose −2 choose −0.5 choose 0.5 choose 2
f 0 (−2) = − 89 < 0 f 0 (−0.5) < 0 f 0 (0.5) > 0 f 0 (2) = 8
9
>0
From this figure, f is decreasing on the intervals (−∞, −1) and (−1, 0), and
increasing on the intervals (0, 1) and (1, ∞).
Relative Extrema. The relative minimum is f (0) = 0. There is no relative
maximum.
(e) Concavity and Inflection Points.
The second derivative is
00 d 0 d 4x
f (x) = [f (x)] =
dx dx (x2 − 1)2
d d
dx
[4x](x2 − 1)2 − 4x dx [(x2 − 1)2 ]
= Quotient Rule
(x2 − 1)4
4(x2 − 1)2 − 4x(2)(x2 − 1)(2x)
=
(x2 − 1)4
4(x2 − 1)2 − 16x2 (x2 − 1)
=
(x2 − 1)4
(x2 − 1)[4(x2 − 1) − 16x2 ]
= Factor out x2 − 1
(x2 − 1)4
(x2 − 1)(4x2 − 4 − 16x2 )
=
(x2 − 1)4
−12x2 − 4 −4(3x2 + 1) Simplify and factor out
= =
(x2 − 1)3 (x2 − 1)3 −4
2
Set f 00 (x) = 0. Then −4(3x +1)
(x2 −1)3
= 0. Multiplying both sides by (x2 − 1)3 , we
get −4(3x2 + 1) = 0. Dividing both sides by −4, we get 3x2 + 1 = 0. This
latter equation has no solution because 3x2 + 1 is always a positive number.
The sign of f 00 (x) is shown in the following figure.
From this figure, f is concave downward on the intervals (−∞, −1) and
(1, ∞), and concave upward on the interval (−1, 1).
5.4. Curve Sketching 297
Although the concavity changes at −1 and 1, these are not inflection points
because f (x) in undefined at −1 and 1.
(f ) Summary. The information above can be summarized in the following table,
where C.U stands for concave upward and C.D stands for concave downward.
y
x = −1 x=1
−1 1 x
−2 y = −2
−2x2
Figure 5.38: Graph of f (x) = x2 −1
6
(a) Domain. The function f (x) = x2 −x−6 is defined everywhere except where the
denominator is zero. Solving the equation x2 −x−6 = 0, we get (x+2)(x−3) = 0;
that is, x = −2 or x = 3. So, the domain of f is the set of all real numbers
except −2 and 3.
(b) Intercepts.
The y-intercept is f (0) = 6
02 −0−6
= −1.
298 Chapter 5. Applications of the Derivative I
Since this latter equality is not true, it follows that the equation f (x) = 0
has no solution. Therefore, f has no x-intercept.
(c) Asymptotes.
Finding the horizontal asymptotes. To find the horizontal asymptotes, we
need to find two limits: lim f (x) and lim f (x).
x→∞ x→−∞
6
lim f (x) = lim
x→∞ x→∞ x2 −x−6
6
x2
= lim 2 Divide each term by x2
x→∞ x − x
− 6
x2 x2 x2
6
x2
= lim 1 Simplify
x→∞ 1− x
− x62
6
lim x2
x→∞
= 1
6
lim (1) − lim x
− lim x
x→∞ x→∞ x→∞
0 1
= lim = 0 for all n > 0
1−0−0 x→∞ xn
0
= =0
1
Similarly, we have lim f (x) = 0. So, the line y = 0 is a horizontal asymp-
x→−∞
tote of f at both −∞ and ∞.
Finding the vertical asymptotes. The denominator of f (x), x2 − x − 6, is
zero when x = −2 or x = 3. To find the vertical asymptotes, we need to
find the limits lim f (x) and lim f (x). For the first limit, if we substitute
x→−2 x→3
x with −2 into f (x), we have 06 , which is a limit of the form k0 with k 6= 0.
Using the method from Section 3.3, we need to determine the sign of f (x).
This is given by the following figure.
+ − +
−∞ −2 3 ∞
choose −3 choose 0 choose 4
f (−3) = 1 > 0 f (0) = −1 < 0 f (4) = 1 > 0
The sign analysis for f (x) tells us that the one-sided limits of f (x) as x → −2
are
lim − f (x) = ∞ and lim + f (x) = −∞
x→−2 x→−2
5.4. Curve Sketching 299
This shows that the line x = −2 is a vertical asymptote. Likewise, the line
x = 3 is a vertical asymptote of f (x).
(d) Increasing/Dcreasing and Relative Extrema.
6
The derivative of f (x) = x2 −x−6 is
d d
[6](x2 − x − 6) − 6 dx [x2 − x − 6]
f 0 (x) = dx
Quotient Rule
(x2 − x − 6)2
0(x2 − x − 6) − 6(2x − 1)
=
(x2 − x − 6)2
−6(2x − 1)
=
(x2 − x − 6)2
Set f 0 (x) = 0. Then
−6(2x − 1)
=0
(x2 − x − 6)2
−6(2x − 1) = 0 Multiply both sides by (x2 − x − 6)2
2x − 1 = 0 Divide both sides by −6
1
x= Solve for x
2
So, the critical number for f is 12 . Evaluating f at the critical number, we get
1 6 6 6 −4 24
f = 1 2 1 = 1 2 24 = 25 = 6 =−
2 (2) − 2 − 6 4
−4− 4 −4 25 25
+ + − −
−∞ −2 1 3 ∞
2
choose −3 choose 0 choose 1 choose 4
f 0 (−3) = 7
6
>0 f 0 (0) > 0 f 0 (1) < 0 f 0 (4) = − 76 < 0
This shows that f is increasing on the intervals (−∞, −2) and (−2, 21 ), and
decreasing on the intervals ( 12 , 3) and (3, ∞). The relative maximum of f (x) is
f ( 12 ) = − 25
24
. There is no relative minimum.
(e) Concavity and Inflection Points.
The second derivative is
00 d 0 d −12x + 6
f (x) = [f (x)] =
dx dx (x2 − x − 6)2
300 Chapter 5. Applications of the Derivative I
d
−12(x2 − x − 6)2 − (−12x + 6)2(x2 − x − 6) dx [x2 − x − 6]
= Quotient Rule
(x2 − x − 6)4
−12(x2 − x − 6)2 − (−12x + 6)2(x2 − x − 6)(2x − 1)
=
(x2 − x − 6)4
(x2 − x − 6) [−12(x2 − x − 6) − (−12x + 6)(4x − 2)]
= Factor out x2 − x − 6
(x2 − x − 6)4
(x2 − x − 6) (−12x2 + 12x + 72 + 48x2 − 48x + 12)
=
(x2 − x − 6)4
(x2 − x − 6)(36x2 − 36x + 84)
=
(x2 − x − 6)4
12(3x2 − 3x + 7)
= Simplify
(x2 − x − 6)3
Set f 00 (x) = 0. Then
12(3x2 − 3x + 7)
=0
(x2 − x − 6)3
12(3x2 − 3x + 7) = 0 Multiply both sides by (x2 − x − 6)3
3x2 − 3x + 7 = 0 Divide both sides by 12
√
−b± b2 −4ac
Using the quadratic formula we have
2a
,
p √
−(−3) ± (−3)2 − 4(3)(7) 3 ± −75
x= =
2(3) 6
Since the square root of −75 is undefined, the equation f 00 (x) = 0 has no solution.
This implies that f has no inflection point.
The sign of f 00 (x) is shown in the following figure.
concave up concave down concave up
+ − +
−∞ −2 3 ∞
choose −3 choose 0 choose 4
f 00 (−3) > 0 f 00 (0) < 0 f 00 (4) > 0
(f ) Summary.
−2 3 x
6
Figure 5.39: Graph of f (x) = x2 −x−6
302 Chapter 5. Applications of the Derivative I
Chapter6
y
Absolute
2 maximum
−1 3 x
−2
Absolute
minimum
Figure 6.1
Absolute Extrema–Definition
Let f be a function and I be an interval. Suppose f is defined on I, and let c
be a number in I.
303
304 Chapter 6. Applications of the Derivative II
For the function whose graph is shown in Figure 6.1, the absolute maximum is 2,
and the absolute minimum is −2.
(a) Solution.
Step 1. Find the critical numbers of f on the open interval (a, b).
x 0 2 5
f (x) 12 16 7
Table 6.1
Evaluating f and the endpoints 0 and 5 of the interval [0, 5], we get
We summarize the values of f in Table 6.1. From Table 6.1, the absolute
maximum is 16 and it occurs when x = 2. And the absolute minimum
is 7, and it occurs when x = 5.
(b) Solution.
The derivative of f (x) = −x3 + 3x2 + 1 is
x −1 0 2
f (x) 5 1 5
Table 6.2
x −2 −1 1
f (x) −3 8 −12
Table 6.3
(−4)2 − 4 16 − 4 12 3
f (−4) = 2
= = = = f (4).
4 +4 16 + 4 20 5
We summarize these values in Table 6.4.
x −4 0 4
3 3
f (x) 5
−1 5
Table 6.4
6x = 0 or (x2 − 4)2 = 0
x=0 or x2 − 4 = 0
x=0 or (x + 2)(x − 2) = 0
x=0 or x = −2 or x = 2
Evaluating f at the critical numbers in the open interval (−2, 3), we get
f (0) = (02 − 4)3 = (−4)3 = −64 and f (2) = ((2)2 − 4)3 = (4 − 4)3 = 0.
x −2 0 2 3
f (x) 0 −64 0 125
Table 6.5
From Table 6.5, the absolute maximum is 125, and it occurs when x = 3.
The absolute minimum is −64, and it occurs when x = 0.
(f ) Solution.
The derivative of f (x) = x
x2 −x+1
is
(1)(x2 − x + 1) − x(2x − 1)
f 0 (x) = Quotient Rule
(x2 − x + 1)2
−x2 + 1 −(x2 − 1) −(x − 1)(x + 1)
= 2 2
= 2 2
=
(x − x + 1) (x − x + 1) (x2 − x + 1)2
x 0 1 3
3
f (x) 0 1 7
Table 6.6
(g) Solution.
8 2
The derivative of f (x) = 3x 3 − 108x 3 is
0 8 5 2 −1 d n
f (x) = 3 x 3 − 108 x 3 [x ] = nxn−1
3 3 dx
5 1
= 8x 3 − 72x− 3
5 72 1
= 8x 3 − 1 x−n =
x 3 xn
2
8x − 72 b ac−b
= 1 a− c
= c
, am an = am+n
x 3
8(x2 − 9)
= 1 Factor out 8
x3
8(x − 3)(x + 3)
= 1 a2 − b2 = (a − b)(a + b)
x3
5
!
5 1 1 8x 3
f 0 (x) = 8x 3 − 72x− 3 = x− 3 1 − 72
x− 3
1 am
= x− 3 8x2 − 72 = am−n
an
8(x2 − 9)
= 1
x3
8(x − 3)(x + 3)
= 1
x3
1
Set f 0 (x) = 0. Then 8(x+3)(x−3)
1 = 0. Multiplying both sides by x 3 , we get
x3
8(x + 3)(x − 3) = 0. This implies that x = −3 or x = 3. But −3 does not lie
in (−2, 8), so we ignore it. So 3 is a critical number in (−2, 8). Also, since
the derivative is undefined at x = 0 and f (0) is defined, it follows that 0 is
another critical number.
Evaluating f at the critical numbers, we get f (0) = 0, and
8 2
f (3) = 3(3) 3 − 108(3) 3 ≈ 56.162 − 224.649 = −168.487
6.1. Absolute Extrema 309
and
8 2
f (8) = 3(8) 3 − 108(8) 3 = 768 − 432 = 336
These values are summarized in Table 6.7.
x −2 0 3 8
f (x) −152.39 0 −168.49 336
Table 6.7
From Table 6.7, the absolute maximum is 336, and it occurs when x = 8.
The absolute minimum is approximately −168.49, and it occurs when
x = 3.
(a) Solution.
The derivative of C(x) = x3 − 3x + 100 is
C(0) = 100 and C(20) = 203 − 3(20) + 100 = 8000 − 60 + 100 = 8040.
x 0 1 20
C(x) 100 98 8040
Table 6.8
+ −
0 350 ∞
choose 1 choose 400
R0 (1) = 53397 > 0 R0 (400) = −67500 < 0
Figure 6.2 tells us that the production level x = 350 will yield a maximum
revenue.
No fence is needed
x x
Figure 6.3
Step 4. Express the quantity you want to optimize in terms of the other variables
from Step 3.
Here we have A = xy, which is the formula for the area of a rectangle of
sides x and y.
Step 5. If the quantity you want to optimize depends on more than one variable,
use the given information to write it as a function of one variable.
Here, from the problem, the total length of the fencing is 2000 ft,
that is, x + x + y = 2000 or 2x + y = 2000. (Remember that from the
problem no fence is needed along the river. This is why the variable y
appears only once.) The equation 2x + y = 2000 is called constraint.
From the constraint we have y = 2000 − 2x. Substituting this into
the expression for A, we get
Step 6. Find the interval and the absolute maximum (or absolute minimum) of
the function found in Step 5. This can be done by using the First Deriva-
tive Test or the Second Derivative Test. Or, if the interval is closed, one
could use the method of Section 6.1.
Here the largest x can be is 1000 (this uses all the fence for the depth and
none for the width). Moreover, x is a nonnegative number. So the interval
is [0, 1000]. From now on, our goal is to find the absolute maximum of
the function A(x) = 2000x − 2x2 on the interval [0, 1000].
Finding the critical numbers. The derivative is A0 (x) = 2000 − 4x.
Set A0 (x) = 0. Then 2000 − 4x = 0. Subtracting 2000 from both
sides, we get −4x = −2000. Dividing both sides by −4, we get
x = 500.
Evaluating A(x) at 500, we have
Conclusion. The dimensions of the field that maximize the area are x = 500 ft
and y = 1000 ft.
3. [Exercise on page 46]
Solution. Consider the same picture as before (see Figure 6.3). The quantity we
want to minimize is the cost, which we denote by C.
Function. Since fencing costs $27 per meter, and since the landlord will pay
half of the cost of one of the sides perpendicular to the river, it follows that
27x 54x 27x
C = 27x + 27y + = + 27y +
2 2 2
54x + 27x 81
= + 27y = x + 27y
2 2
To express C as a function of one variable, we use the fact that the area is 1536,
that is, xy = 1536. Dividing both sides of the latter equation by x, we get
y = 1536
x
. Substituting this into the formula for the cost, we get
81 1536 81 41472
C = x + 27 = x+
2 x 2 x
81 41472
So the function we want to minimize is C(x) = 2
x + x
Interval. Since x can’t be 0 here (otherwise the area would be 0 and this would
contradict the fact that the area is 1536), and since there is no maximum value
for x, the interval for x is the open interval (0, ∞).
Critical numbers. The first derivative is
0 d 81 41472 d 81 d 41472
C (x) = x+ = x + Sum Rule
dx 2 x dx 2 dx x
81 41472 d 1 1
= − =− 2
2 x2 dx x x
81 41472
− =0
2 x2
81 41472
=
2 x2
81x2 = 41472(2) Multiply both sides by 2x2
41472(2)
x2 = Divide both sides by 81
81
2
x = 1024
√
x = ± 1024 = ±32
Since x can’t be negative, the critical number −32 is to be rejected. So, the only
critical number is 32.
6.2. Optimization Problems 313
Since C 0 (32) = 0 and C 00 (32) > 0, the function C has a relative minimum at
32. That local minimum is actually the absolute minimum since C 00 (x) > 0 on
the interval (0, ∞) (which means that C is concave upward on (0, ∞)). So the
absolute minimum cost occurs when x = 32.
− +
0 32 ∞
choose 1 choose 33
C 0 (1) = −41431.5 < 0 C 0 (33) ≈ 2.417 > 0
From this figure, the cost function is decreasing on the interval (0, 32) and
increasing on (32, ∞). So the absolute minimum is obtained when x = 20.
Hence, the dimensions of the field that will minimize the cost to the farmer are x = 32
m and y = 48 m.
A = 2πr2 + 2πrh
314 Chapter 6. Applications of the Derivative II
The constraint states that the volume must be equal to 2500, that is, πr2 h =
2500, or h = 2500
πr2
. Substituting this into the expression for A, we get
2 2500 2 5000
A = 2πr + 2πr = 2πr +
πr2 r
Since the radius r can’t be negative, and since r 6= 0 (otherwise, the volume
would be 0, and this would contradict the fact that the volume is 2500), we have
r > 0. So the interval for r is (0, ∞).
We now find the critical numbers. The first derivative is Setting this equal to 0,
we get 4πr3 − 5000 = 0, that is, r3 = 5000
4π
or r3 = 1250
π
. Taking the cube root of
q
both sides, we find r = 3 1250
π
, which is the only critical number.
Using the same rules as before, the second derivative is
00 d 0 d 5000 d d 5000 10000r
A (r) = [A (r)] = 4πr − 2 = [4πr] − = 4π +
dr dr r dr dr r2 r4
Since this is positive for every r in (0, ∞), theqfunction A is concave upward
and therefore the (only) critical number r = 3 1250 π
gives rise to an absolute
minimum.
q
The value of h corresponding to r = 3 1250π
is
1 13
2(1250) 2(1250) 3 1250
= 2 1 = 1 =2 = 2r
(1250) 3 π 3 π3 π
Thus, to minimize the total surface area, the radius and height of the cylinder should
be
1 1
1250 3 1250 3
r= ≈ 7.355 cm and h=2 ≈ 14.71 cm
π π
Let x represent the length of a side of the base (which is a square), and let y
denote the height of the box as shown in Figure 6.4.
Then V = x×x×y = x2 y. (Recall that the volume of a box of width a, length b,
and height c is given by the formula V = abc.) From the problem, the constraint
is that the total surface is 1875, that is, the sum of areas of all faces (except the
top) is equal to 1875, or
x2 + xy + xy + xy + xy = 1875,
6.2. Optimization Problems 315
Figure 6.4
2
that is, x2 + 4xy = 1875. Solving this for y, we get y = 1875−x
4x
. Substituting
this into V , we get
1875 − x2 x(1875 − x2 ) 1875x − x3
2 1
V =x = = = (1875x − x3 )
4x 4 4 4
1875−x2
Using the equation y = 4x
, we find
1875 − (25)2 1875 − 625 1250
y= = = = 12.5
4(25) 100 100
So, the largest possible volume of the box is
V = x2 y = (25)2 (12.5) = (625)(12.5) = 7812.5 cm3
Setting this equal to 0, wet get 36x3 − 18432 = 0. Adding 18432 to both
sides, we get 36x3 = 18432. Dividing both sides by√36, we get x3 = 512.
Taking now the cube root of both sides, we get x = 3 512 = 8, which is the
only critical number.
The second derivative is
00 d 18432 d d 18432 36864
C (x) = 36x − 2
= [36x] − 2
= 36 +
dx x dx dx x x3
Since C 0 (8) > 0, it follows by the Second Derivative Test that the relative
minimum of C(x) occurs when x = 8. Actually, C(8) is the absolute mini-
mum because the function C is concave upward on the interval (0, ∞) (since
C 00 (x) > 0 for all x > 0).
Thus, the value x = 8 is the one that should be used for the cheapest cost.
(a) Solution.
6.3. Implicit Differentiation and Related Rates 317
y − x3 = 0, xy − 2x = 1, x3 y 3 − x2 y + 1 = 0,
d n d dy
[y ] = [y n ] · Chain Rule
dx dy dx
dy
= ny n−1 Power Rule
dx
So, for every real number n,
d n dy
[y ] = ny n−1
dx dx
Warning. In general, the derivative of y n with respect to x is not
d
equal to ny n−1 , that is, dx [y n ] 6= ny n−1 .
dy
We want to find dx
for the implicit function x2 + y 2 = 25.
x2 + y 2 = 25
d 2 d
x + y2 =
[25] Take the derivative of both sides
dx dx
d 2 d
[x ] + [y 2 ] = 0 Sum Rule, Constant Rule
dx dx
dy d n d n dy
2x + 2y 2−1 =0 [x ] = nxn−1 , [y ] = ny n−1
dx dx dx dx
dy
2x + 2y =0
dx
318 Chapter 6. Applications of the Derivative II
dy
2y = −2x Subtract 2x from both sides
dx
dy −2x
= Divide both sides by 2y
dx 2y
dy x
=− Simplify
dx y
(b) Solution.
x3 + y 3 = 1
d 3 d
[x + y 3 ] = [1] Take the derivative of both sides
dx dx
d 3 d
[x ] + [y 3 ] = 0 Sum Rule, Constant Rule
dx dx
dy d n d n dy
3x2 + 3y 2 =0 [x ] = nxn−1 , [y ] = ny n−1
dx dx dx dx
dy
3y 2 = −3x2 Subtract 3x2 from both sides
dx
dy −3x2 −x2
= = 2 Divide both sides by 3y 2
dx 3y 2 y
(c) Solution.
2x2 − y 2 = x
d d
[2x2 − y 2 ] = [x] Take the derivative of both sides
dx dx
d d
[2x2 ] − [y 2 ] = 1
dx dx
dy d n dy
4x − 2y =1 [y ] = ny n−1
dx dx dx
dy
−2y = 1 − 4x
dx
dy 1 − 4x 4x − 1
= =
dx −2y 2y
(d) Solution.
x4 + 3y 3 = 5y
d 4 d
[x + 3y 3 ] = [5y] Take the derivative of both sides
dx dx
d 4 d dy
[x ] + 3 [y 3 ] = 5
dx dx dx
dy dy d n dy
4x3 + 9y 2 =5 [y ] = ny n−1
dx dx dx dx
6.3. Implicit Differentiation and Related Rates 319
dy dy
9y 2 −5 = −4x3
dx dx
dy
(9y 2 − 5) = −4x3 Factor the left hand side
dx
dy −4x3
=
dx 9y 2 − 5
(e) Solution.
d √ d 2
[ x − y] = [y + 3] Take the derivative of both sides
dx dx
d √ d d 2 d
x − [y] = [y ] + [3]
dx dx dx dx
1 dy dy d √ 1 d n dy
√ − = 2y +0 x = √ , [y ] = ny n−1
2 x dx dx dx 2 x dx dx
dy dy 1
− − 2y =− √
dx dx 2 x
dy 1
(−1 − 2y) =− √
dx 2 x
dy − 2√1 x −1 a
a
b
= = √ =
dx −1 − 2y 2 x(−1 − 2y) c bc
1
= √
2 x(1 + 2y)
(f ) Solution.
d d
[xy] = [5] Take the derivative of both sides
dx dx
d d
[x]y + x [y] = 0 Product Rule, Constant Rule
dx dx
dy
(1)y + x =0
dx
dy
x = −y
dx
dy −y
=
dx x
(g) Solution.
d d
[3x2 + 2xy + y 2 ] = [2] Take the derivative of both sides
dx dx
d d d
3 [x2 ] + 2 [xy] + [y 2 ] = 0
dx dx dx
dy dy d n dy
6x + 2 y + x + 2y =0 [y ] = ny n−1
dx dx dx dx
320 Chapter 6. Applications of the Derivative II
dy dy
6x + 2y + 2x + 2y =0
dx dx
dy
(2x + 2y) = −6x − 2y
dx
dy −6x − 2y −3x − y
= =
dx 2x + 2y x+y
(h) Solution.
(i) Solution.
d 3 2 d Take the derivative of
[x y + y 4 ] = [x]
dx dx both sides
d 3 2 d
[x y ] + [y 4 ] = 1
dx dx
d 3 2 d d
[x ]y + x3 [y 2 ] + [y 4 ] = 1 Product Rule
dx dx dx
d 3 2 d dy d n dy
[x ]y + x3 [y 2 ] + 4y 3 =1 [y ] = ny n−1
dx dx dx dx dx
dy dy
3x2 y 2 + x3 (2y) + 4y 3 =1
dx dx
dy dy
2x3 y + 4y 3 = 1 − 3x2 y 2
dx dx
dy
(2x3 y + 4y 3 ) = 1 − 3x2 y 2 Factor the left hand side
dx
dy 1 − 3x2 y 2
=
dx 2x3 y + 4y 3
6.3. Implicit Differentiation and Related Rates 321
(j) Solution.
d x+y
d Take the deriva-
= [1] tive of both sides
dx x − y dx
d d
dx
[x + y](x − y) − (x + y) dx [x − y]
2
=0 Quotient Rule
(x − y)
d d Multiply both sides
[x + y](x − y) − (x + y) [x − y] =0
dx dx by (x − y)2
dy dy
1+ (x − y) − (x + y) 1 − =0
dx dx
dy dy dy dy
x−y+x −y − x+y−x −y =0 Distribute
dx dx dx dx
dy dy dy dy
x−y+x −y −x−y+x +y =0
dx dx dx dx
dy
−2y + 2x =0 Simplify
dx
dy
2x = 2y
dx
dy 2y y
= =
dx 2x x
(k) Solution.
d y d
[e ] = [xy] Take the derivative of both sides
dx dx
dy d d y dy
ey = [xy] [e ] = ey
dx dx dx dx
dy d d
ey = [x]y + x [y] Product Rule
dx dx dx
dy dy
ey =y+x
dx dx
dy dy
ey −x =y
dx dx
dy
(ey − x) =y Factor the left hand side
dx
dy y
= y
dx e −x
(l) Solution.
d d
[yex − y] = [x] Take the derivative of both sides
dx dx
d dy
[yex ] − =1
dx dx
322 Chapter 6. Applications of the Derivative II
d d dy
[y]ex + y [ex ] − =1 Product Rule
dx dx dx
dy x dy d x
e + yex − =1 [e ] = ex
dx dx dx
dy x dy
e − = 1 − yex
dx dx
dy
(ex − 1) = 1 − yex Factor the left hand side
dx
dy 1 − yex
=
dx ex − 1
(m) Solution.
d xy3 d Take the derivative of
[e ] = [2x + 3y + 1]
dx dx both sides
3 d d d g(x) d
exy [xy 3 ] = [2x + 3y + 1] e = eg(x) [g(x)]
dx dx dx dx
3 d d d d
exy [xy 3 ] = 2 [x] + 3 [y] + [1]
dx dx dx dx
3 d dy
exy [xy 3 ] = 2 + 3 + 0
dx dx
3 d d dy
exy [x]y 3 + x [y 3 ] = 2 + 3 + 0 Product Rule
dx dx dx
xy 3 3 2 dy dy
e y + 3xy =2+3
dx dx
3 dy3 dy
y 3 exy + 3xy 2 exy =2+3
dx dx
3 dy dy 3
3xy 2 exy −3 = 2 − y 3 exy
dx dx
3
dy 3
3xy 2 exy − 3 = 2 − y 3 exy Factor the left hand side
dx
3
dy 2 − y 3 exy
=
dx 3xy 2 exy3 − 3
(n) Solution.
d d 3 3 Take the derivative of
[x + ln y] = [x y ]
dx dx both sides
d d d 3 3 d
[x] + [ln y] = [x ]y + x3 [y 3 ]
dx dx dx dx
d dy
d [y] d 3 3 d d
[x] + dx = [x ]y + x3 [y 3 ] [ln y] = dx
dx y dx dx dx y
dy
2 3 3 2 dy d n dy
1+ dx
= 3x y + x 3y [y ] = ny n−1
y dx dx dx
6.3. Implicit Differentiation and Related Rates 323
1 dy dy
1+ = 3x2 y 3 + 3x3 y 2
y dx dx
1 dy dy
− 3x3 y 2 = 3x2 y 3 − 1
y dx dx
1 dy
− 3x3 y 2 = 3x2 y 3 − 1 Factor the left hand side
y dx
1 − 3x3 y 3 dy
a a − bc
= 3x2 y 3 − 1 −c=
y dx b b
dy (3x2 y 3 − 1)y y
= Multiply both sides by
dx 1 − 3x3 y 3 1 − 3x3 y 3
x2 − xy − y 2 = 1
d 2 d Take the derivative of
[x − xy − y 2 ] = [1]
dx dx both sides
d 2 d d
[x ] − [xy] − [y 2 ] = 0
dx dx dx
dy d
2x − y + x − [y 2 ] = 0 Product Rule
dx dx
dy dy d n dy
2x − y + x − 2y =0 [y ] = ny n−1
dx dx dx dx
dy dy
2x − y − x − 2y =0
dx dx
dy
(−x − 2y) = −2x + y
dx
dy −2x + y
=
dx −x − 2y
dy
• To get the slope at (2, 1), we substitute x with 2 and y with 1 into dx
:
−2(2) + 1 −4 + 1 −3 3
m= = = =
−(2) − 2(1) −2 − 2 −4 4
• The equation of the tangent line to the curve x2 − xy − y 2 = 1 at (2, 1) is
3
y − 1 = (x − 2)
4
3 3 Distribute the right
y−1= x−
4 2 hand side
3 1
y = x− Add 1 to both sides
4 2
324 Chapter 6. Applications of the Derivative II
(b) Solution.
dy
• First we need to find the derivative dx
. Using implicit differentiation, we
have
(x2 + y 2 )2 = 2(x3 + y 2 )
x2 − y 2 = 4
6.3. Implicit Differentiation and Related Rates 325
d 2 d
[x − y 2 ] = [4] Take the derivative of both sides
dx dx
d 2 d
[x ] − [y 2 ] = 0
dx dx
dy d n dy
2x − 2y =0 [y ] = ny n−1
dx dx dx
dy
−2y = −2x
dx
dy −2x x
= =
dx −2y y
• We now find the second derivative.
00 d dy d x
y = =
dx dx dx y
d d
dx
[x]y − x dx [y]
= Quotient Rule
y2
dy
y − x dx
=
y2
y − x xy dy x
= =
y2 dx y
x2 y 2 −x2
y− y y y 2 − x2
= = =
y2 y2 y3
4. [Exercise on page 47]
Solution.
We first need to find the marginal cost using implicit differentiation.
√
C 2 − 40 = x2 + 80 x
d 2 d 2 √
C − 40 = x + 80 x Take the derivative of both sides
dx dx
d 2 d d 2 d √
[C ] − [40] = [x ] + 80 x
dx dx dx dx
dC 1 d 2 dC d √ 1
2C − 0 = 2x + 80 √ [C ] = 2C , x = √
dx 2 x dx dx dx 2 x
dC 2x + √40x
=
dx 2C
√ a
2x x + 40 b ac + b a
= √ = , b =
a+
2C x c c c bc
√
When x = 4, the equation C 2 − 40 = x2 + 80 x becomes
√
C 2 = 42 + 80 4 + 40 = 16 + 160 + 40 = 216
√
This implies that C = 216.
326 Chapter 6. Applications of the Derivative II
√
Substituting x with 4 and C with 216 into dC
dx
, we get
√
dC 2(4) 4 + 40
= √ √ ≈ 0.95
dx 2( 216) 4
So the marginal cost at x = 4 is 0.95.
The interpretation is that at the level of production of 4 items, the cost
of producing one additional item is approximately 0.95.
(a) Solution.
Rate of Change
If y is a function of time t, the rate of change of y with respect to t
is dy
dt
. This can also be interpreted as the speed/rate at which y is
changing.
d n dx
[x ] = nxn−1
dt dt
dR
= 80(5) − 0.5(100)(5) = 400 − 250 = $150/day.
dt
(b) Solution. The rate of change of cost with respect to time is
dC d dx d dx
= [7x + 20] = 7 + [20] = 7 + 0 = 7(5) = $35/day
dt dt dt dt dt
(c) Solution. Let P be the profit. Since P = R − C, the rate of change of P is
dP d dR dC
= [R − C] = − = 150 − 35 = $115/day
dt dt dt dt
6.3. Implicit Differentiation and Related Rates 327
Given: dx
dt
= 10 units/month Unknown: dC
dt
when x = 95 units.
The rate of change of C is
dC d d d
= [0.3x2 + 7000] = 0.3 [x2 ] + [7000]
dt dt dt dt
dx d n dx
= 0.3 2x +0 [x ] = nxn−1
dt dt dt
dx
= 0.6x
dt
dx
= 0.6(95)(10) = $570/month x = 95, = 10
dt
Given: dC
dt
= $10/month Unknown: dR
dt
when R = $20000.
R2
To find dR
dt
, we first differentiate both sides of the equation C = 300000
+ 8000
with respect to t.
R2
C= + 8000
300000
R2
dC d
= + 8000
dt dt 300000
R2
dC d d
= + [8000]
dt dt 300000 dt
dC 1 d 2
= [R ] + 0
dt 300000 dt
dC 1 dR d 2 dR
= 2R [R ] = 2R
dt 300000 dt dt dt
dC R dR
=
dt 150000 dt
Multiplying both sides of the latter equation by 150000, we get
dC dR
150000 =R
dt dt
Dividing both sides by R, we get
dR 150000 dC 150000
= = (10) = $75/month
dt R dt 20000
328 Chapter 6. Applications of the Derivative II
In a related rates problem, two quantities (or variables) are involved, the rate
of change of one quantity is given, and the rate of change of the other quantity
is to be found. To solve such problems, one can go through the following steps.
Step 2. Write the given rate of change and the rate of change that is to be
found.
Step 4. Take the derivative of both sides of the equation with respect to
time t.
6.3. Implicit Differentiation and Related Rates 329
Step 5. Solve for the unknown rate of change and substitute the given in-
formation.
Here the quantities that are functions of time are the volume and the radius.
Let V represent the volume, and let r represent the radius.
dr dV 6
Given: = 0.5 cm/s Unknown: when r = = 3 cm.
dt dt 2
Equation that relates V and r: we know that the volume of a sphere of radius
r is given by the formula V = 34 πr3 .
Differentiating both sides of the equation V = 34 πr3 with respect to time t, we
get
dV d 4 3 4 d 4 dr d n dr
= πr = π [r3 ] = π(3r2 ) [r ] = nrn−1
dt dt 3 3 dt 3 dt dt dt
dr
= 4πr2 = 4π(3)2 0.5 = 18π cm3 /s
dt
dV dD
Given: = 180 cm3 /s Unknown: when R = 75 cm
dt dt
The volume is given by
4
V = πR3
3
3
4 D D
= π R=
3 2 2
4 D3
= π
3 8
1
V = πD3
6
dD
Solving this latter equation for dt
, we get
dD 2 dV
=
dt πD2 dt
2 360 dV
= 2
(180) = D = 2R = 2(75) = 150, = 180
π(150) 22500π dt
2
= cm/s
125π
dV dh
Given: = 4 m3 /min Unknown: .
dt dt
We know that the volume of a cylinder with radius r and height h is given by the
formula V = πr2 h. Differentiating both sides with respect to time t, we get
dV d dh
= [πr2 h] = πr2
dt dt dt
dV
So dt
= πr2 dh
dt
. Solving this latter equation for dh
dt
, we get
dh 1 dV
= 2
dt πr dt
dV
Substituting r with 7 and dt
with 4, we get
dh 1 4
= 2
(4) = m/min
dt π(7) 49π
x
drone at some instant
0.06 z
observer
Figure 6.5
dx dz
Given: = 90 mi/h Unknown: when z = 0.1 mi.
dt dt
6.4. Elasticity of Demand 331
(a) Solution.
Elasticity of Demand
Suppose that the demand q and the price p are related by an equation.
The elasticity of demand, denoted E, is given by
p dq
E=−
q dp
If E < 1, demand is inelastic. This means that demand will not
change much if there is a slight price change.
We want to find the elasticity of demand for the equation q = −0.05p + 8.5 when
p = $102.
First, we need to find dq
dp
.
dq d d d
= [−0.05p + 8.5] = −0.05 [p] + [8.5]
dp dp dp dp
332 Chapter 6. Applications of the Derivative II
= −0.05(1) + 0 = −0.05
The elasticity is
p dq p 0.05p
E=− =− (−0.05) =
q dp −0.05p + 8.5 −0.05p + 8.5
When p = $102,
0.05(102) 5.1
E= = = 1.5
−0.05(102) + 8.5 3.4
dq d d
= [10000] − 30 [p2 ] = 0 − 30(2p) = −60p
dp dp dp
The elasticity is
p dq p
E=− =− (−60p)
q dp 10000 − 30p2
60p2 60(10)2
= = Replace p with 10
10000 − 30p2 10000 − 30(10)2
6000
= ≈ 0.857
7000
dq d d −0.1p
100e−0.1p = 100
= e Constant Multiple Rule
dp dp dp
−0.1p d d g(p) d
= 100 e [−0.1p] e = eg(p) [g(p)]
dp dp dp
= 100 e−0.1p (−0.1) = −10e−0.1p
The elasticity is
p dq p
−10e−0.1p
E=− =− −0.1p
q dp 100e
p 10
= = =1 Replace p with 10
10 10
Interpretation. Since E = 1, the demand has unit elasticity.
dq
(a) Solution. We first need to find dp
.
dq d 2 d 1 1 −2
= =2 =2 − 2 = 2
dp dp p dp p p p
We now calculate E.
p2
p dq p −2 −2
E=− = −2 =− =1
q dp p
p2 2 p2
dq d 5−0.3 ln p d d g(p) d
= e = e5−0.3 ln p [5 − 0.3 ln p] e = eg(p) [g(p)]
dp dp dp dp dp
d d 1 d 1
= e5−0.3 ln p [5] − 0.3 [ln p] = e5−0.3 ln p 0 − 0.3 [ln p] =
dp dp p dp p
1 0.3
= e5−0.3 ln p 0 − 0.3 = − e5−0.3 ln p
p p
The elasticity is
p dq p 0.3 5−0.3 ln p
E=− = − 5−0.3 ln p − e
q dp e p
= −(−0.3) = 0.3 Simplify
dq d d −1.68
15000p−1.68 = 15000
= p Constant Multiple Rule
dp dp dp
d −1.68 ln p
= 15000 e AB = eB ln A
dp
d d g(p) d
= 15000e−1.68 ln p [−1.68 ln p] e = eg(p) [g(p)]
dp dp dp
1 −25200 −1.68 ln p
= 15000e−1.68 ln p −1.68 = e
p p
The elasticity is
p dq p −25200 −1.68 ln p
E=− − e
q dp 15000e−1.68 ln p p
25200
= = 1.68 Simplify
15000
Interpretation. Since E > 1, the demand is elastic.
334 Chapter 6. Applications of the Derivative II
(a) Solution. Since q is given implicitly in terms of p, we will use implicit differ-
dq
entiation to find dp .
dq
If p is given explicitly in terms of q, to get dp
, one can use the fact that
dq 1
= dp
dp dq
dq
to find dp
. For the equation p = 0.604q 2 − 20.16q + 263.067,
dq 1 1
= dp = d
dp dq dq
[0.604q 2 − 20.16q + 263.067]
1
= d d d
0.604 dq [q 2 ] − 20.16 dq [q] + dq
[263.067]
1 1
= =
0.604(2q) − 20.16(1) + 0 1.208q − 20.16
The elasticity is
p dq
E=−
q dp
0.604q 2 − 20.16q + 263.067
1
=−
q 1.208q − 20.16
−(0.604q 2 − 20.16q + 263.067)
=
q(1.208q − 20.16)
6.4. Elasticity of Demand 335
Step 2. Select a test number p1 such that p1 < p0 and f (p1 ) ≥ 0, and
substitute it into the expression for E to see if E < 1 or E > 1
when p < p0 .
2p2
Consider the expression E = 108−p2
from part (a).
Solving the equation E = 1.
E=1
2p2
=1
108 − p2
2p2 = 108 − p2 Multiply both sides by 108 − p2
3p2 = 108 Add p2 to both sides
p2 = 36 Divide both sides by 3
√
p ± 36 = ±6 Take square root of both sides
Since the price is positive, we reject the negative value. So the solution to
the equation E = 1 is p0 = 6.
Select a test number p less than 6. Take for example p = 5. Substituting
this into E, we get
2(5)2 50
E= 2
= <1
108 − (5) 83
So the demand is inelastic when p < 6.
Select a test number p greater than 6. Take for example p = 7. Substituting
this into E, we get
2(7)2 98
E= 2
= >1
108 − (7) 59
So the demand is elastic when p > 6.
6.4. Elasticity of Demand 337
So, if the demand is inelastic (that is, E < 1), then 1 − E > 0, and
therefore dR
dp
> 0. This implies that total revenue increases as price
increases when E < 1.
Similarly, if the demand is elastic (that is, E > 1), total revenue
decreases as price increases.
Total revenue
+ −
0 E<1 E=1 E>1 p
dR dR
dp
>0 dp
<0
Since total revenue is maximized at the price where demand has unit elasticity,
and since E = 1 when p = 6 (see part (b)), it follows that the price p = $6
results in the maximum revenue. The demand at this price is
q = 108 − p2 = 108 − (6)2 = 72
The maximum revenue is then
R = pq = 6(72) = $432
The elasticity is
p dq −p 4 4
E=− =− − =
q dp 16 − 4 ln p p 16 − 4lnp
(b) To find the price intervals where demand is elastic and where demand is inelastic,
we first need to solve the equation E = 1 for p.
E=1
4
=1
16 − 4 ln p
4 = 16 − 4 ln p Multiply both sides by 16 − 4 ln p.
−12 = −4 ln p Subtract 16 from both sides
3 = ln p Divide both sides by −4.
e3 = p Take the natural exponential of both sides
4 4
E(20) = ≈ <1
16 − 4 ln 20 4.017
4 4
E(21) = ≈ >1
16 − 4 ln 21 3.822
q = 16 − 4 ln p = 16 − 4 ln(e3 ) = 16 − 4(3) = 4
(a) y = x2
Solution.
6.5. Differentials and Linear Approximations 339
Differentials
Let y = f (x) be a function.
dy = f 0 (x)dx
where f 0 (x) is the derivative of f (x). In words, this says that the
differential of y is f 0 (x) multiplied by dx. The geometric meaning of
dy is shown below.
Note that dy depends on two independent variables, namely x and
dx.
dy = f 0 (x)dx = 3x2 dx
(c) Solution. Let f (x) = 4x3 − 5x2 + x − 8. To find the differential of y = f (x),
we first need to find the derivative of f .
d 3 d d d d
f 0 (x) = 4x − 5x2 + x − 8 = [4x3 ] − [5x2 ] + [x] − [8]
dx dx dx dx dx
2 2
= 4(3x ) − 5(2x) + 1 − 0 = 12x − 10x + 1
5 1
(d) Solution. Let f (x) = 3x 4 + x 4 . The derivative of y = f (x) is
d h 5i d h 1i d h 5i d h 1i
f 0 (x) = 3x 4 + x4 = 3 x4 + x4
dx dx dx dx
5 5 −1 1 1 −1 5 1 1 3 d n
=3 x4 + x4 = 3 x 4 + x− 4 [x ] = nxn−1
4 4 4 4 dx
15 1 1 3
= x 4 + x− 4
4 4
So the differential of y is
0
15 1 1 3
−4
dy = f (x)dx = x + x
4 dx
4 4
1
dy = f 0 (x)dx = √ dx
2 x+3
0 d h√ 2 i d
dx
[4x2 + 13] d hp i d
dx
[g(x)]
f (x) = 4x + 13 = √ g(x) = p
dx 2 4x2 + 13 dx 2 g(x)
8x 4x
= √ =√
2
2 4x + 13 4x2 + 13
The differential is
4x
dy = f 0 (x)dx = √ dx
4x2 + 13
When x = 3 and dx = 0.04, this becomes
4(3) 12
dy = p (0.04) = (0.04) ≈ 0.0686
2
4(3) + 13 7
6.5. Differentials and Linear Approximations 341
15x+6
(c) Solution. Let f (x) = −x+2
. Then
d d
[15x + 6](−x + 2) − (15x + 6) dx [−x + 2]
f 0 (x) = dx
Quotient Rule
(−x + 2)2
15(−x + 2) − (15x + 6)(−1) −15x + 30 + 15x + 6 36
= 2
= 2
=
(−x + 2) (−x + 2) (−x + 2)2
The differential of y = f (x) is
36
dy = f 0 (x)dx = dx
(−x + 2)2
Substituting x with −2 and dx with 0.03 into this, we get
36 36 36
dy = 2
(0.03) = 2
(0.03) = (0.03) = 0.0675
(−(−2) + 2) (2 + 2) 16
x
(d) Solution. Let f (x) = e 8 . Then
d x x d
hxi d g(x) d
f 0 (x) = e8 = e8 e = eg(x) [g(x)]
dx dx 8 dx dx
x 1 d x 1 1 x
= e8 [x] = e 8 (1) = e 8
8 dx 8 8
(a) Solution.
∆x = x2 − x1
dx = ∆x
342 Chapter 6. Applications of the Derivative II
∆y = f (x2 ) − f (x1 )
∆y ≈ dy
Geometrically, ∆x, ∆y, and dy are shown. (T is the tangent line to the
graph of f at P (x, f (x + ∆x)), and x = x1 , x + ∆x = x2 .)
y
y = f (x)
f (x + ∆x)
∆y
dy
P
f (x)
∆x
x x + ∆x x
dy = f 0 (x1 )∆x
x = x1 = 1 and dx = ∆x = x2 − x1 = 2 − 1 = 1
we find
dy = (−3(1)2 − 2(1))(1) = (−3 − 2)(1) = −5
Since ∆y ≈ dy, it follows that ∆y ≈ −5.
Thus, as x changes from 1 to 2, the change in y is approximately −5.
(b) Solution. First, we need to find the derivative of f (x) = xe−2x .
d −2x d d
f 0 (x) = xe = [x]e−2x + x [e−2x ] Product Rule
dx dx dx
d d g(x) d
= (1)e−2x + x e−2x [−2x] e = eg(x) [g(x)]
dx dx dx
= e−2x + xe−2x (−2) = (1 − 2x)e−2x Factor out e−2x
dy = f 0 (x)dx = (1 − 2x)e−2x dx
Evaluate dy at x = x1 = 0 and
= (1 − 2(0))e−2(0) (0.2)
dx = ∆x = x2 −x1 = 0.2−0 = 0.2
= (1 − 0)e0 (0.2) = 0.2
Evaluating dy at
x = x1 = 3 and dx = ∆x = x2 − x1 = 2.9 − 3 = −0.1
we get !
3+1
− ln(3 − 2)
3−2 4 − ln 1
dy = (−0.1) = = −0.025
(3 + 1)2 16
Since ∆y ≈ dy, it follows that ∆y ≈ −0.025.
(a) Solution.
Linear Approximations
Let f be a function, and let a be a number. Suppose f is differentiable at
a. Then the equation of the tangent line to the graph of f at the point
(a, f (a)) is
d 3x+1 d (3x+1) ln 7
f 0 (x) = 7 = e AB = eB ln A
dx dx
d d g(x) d
= e(3x+1) ln 7 [(3x + 1) ln 7] e = eg(x) [g(x)]
dx dx dx
(3x+1) ln 7
=e (3) ln 7 = (3 ln 7)e(3x+1) ln 7
(d) Solution. First we find the derivative of f (x) = log5 (6x + 7).
d d
0 d dx
[6x + 7] d dx
[g(x)]
f (x) = [log5 (6x + 7)] = [loga g(x)] =
dx (ln 5)(6x + 7) dx (ln a)g(x)
6
=
(ln 5)(6x + 7)
Step 1. Find the closest number to b, say a, such that f (a) is exact. By
exact we mean a number
√ that has a finite √ number of decimal
places. For example, 25 is exact while 26 is not.
√
We want to approximate 5 using linear approximations.
Finding√the function
√ and b. The natural function we can consider here is
f (x) = x. Since 5 = f (5), it follows that b = 5.
Finding a. Since the closest number to 5 such that its image under f is
exact and easy to calculate is 4, it follows that a = 4.
Now, we find the linear approximation of f at a = 4. First, f 0 (x) = 1
√
2 x
and
1 1 1
f 0 (4) = √ = =
2 4 2(2) 4
1 1 1
f 0 (49) = √ = =
2 49 2(7) 14
(e) Solution.
Let f (x) = ln x and b = 0.99. Then estimating ln(0.99) amounts to esti-
mating f (b).
348 Chapter 6. Applications of the Derivative II
The closest number to 0.99 such that its image under f (x) is exact and easy
to calculate is a = 1 (ln 1 = 0).
The derivative of f is f 0 (x) = x1 and f 0 (1) = 11 = 1.
The linear approximation of f at a = 1 is
(f ) Solution.
To estimate (−2.001)5 , we consider the function f (x) = x5 and we let b =
−2.001.
The closest number to −2.001 such that its image under f (x) is exact and
easy to calculate is a = −2 ((−2)5 = −32).
The derivative of f is f 0 (x) = dx
d
[x5 ] = 5x4 and f 0 (−2) = 5(−2)4 = 80
The linear approximation of f at a = −2 is
Note that this coincides with the answer found in part (a). This is true in general
when ∆x = 1.
Change in y when ∆x = 1
Since ∆p ≈ dp, it follows that ∆p ≈ −70.57. This means that when the quantity q
changes from 300 to 370 units, the price p decreases by about $70.57.
9. [Exercise on page 50]
Solution.
Let V be the volume of the balloon, and let r be its radius. We want to approx-
imate the change in V , ∆V, if the diameter increases from 10 cm to 10.3 cm (or,
if the radius increases from r1 = 5 cm to r2 = 5.15 cm).
We know that the volume of a sphere of radius r is given by the formula
4
V = V (r) = πr3
3
Taking the derivative of this with respect to r, we get
0 d 4 3 4 d 4
V (r) = πr = π [r3 ] = π(3r2 ) = 4πr2
dr 3 3 dr 3
Substituting r with r1 = 5 into V 0 (r), we have
Thus, if the radius increases from r1 = 5 cm to r2 = 5.15 cm, the approximate change
in volume is 47.12 cm3 .
10. [Exercise on page 50]
Solution. Let A be the area of the field. We want to estimate the change in A if
x = 225 and dx = ∆x = ±0.005. The formula for the area of a square of side x is
A = x2
So the error that might occur when the area of the field is calculated is about
±2.25 ft2 .
Newton’s Method
Let f be a function. Suppose that f is continuous on [a, b]. Also suppose
that f (a) and f (b) have opposite signs (that is, f (a)f (b) < 0). Then, by the
intermediate value theorem, there exists a number c in (a, b) such that f (c) = 0.
In other words, c is a solution to the equation f (x) = 0. Suppose that f
is differentiable on [a, b]. Newton’s method consists of producing successively
better approximations
c1 , c2 , c3 , · · · , cn , cn+1 , · · ·
to c as follows.
Let c1 be an initial guess. Of course, c1 must be chosen in the interval
[a, b].
Suppose an approximation cn to c has been found. If f 0 (cn ) 6= 0, the
number cn+1 is obtained by the formula
f (cn )
cn+1 = cn − (6.6.1)
f 0 (cn )
352 Chapter 6. Applications of the Derivative II
f (c1 )
c2 = c1 −
f 0 (c1 )
f (c2 )
c3 = c2 −
f 0 (c2 )
f (c3 )
c4 = c3 −
f 0 (c3 )
···
Actually, the formula (6.6.1) is obtained by considering tangent lines and look-
ing at where they cross the x-axis. Specifically, let cn be a number in the
interval [a, b], and let Tn be the tangent line to the graph of f at (cn , f (cn )).
The equation of Tn is
The tangent line crosses the x-axis when y = 0. That is, when
The number cn+1 is defined as the point of intersection between the tangent
line Tn and the x-axis. So
f (cn )
cn+1 = cn −
f 0 (cn )
a c1 c c2 b x
T1
f (c1 ) f (0)
c2 = c1 − 0
=0− 0
f (c1 ) f (0)
02 + 0 − 1 −1
=0− =− =1
2(0) + 1 1
f (c2 ) f (1)
c3 = c2 − 0
=1− 0
f (c2 ) f (1)
12 + 1 − 1 1 2
=1− = 1 − = ≈ 0.67
2(1) + 1 3 3
Similarly,
(a) Solution. Here the function is f (x) = 3x2 − 8x − 2, and the interval is [2, 3].
The derivative of f is f 0 (x) = 6x − 8. Let c1 = 2.
f (c1 ) 13 + 12 − 5
c2 = c1 − = 1 − ≈ 1.6
f 0 (c1 ) 3(1)2 + 2(1)
f (c2 ) (1.6)3 + (1.6)2 − 5
c3 = c2 − = 1.6 − ≈ 1.45
f 0 (c2 ) 3(1.6)2 + 2(1.6)
f (c3 ) (1.45)3 + (1.45)2 − 5
c4 = c3 − 0 = 1.45 − ≈ 1.43
f (c3 ) 3(1.45)2 + 2(1.45)
f (c4 ) (1.43)3 + (1.43)2 − 5
c5 = c4 − 0 = 1.43 − ≈ 1.43
f (c4 ) 3(1.43)2 + 2(1.43)
(d) Solution. Let f (x) = x4 + 3x3 − 5x2 − 9, so that f 0 (x) = 4x3 + 9x2 − 10x. First,
let’s approximate c in the interval [−5, −4]. Let c1 = −5.
Let c1 = 1.
Since c7 = c6 , c ≈ 1.5.
(f ) Solution. Let f (x) = x3 e−2x + x2 + 1. The derivative of f is
d 3 −2x d d
f 0 (x) = xe + [x2 ] + [1]
dx dx dx
d 3 −2x d d d
= [x ]e + x3 [e−2x ] + [x2 ] + [1] Product Rule
dx dx dx dx
356 Chapter 6. Applications of the Derivative II
Since f (2) and f (3) have opposite signs, and since the function f is con-
tinuous, it follows by the intermediate value theorem that there exists c in
[2, 3] such that f (c) = 0.
Let c1 = 2 be the initial guess. Then
f (c1 ) f (2) (2)2 − 5
c2 = c1 − 0 =2− 0 =2− ≈ 2.25
f (c1 ) f (2) 2(2)
f (c2 ) f (2.25) (2.25)2 − 5
c3 = c2 − = 2.25 − = 2.25 − ≈ 2.2361
f 0 (c2 ) f 0 (2.25) 2(2.25)
f (c3 ) f (2.2361) (2.2361)2 − 5
c4 = c4 − = 2.2361 − = 2.2361 − ≈ 2.2361
f 0 (c3 ) f 0 (2.2361) 2(2.2361)
√
Since c4 = c3 , we conclude that 5 ≈ 2.2361, to four decimal places.
√
(b) Solution. First, observe that 23 is the positive solution to the equation
x2 − 23 = 0. So we take f (x) = x2 − 23. Evaluating f at 0, 1, 2, · · · , we get
f (0) = −23, f (1) = −22, f (2) = −19, f (3) = −14, f (4) = −7, f (5) = 2
Since f (4) and f (5) have opposite signs, the solution to the equation f (x) = 0
lies in the interval [4, 5]. Let the initial guess be c1 = 4. Then
Since
√ f (3) and f (4) have opposite signs, the solution to the equation f (x) = 0,
3
50, is between 3 and 4. Let c1 = 3 be the initial guess. Then, since f 0 (x) = 3x2 ,
f (c1 ) f (3) (3)3 − 50
c2 = c1 − 0 =3− 0 =3− ≈ 3.8519
f (c1 ) f (3) 3(3)2
f (c2 ) f (3.8519) (3.8519)3 − 50
c3 = c2 − 0 = 3.8519 − 0 = 3.8519 − ≈ 3.6912
f (c2 ) f (3.8519) 3(3.8519)2
f (c3 ) f (3.6912) (3.6912)3 − 50
c4 = c3 − = 3.6912 − = 3.6912 − ≈ 3.6841
f 0 (c3 ) f 0 (3.6912) 3(3.6912)2
358 Chapter 6. Applications of the Derivative II
Since f (2) and f (3) have opposite signs, the solution to the equation f (x) = 0 lies
in the interval [2, 3]. Let c1 = 2 be the initial approximation. Since f 0 (x) = 7x6 ,
we have
f (c1 ) f (2) (2)7 − 139
c2 = c1 − 0 =2− 0 =2− ≈ 2.0246
f (c1 ) f (2) 7(2)6
f (c2 ) f (2.0246) (2.0246)7 − 139
c3 = c2 − 0 = 2.0246 − 0 = 2.0246 − ≈ 2.0237
f (c2 ) f (2.0246) 7(2.0246)6
f (c3 ) f (2.0237) (2.0237)7 − 139
c4 = c3 − = 2.0237 − = 2.0237 − ≈ 2.0237
f 0 (c3 ) f 0 (2.0237) 7(2.0237)6
√
7
Since c4 = c3 , it follows that 139 ≈ 2.0237, to four decimal places.
4. Solution. [Exercise on page 51]
The equation x2 + 3x + 4 = 0 is a quadratic equation with a = 1, b = 3, and c = 4.
Using the quadratic formula, the solution is
√ p √
−b ± b2 − 4ac −3 ± 32 − 4(1)(4) −3 ± −7
x= = =
2a 2(1) 2
√ √
Since −7 is undefined (remember that x is defined for x ≥ 0), the equation
x2 + 3x + 4 = 0 has no solution. In other words, if f (x) = x2 + 3x + 4, there is no c
such that f (c) = 0. So Newton’s method does not work here since it is not possible
to approximate something that does not exist.
5. [Exercise on page 51]
(a) Solution. We want to solve the equation 1 + ln(x − 2) = 0. Subtracting 1 from
both sides, we get ln(x − 2) = −1. Taking the natural exponential on both sides,
we have x − 2 = e−1 . Adding 2 to both sides, we get
x = 2 + e−1 ≈ 2.37
f (c1 ) 1 + ln(2.1 − 2)
c2 = c1 − = 2.1 − 1 ≈ 2.23
f 0 (c1 ) 2.1−2
f (c2 ) 1 + ln(2.23 − 2)
c3 = c2 − 0
= 2.23 − 1 ≈ 2.34
f (c2 ) 2.23−2
f (c3 ) 1 + ln(2.34 − 2)
c4 = c3 − 0
= 2.34 − 1 ≈ 2.37
f (c3 ) 2.34−2
f (c4 ) 1 + ln(2.37 − 2)
c5 = c4 − = 2.37 − 1 ≈ 2.37
f 0 (c4 ) 2.37−2
Since c5 = c4 , it follows that c ≈ 2.37. Note that this matches with the answer
found in part (a).
(c) Solution. Here c1 = 3.
f (c1 ) 1 + ln(3 − 2)
c2 = c1 − 0
=3− 1 =2
f (c1 ) 3−2
The value c3 is undefined because ln(0) (as well as 1/0) is not defined. So
Newton’s method does not work for the equation 1 + ln(x − 2) = 0 if the initial
guess is c1 = 3.
Comment. This question is an illustration of the fact that in some situations,
if the initial guess is “not close” to the exact solution, Newton’s method may
not work.
To find the critical number for f (x), we need to solve the equation f 0 (x) = 0.
Since this is not easy to solve, we will use Newton’s method to approximate the
solution.
First, we need to find an interval [a, b] containing the solution. To this
end, we will use the intermediate value theorem. The idea is to find two
numbers a and b such that f (a) and f (b) have opposite signs. We will use
the trial and error method. To be more precise, we will try the numbers
0, ±1, ±2, · · · until we get two numbers whose images have opposite signs.
– Let’s begin with 0. Substituting x with 0 into f 0 (x), we get
Since f 0 (0) and f 0 (1) have opposite signs, it follows that N = 0 is between
f 0 (0) and f 0 (1). So, since f 0 is continuous, by the intermediate value theo-
rem, there exists a number c in the interval [0, 1] such that f 0 (c) = 0.
We now use Newton’s method to approximate a solution to the equation
f 0 (x) = 0 in the interval [0, 1]. First, the derivative of f 0 (x) is
d 0 d
f 00 (x) = [f (x)] = [4x3 + 3x2 − 2] = 12x2 + 6x
dx dx
As an initial guess, let c1 = 1 (we don’t choose 0 because f 00 (0) = 0). Then
f 0 (c1 ) f 0 (1) 4(1)3 + 3(1)2 − 2
c2 = c1 − = 1 − = 1 − ≈ 0.72
f 00 (c1 ) f 00 (1) 12(1)2 + 6(1)
f 0 (c2 ) f 0 (0.72) 4(0.72)3 + 3(0.72)2 − 2
c3 = c2 − = 0.72 − = 0.72 − ≈ 0.62
f 00 (c2 ) f 00 (0.72) 12(0.72)2 + 6(0.72)
f 0 (c3 ) f 0 (0.62) 4(0.62)3 + 3(0.62)2 − 2
c4 = c3 − = 0.62 − = 0.62 − ≈ 0.61
f 00 (c3 ) f 00 (0.62) 12(0.62)2 + 6(0.62)
f 0 (c4 ) f 0 (0.61) 4(0.61)3 + 3(0.61)2 − 2
c5 = c4 − = 0.61 − = 0.61 − ≈ 0.61
f 00 (c4 ) f 00 (0.61) 12(0.61)2 + 6(0.61)
So, to the nearest hundredth, the critical number is c = 0.61.
(b) Solution. The critical number c ≈ 0.61 determines two intervals: (−∞, 0.61)
and (0.61, ∞). And the sign of the derivative in each of these intervals is shown
in Figure 6.6.
− +
−∞ 0.61 ∞
choose 0 choose 1
f 0 (0) = −2 < 0 f 0 (1) = 5 > 0
From Figure 6.6, the function f is decreasing on the interval (−∞, 0.61) and
increasing on (0.61, ∞). So f has a relative minimum of
when x = 0.61.
7. [Exercise on page 51]
Solution.
6.6. Newton’s Method 361
Set f 0 (x) = 0. Then −xex + 1 = 0. Since this latter equation is not easy to
solve, we will find an approximate solution using Newton’s method.
– First, we need to find an interval [a, b] such that f 0 (a) and f 0 (b) have opposite
signs. To do this, we calculate f 0 (0), f 0 (1), f 0 (−1), f 0 (2), f 0 (−2), · · · until we
get two values of opposite signs.
f 0 (0) = −0e0 + 1 = 1, f 0 (1) = −(1)e1 + 1 ≈ −1.718
Since f 0 (0) and f 0 (1) have opposite signs, and since f 0 is continuous, the
solution to the equation f 0 (x) = 0 lies in the interval [0, 1] (thanks to the
intermediate value theorem).
– Now, the derivative of f 0 (x) is
d d d
f 00 (x) = [−xex + 1] = [−xex ] + [1]
dx dx dx
d d
= [−x]ex − x [ex ] + 0 = (−1)ex − xex = (−1 − x)ex
dx dx
As an initial guess, let c1 = 0. Then
f 0 (c1 ) −(0)e0 + 1
c2 = c1 − = 0 − ≈1
f 00 (c1 ) (−1 − 0)e0
f 0 (c2 ) −(1)e1 + 1
c3 = c2 − = 1 − ≈ 0.68
f 00 (c2 ) (−1 − 1)e1
f 0 (c3 ) (−0.68)e0.68 + 1
c4 = c3 − = 0.68 − ≈ 0.58
f 00 (c3 ) (−1 − 0.68)e0.68
f 0 (c4 ) (−0.58)e0.58 + 1
c5 = c4 − = 0.58 − ≈ 0.57
f 00 (c4 ) (−1 − 0.58)e0.58
f 0 (c5 ) (−0.57)e0.57 + 1
c6 = c5 − 00 = 0.57 − ≈ 0.57
f (c5 ) (−1 − 0.57)e0.57
Since c6 = c5 , the critical number is c = 0.57, to the nearest hundredth.
Froxm the sign of the derivative, which is shown in Figure 6.7, the function
f (x) = (1 − x)ex + x has a relative maximum of
f (0.57) = (1 − 0.57)e0.57 + 0.57 ≈ 1.33
when x = 0.57.
362 Chapter 6. Applications of the Derivative II
+ −
−∞ 0.57 ∞
choose 0 choose 1
f 0 (0) = 1 > 0 f 0 (1) = −e + 1 < 0
Since f (2) and f (3) have opposite signs, and since f is continuous, it follows by
the intermediate value theorem that the solution to the equation f (x) = 0 is in the
interval [2, 3]. Let the initial guess be c1 = 2. Then
1
f (c1 ) 15(14) 3 − 3(2) − 29
c2 = c1 − 0 =2− 2 ≈ 2.54
f (c1 ) 5(14)− 3 − 3
1
f (c2 ) 15(14.54) 3 − 3(2.54) − 29
c3 = c2 − 0 = 2.54 − 2 ≈ 2.54
f (c2 ) 5(14.54)− 3 − 3
Since c3 = c2 , to the nearest hundredth, the break-even quantity is 2.54.
Suppose the savings and costs in dollars of a firm are given by
(a) Sketch the graphs of S and C on the same axes and show that the equation
S(t) = C(t) has only one solution t ≥ 0.
(b) Find the number of years, to the nearest hundredth, that the firm will realize
savings.
(a) Solution.
Sketching the graph of S(t) = t2 + 55t + 30.
– Since t represents time, t can’t be a negative quantity. So the domain
of S is [0, ∞).
– The derivative is S 0 (t) = 2t + 55. Since 2t ≥ 0 and 55 > 0 for all t ≥ 0,
it follows that S 0 (t) is positive for every t ≥ 0. So the function S is
increasing on the interval [0, ∞).
– The second derivative is S 00 (t) = 2. Since S 00 (t) is always positive, it
follows that S is concave upward on [0, ∞).
– Putting all these information together, we get the graph of S as shown
in Figure 6.8.
Sketching the graph of C(t) = t3 + 7t2 + 11.
– As before, the domain of C is [0, ∞).
– The derivative is C 0 (t) = 3t2 + 14t = t(3t + 14). Clearly, this is positive
for all t > 0 (C 0 (t) is zero when t = 0). So the function C is increasing
on (0, ∞).
– The second derivative is C 00 (t) = 6t + 14. Since 6t ≥ 0 and 14 > 0 for
all t, it follows that 6t + 14 > 0 for every t ≥ 0. Therefore, the function
C is concave upward on [0, ∞).
– The graph of C is shown in Figure 6.8.
y
400 C S
300
200
100
2 4 τ 6 8 10 t
Figure 6.8
364 Chapter 6. Applications of the Derivative II
From Figure 6.8, there is only one point of intersection between the graphs of C
and S. This means that the equation S(t) = C(t) has a unique solution.
(b) Solution. The firm will realize savings in the time interval where S(t) ≥ C(t).
Graphically, this means that the firm will realize savings in the interval where
the graph of S is above the graph of C. From Figure 6.8, this happens when
t is between 0 and τ , where τ is the x-coordinate of the point of intersection
between the graphs. And τ is to be found.
By definition, τ is the solution to the equation S(t) = C(t) or S(t) − C(t) =
0. Let
f (t) = S(t) − C(t) = t2 + 55t + 30 − t3 + 7t2 + 11
we have
P 0 (c1 ) P 0 (3) − 57 (3)4 + 6(3)2 + 6(3) + 5
c2 = c1 − = 3 − = 3 − ≈ 3.54
P 00 (c1 ) P 00 (3) − 20
7
(3)3 + 12(3) + 6
P 0 (c2 ) P 0 (3.54)
c3 = c2 − = 3.54 −
P 00 (c2 ) P 00 (3.54)
− 57 (3.54)4 + 6(3.54)2 + 6(3.54) + 5
= 3.54 − ≈ 3.4
− 20
7
(3.54)3 + 12(3.54) + 6
P 0 (c3 ) P 0 (3.4)
c4 = c3 − = 3.4 −
P 00 (c3 ) P 00 (3.4)
− 57 (3.4)4 + 6(3.4)2 + 6(3.4) + 5
= 3.4 − ≈ 3.39
− 20
7
(3.4)3 + 12(3.4) + 6
P 0 (c4 ) P 0 (3.39)
c5 = c4 − = 3.39 −
P 00 (c4 ) P 00 (3.39)
− 57 (3.39)4 + 6(3.39)2 + 6(3.39) + 5
= 3.39 − ≈ 3.39
− 20
7
(3.39)3 + 12(3.39) + 6
So, to the nearest hundredth, the critical number is 3.39. This determines two
intervals: (0, 3.39) and (3.39, ∞). The sign of P 0 in these intervals is shown in
Figure 6.9.
+ −
0 3.39 ∞
choose 1 choose 4
P 0 (1) = 16.29 > 0 P 0 (4) = −57.86 < 0
From Figure 6.9, the number of kilograms, to the nearest hundredth, that should
be sold in order to maximize the total profit is 3.39 kilograms.
(b) Solution. The maximum profit is
1
P (3.39) = − (3.39)5 + 2(3.39)3 + 3(3.39)2 + 5(3.39) − 4 = 61.3839
7
Since the profit is in thousands of dollars, the maximum profit is approximately
$61383.9.